FNP Pediatric Exam Bank; complete A+ guide latest fall 2022/2023.

The concept of health promotion consists of efforts to prevent rather than to cure disease or disability. This description best describes:
a. tertiary prevention
b. secondary prevention
c. primary prevention
d. morbidity prevention
c

Standards for well child care and health promotion are set forth by the:
a. Task Force on Preventative Services, US Department of Health and Human services
b. American Academy of Family Physicians
c. American Academy of pediatrics
d. Healthy People 2010 and 2020
c

A 15 yo female comes to the clinic for a health maintenance visit. The patient reports no complaints. She does well in school, has many friends, and gets along well with her parents and two siblings. She has recently become sexually active with her boyfriend of 6mo. The physical exam is normal for her age. What screening, lab tests, and/or immunizations should the PNP order?
a. Vision, hearing, urinalysis, STD screen, pelvic exam, and immunizations as needed
b. STD screen, pneumococcal vaccine, pelvic exam, and cholesterol screening as indicated
c. vision, hearing, STD screen, pelvic exam, and hemoglobin if indicated.
d. Measles vaccine, vision, hearing, STD screen, pelvic exam exam, and cholesterol screen if indicated.
a

Which of the following is the best written chief complaint?
a. 5yo asian female with itchy red rash
b. 1 mo with difficulty feeding
c. 15yo african american female complaining of lower abdominal pain for 3 days
d. 12yo caucasian with itchy eye for 1 week
c

The past history should include
a. family history
b. social hx
c. symptom analysis
d. hospitalization
d

The family history should include:
a. the health status of the paternal grandparents
b. the living conditions of the family
c. the work status of the parents
d. school information
a

Which of the following statements would be considered subjective data obtained from the client during the review of systems for the skin?
a. the skin appears dry & excoriated
b. a lesion is noted on the lateral aspect of the right leg
c. there is moderate acne on the forehead and back
d. the patient denies changes in skin texture
d

The PNP is completing the ROS on a 4mo. What response from the mother might indicate a cardiac problem in the infant & require a more thorough history?
a. the baby gets the hiccups often.
b. it takes the baby over 30min to complete a bottle
c. the baby’s heart seems to beat rapidly sometimes
d. sometimes the baby sounds congested
b

The PNP is obtaining a review of systems on a healthy 8yo girl. It would be important to include:
a. the date of last breast self-exam.
b. any limitations related to sports participation.
c. the last hematocrit and hemoglobin
d. the last lead screen
b

The mother of a 3yo states that her son was completely toilet trained at 18mo. The PNP explores further, asking:
a. how the mother did the toilet training
b. if the child is dry at night and fully trained for urine and stool day and night
c. at what age did the mother begin toilet training the other child.
d. if the child gets up at night to use the bathroom
b

Jasmine, a 5-month-old infant is in the clinic for a well child visit and is sleeping in her mother’s arms. What should the PNP do first?
a. Examine the ears, since this is uncomfortable for the infant.
b. Quickly give any needed immunizations.
c. Begin with assessment of the head and proceed from head to toe.
d. auscultate the heart & lungs while the infant is sleeping.
d

The mother of a newborn brings the baby to the clinic because she is afraid something is wrong with her baby (“his head is so big”). The PNP responds that:
a. “this is normal. The head of a newborn is proportionately large to the rest of the body”
b. “Yes, this does appear abnormal. I will make an appointment for the baby to be seen by a neurologist.”
d. “your head appears large so it is probably genetic.”
a

A healthy 12yo female is at the clinic for a well checkup. On physical examination, a marked elevation of the right scapula and right thoracic hump and spinal curve abnormally are noted. Spinal films indicate 20 degree curve. The PNP should:
a. refer the patient to an orthopedist
b. monitor the patient every 3 mo until menarche
c. refer the patient for physical therapy
d. recommend bedrest and back brace
a

What is the % of risk an autosomal dominant gene is being passed on to each offspring?
a. 25
b. 50
c. 100
d. none
b

A 13yo female is at the clinic for a routine check up. What presenting symptoms might alert the PNP to the presence of a possible genetic disorder?
a. <10% on growth chart, Tanner I, and learning difficulties
b. Precocious puberty, developmental delays, gifted piano player
c. Lactose intolerance, frequent ear infections, speech delays
d. Peanut allergy, right-sided weakness, and exhibits bullying behavior
a

The parents of a child with cystic fibrosis are considering another pregnancy. They want to know what their chances are of having another child with cystic fibrosis. The PNP explains that cystic fibrosis is an autosomal recessive disorder and that each conception of carrier parents has a:
a. 25% chance of being affected
b. 50% chance of being affected
c. 75% chance of being affected
d. 100% chance of being affected
a

The parents of a son with hemophilia are considering another pregnancy. The mother has been identified as a carrier of the hemophilia gene. What chance does each female offspring have of having hemophilia.
a. 100%
b. 50%
c. 25%
d. none
d

A male with Vitamin D-resistant rickets asks what his chances are of passing his disease to his parents. The PNP answers:
a. all sons will be affected by only 25% daughters
b. all daughters are affected by only 25% sons
c. 50% of all offspring will have the disease
d. all daughters will be affected but no sons.
d

The PNP is following a 15-year-old male adolescent with consistent blood pressure readings of 132 to 138/84 to 86 mm Hg, which is classified as significant hypertension. After performing a workup, the PNP determines that the adolescent has primary hypertension. The most judicious recommendation for therapy is:
A) Diet and exercise counseling and referral to a specialist
B) A diuretic, low-salt diet, exercise, and counseling by a dietician
C) Perform an extensive family history to determine other risk factors
D) A vasodilator, restricted activity, and a low-fat diet
a

The PNP is examining a neonate with a heart murmur. The S2 sound is loudest at the apex. The respiratory rate is 65 breaths per minute, and the heart rate is 180 beats per minute. Which of the following would be an appropriate action by the PNP?
A) Reevaluate the neonate in 24 hours
B) Increase the number of oral feedings
C) Order cardiac catheterization
D) Refer the neonate to a cardiologist
d

A 2-week-old neonate is brought to the clinic with excessive lacrimation of both eyes. The eyes are clear, with no areas of redness, but the corneas appear hazy. The PNP explains to the parents the possible diagnosis and treatment as follows:
A) Congenital glaucoma requiring an ophthalmology referral
B) Corneal foreign body requiring fluorescein staining and removal
C) Chalazion requiring treatment with antibiotic eyedrops
D) Allergic conjunctivitis requiring treatment with Benadryl eyedrops
a

On examination of a 3-month-old infant the PNP is unable to elicit a red reflex in the right eye. Previous examinations failed to note the presence or absence of a red reflex. The PNP’s response is to:
A) Reassure the parents that this is not a problem because the infant has dark eyes
B) Note this finding in the infant’s chart and check again in a few months
C) Refer the infant to an ophthalmologist to rule out retinal trauma
D) Immediately refer the infant to a pediatric ophthalmologist to rule out a congenital cataract or retinoblastoma
d

A 4-year-old child with chickenpox has vesicles on the skin of the right eyelid. The child complains of eye pain and blurred vision. The PNP should treat the condition by:
A) Applying cool compresses to the eye and lesions
B) Prescribing eyedrops containing steroids to decrease inflammation and pain
C) Prescribing polyspoium ophthalmic ointment for the secondary bacterial infection
D) Immediately referring the child to an ophthalmologist
d

An adolescent is being seen at a community health center because of recurrent respiratory tract infections. The complete blood cell count (CBC) with differential shows a white blood cell count of 20,500 with 35% blast cells. The next step in managing the adolescent’s care is to:
A) Repeat the CBC with differential in 1 week
B) Refer to a specialist in pediatric hematology
C) Perform bone marrow aspiration
D) Hospitalize the adolescent immediately
b

A 9-month-old infant was diagnosed with sickle cell disease shortly after birth. The mother telephones the PNP to report that the infant has a fever of 103.2°F. The best response to the mother is:
A) “Take the infant to the emergency room immediately.”
B) “Administer a dose of ibuprofen, and call back in 6 hours if the fever continues.”
C) “Give extra fluids and acetaminophen, and call back tomorrow if the fever continues.”
D) “Give extra fluids and acetaminophen, and bring the infant to the clinic tomorrow morning.”
a

A 5-year-old child has sudden onset of nonblanching purpuric lesions scattered over the body and petechiae scattered over the neck and shoulders. The mother reports that the child has been healthy, except for a cold a few weeks ago. The child is not taking any medications. Physical examination reveals a healthy, afebrile child with no other significant findings. The laboratory data show a hemoglobin level of 12.5 g/dL, white blood cell count of 6500/mm3, and platelet count of 20,000/mm3. Based on this information, what should the PNP do next?
A) Reassure the parents that these findings are consistent with acute idiopathic thrombocytopenia purpura (ITP), and advise a hematology consultation for confirmation
B) Refer the child immediately to the pediatric hematology/oncology department of the nearest tertiary care center
C) Report the family to the local protective services department as soon as possible because of the possibility of child abuse
D) Order additional laboratory tests, including bleeding studies, an autoimmune panel, and an Epstein-Barr titer; more information is needed before a diagnosis can be made
a

A 4-year-old child is scheduled for a tonsillectomy and adenoidectomy. The preoperative laboratory tests indicate a prolonged active partial thromboplastin time (aPTT). The PNP should suggest that they:
A) Continue with the surgery, and monitor the child closely for bleeding complications
B) Cancel the surgery, and recheck the aPTT in 1 week
C) Cancel the surgery, and refer the child to a hematologist
D) Obtain a family history, and determine whether there are other relatives with a bleeding disorder
c

A 14-year-old adolescent, who appears to be in acute distress and is anxious, is brought to the clinic with symptoms of high fever, chills, malaise, pharyngitis, vomiting, peripheral cyanosis, tachypnea, tachycardia, low blood pressure, and erythroderma. The PNP recognizes this as toxic shock syndrome and:
A) Orders a CBC and blood culture immediately
B) Orders a CBC and blood culture, and sends the adolescent to an emergency room for a lumbar puncture
C) Sends the adolescent to the emergency room by ambulance immediately without providing any treatment in the clinic
D) Collaborates with the clinic physician to determine appropriate antibiotic use in this patient
c

The PNP is assessing a neonate in the nursery. The cremasteric reflex is absent, and the right testicle cannot be palpated in the scrotum. The PNP should:
A) Order a sonogram
B) Consult with the physician
C) Wait for the next well visit to see if the situation has changed
D) Order an MRI
b

A 9-year-old child is brought to the clinic for the evaluation and treatment of a rash. While examining the child, the PNP detects a speech dysfluency. The most appropriate plan of treatment should include:
A) Treating the rash and scheduling a well-child visit to address the speech problem
B) Treating the rash and making a referral for speech, language, and hearing evaluation with a speech pathologist
C) Treating the rash and making no referral because it is too late for speech therapy to be beneficial
D) Using a clinical screening tool to determine whether a language disorder exists
b

A 13-year-old adolescent complains of having difficulty walking and the arms feeling weak. The adolescent had a cold 2 weeks ago. On examination, the PNP is not able to elicit deep tendon reflexes. The PNP knows that the most appropriate course of action is to:
A) Refer the adolescent to an orthopedic surgeon
B) Schedule a follow-up visit in 1 week
C) Refer the adolescent to a neurologist
D) Refer the adolescent to an emergency room
d

A 1-year-old child is brought to the clinic with a temperature of 102°F and left flank pain. The urine dipstick test indicates nitrates and leukocyte esterase. The presenting signs and symptoms suggest left pyelonephritis. Based on the data, the most appropriate action for the PNP is to:
A) Refer the child to a urologist for diagnosis and treatment
B) Consult with a physician
C) Provide symptomatic treatment for 24 hours, and repeat the urine dipstick test
D) Send urine for culture and sensitivity testing
b

A neonate is diagnosed with trisomy 21 based on karyotyping. What type of follow-up will the infant need?
A) Echocardiography; thyroid function tests at birth, 3 months, and yearly thereafter; a CBC; and an audiology consult
B) An ophthalmologic evaluation, neck x-ray films by age 3 years, and referral to early intervention and parent education
C) Karyotyping, echocardiography, audiologic and ophthalmologic evaluation, and referral to early intervention and parent education
D) Karyotyping; echocardiography; renal ultrasonography; thyroid function tests at birth, age 3 months, and yearly thereafter; and referral to early intervention and parent education
a

The parents of a 9-year-old with primary enuresis request information regarding treatment options. When discussing alternative treatments for enuresis, the PNP offers pertinent information to help the child and parents make an appropriate decision about which treatment would be best. The family should be told:
A) The alarm is the safest therapy, but the relapse rate is about 10%
B) Pharmacologic therapy has the lowest relapse rate
C) Motivational therapy should be used after pharmacologic treatment
D) Treatment should begin with bladder awareness training
d

A parent asks for suggestions in helping a 6-year-old child who wets the bed. What intervention would the PNP recommend for this child diagnosed with primary nocturnal enuresis?
A) Use a “wet night” calendar to mark the dates of wetting accidents
B) Allow the child to take care of changing wet clothes and linens
C) Criticize the child when a wetting accident has occurred
D) Praise any progress made by the child
d

A 10-year-old child is examined because of recurrent UTIs. A urologic workup is performed. No abnormalities are found. To help prevent future UTIs, the PNP should suggest which of the following interventions?
A) Taking a 30-minute bath daily
B) Avoiding showering
C) Using a voiding schedule to expand the bladder
D) Practicing good perineal hygiene
d

A PNP responsible for neonatal discharge rounds at the hospital examines a male infant and notes that the urethral opening appears displaced ventrally along the glans. A closer assessment reveals an undiagnosed mild hypospadias. What should the parents be told?
A) Hypospadias occurs in approximately 1 in 500 neonates
B) The infant should be evaluated for other anomalies of the upper urinary tract
C) The infant should be assessed for undescended testes and inguinal hernia
D) Routine circumcision should be performed by 6 weeks of age
c

A 15-year-old adolescent has pallor and fatigue. The CBC results are consistent with iron-deficiency anemia. What would be appropriate information to give this adolescent?
A) Antacids increase the absorption of iron
B) Dairy foods are good sources of dietary iron
C) Juices fortified with vitamin C inhibit the absorption of nonheme iron
D) Tannin-containing products, such as tea, inhibit the absorption of nonheme iron
d

When discussing the cause of impetigo with the parents of a child just diagnosed, the PNP tells them it is caused by:
A) Klebsiella species or group A beta-hemolytic streptococci (GABHS)
B) Proteus species or anaerobes
C) Staphylococcus aureus or GABHS
D) Escherichia coli or candidal organisms
c

The PNP examines a 4-year-old child who is home-schooled and immunization-delayed. The child has pain when chewing; a fever; and enlarged, tender salivary nodes. The PNP diagnoses mumps and informs the mother that mumps is contagious for:
A) 7 days before the onset of symptoms
B) 10 days before the onset of symptoms
C) 1 to 2 days before and 5 days after
D) As long as 3 days after the onset of symptoms
a

A 10-year-old child is brought to the clinic after jumping over a rusty fence and receiving a puncture wound to the buttocks. Immunizations are up to date with last tetanus given when child was 4 years old. The PNP cleanses the wound and:
A) Administers a tetanus vaccine and prescribes penicillin for 14 days
B) Starts prophylactic antibiotics
C) Applies a sterile dressing
D) Administers tetanus vaccine and educates family regarding signs of wound infection
d

A 17-year-old adolescent comes to the office to ask about a self-medication regimen for monthly menstrual cramps. The adolescent reports routinely taking 1200 mg of ibuprofen every 4 to 6 hours for the first 3 days of menses each month to relieve cramps. The PNP tells her:
A) “The dose is okay because it is only a few days a month.”
B) “Discontinue the medication because it may impair renal or liver function.”
C) “The dose should be decreased because of the risk of gastrointestinal ulceration.”
D) “Aspirin is a better medication for relieving menstrual cramps.”
c

A 2-month-old infant is brought to clinic with a stuffy, runny nose of 3 days’ duration. The infant is afebrile and has no cough or respiratory difficulty but is not nursing or sleeping well. The tympanic membranes are within normal limits. What advice would the PNP give?
A) Administer amoxicillin suspension 125 mg/5 mL, 1 tsp three times a day for 10 days
B) Elevate the head of the bed, administer saline nose drops, and use a room humidifier
C) Investigate allergic overload in the home; administer amoxicillin and Neosynephrine nose drops
D) Change the infant’s formula to a soy-based formula
b

A 2-year-old child is brought to the clinic with a 2-day history of a harsh, predominantly nocturnal cough, fever of 100°F, and clear rhinorrhea. The child is diagnosed with croup. Which of the following would the PNP tell the parent?
A) “The cold symptoms should be gone in 3 days.”
B) “If the child is unable to swallow and begins to drool, call the office.”
C) “A high fever is normal for the first 2 days of a cold.”
D) “An antibiotic needs to be prescribed.”
b

A mother calls the office and asks what she can do to decrease the discomfort her 9-year-old child is experiencing with an upper respiratory tract infection. The mother should be instructed to:
A) Administer pseudoephedrine and normal saline nose drops
B) Administer normal saline nose drops and benzonatate
C) Place a cool-mist humidifier in the child’s room, and administer benzonatate
D) Administer dextromethorphan and normal saline nose drops
a

A grandfather tells the PNP that with winter coming he is planning to buy several bottles of cough syrup and treat his family members when they get colds and coughs. Although the PNP discourages this practice, the guidelines the PNP provides should include a warning that cough suppressants should be used only for:
A) Coughs that occur in the morning
B) Coughs that occur during exercise
C) Harsh, “brassy” coughs
D) Temporary dry coughs
d

A father asks the PNP if an opioid cough suppressant, such as promethazine hydrochloride with codeine (Phenergan with codeine), would help his school-aged child who is recovering from a cold and complaining of a “nagging cough.” The PNP explains that possible problems of opioid cough suppressants include:
A) Potential for abuse and diarrhea
B) Respiratory depression and diarrhea
C) Dependency and constipation
D) Respiratory stimulation and constipation
c

The mother of a 10-week-old calls the office because the infant is passing hard stools. What suggestions would the PNP offer a mother?
A) “Do nothing. Time will take care of the problem.”
B) “Hold the infant upright so gravity can help the stool pass.”
C) “Give a little extra water with added pasteurized apple or prune juice.”
D) “Add honey to a 4-oz bottle of water.”
c

A parent has questions regarding the use of over-the-counter fever medications for a 2-year-old. The child has had a temperature of 101.2°F with no other symptoms of illness for the past day. The mother has been treating the fever by alternating doses of acetaminophen and ibuprofen every 3 to 4 hours. The child is playful and has a normal appetite. Appropriate counseling for this mother should include:
A) Continue giving the medicines every 3 to 4 hours at the appropriate dosage for the child’s weight until the fever subsides
B) Increase the interval of the medications to every 6 hours
C) This amount and frequency of antipyretics may be inappropriate and harmful in a child whose only symptom is a low-grade fever
D) Stop the medications, because a temperature of 101.2°F in a child with no other symptoms does not necessitate the use of antipyretics
c

A 14-month-old is brought to the clinic with a cough and yellow-green nasal discharge for the past 7 days. The father has been giving the child over-the-counter cold medications every 6 hours. The child has no fever. Appetite, activity, and elimination are unchanged. The father states he had the same symptoms and was put on an antibiotic for a sinus infection. He is concerned that the child has the same infection. The most appropriate action by the PNP is to:
A) Order a Water’s view sinus x-ray film to rule out acute sinusitis
B) Prescribe an appropriate antibiotic to treat the sinus infection
C) Reassure the father that the child probably has a viral infection and suggest supportive care; if symptoms don’t improve in 7 days, return for evaluation
D) Inform the father that no treatment is necessary but suggest she bring the child in if symptoms do not improve within 4 weeks
c

The PNP returns a phone call to the mother of a 2-week-old infant. The infant is breastfed exclusively and at the 1-week checkup weighed 4 oz more than at birth. The infant has not had a bowel movement in 4 days but has had more than eight wet diapers each day, is nursing vigorously, and has no signs or symptoms of illness. Appropriate counseling for this mother should include:
A) Because there has been no bowel movement in 4 days, a glycerin suppository should be administered
B) Rectal stimulation should be performed every hour until a bowel movement is produced
C) No treatment is necessary; the mother should be reassured only
D) The infant should be evaluated in the gastroenterology clinic
c

While examining a 3-year-old child for a well-child visit, labial adhesions are noted. The PNP discusses with the mother:
A) Concern of sexual abuse
B) Proper hygiene
C) Causes of UTI
D) Surgical treatment of the adhesions
b

A 10-month-old infant is brought to the clinic for a rash. The rash is circumscribed, dry, pruritic, slightly flaky, and located on the cheeks and flexor surface of the arms. What treatment should the PNP prescribe?
A) Bathe the infant frequently, and apply Vaseline to the rash
B) Bathe the infant once a week with Dial soap and apply Gold Bond cream to the lesions
C) Bathe the infant daily with mild soap, such as Dove, and apply Eucerin cream daily
D) Bathe the infant daily or every other day with Dove soap and apply a high potency corticosteroid cream, paying particular attention to the cheeks
c

In providing anticipatory guidance to the parents of a 5-year-old child with diabetes, the PNP should teach them to recognize the symptoms of sweating, hunger, drowsiness, and confusion. These are all symptoms of:
A) Diabetic ketoacidosis
B) The dawn phenomenon
C) Hypoglycemia
D) The Somogyi phenomenon
c

An 18-month-old child is brought to the office for irritability, nightmares, and generally being tired for the past few nights. The child has had an upper respiratory tract infection for which the grandmother has administered an over-the-counter medication for rhinorrhea. The child is afebrile and alert, with obvious clear rhinorrhea. The PNP:
A) Orders sinus radiologic studies
B) Prescribes amoxicillin
C) Discusses side effects of decongestants and antihistamines
D) Suggests that the grandmother administer acetaminophen
c

A 16-year-old presents with pallor and fatigue for the past several weeks. The complete blood count (CBC) results are consistent with iron deficiency anemia. What should the PNP tell this teen?
A) Antacids increase the absorption of iron
B) Dairy foods are good sources of dietary iron
C) Juices fortified with vitamin C inhibit the absorption of nonheme iron
D) Tannin-containing products, such as tea, inhibit the absorption of nonheme iron
d

A PNP has been asked by the emergency room physician to evaluate a child for costochondritis. The PNP would expect to find:
A) Tenderness of the midsternal area
B) Atelectasis on chest x-ray film
C) Dullness on percussion
D) Muffled heart sounds
a

The murmur heard when a child has rheumatic heart disease is the result of:
A) Myocarditis
B) Pericarditis
C) Valvulitis
D) Coronary artery involvement
c

A 16-year-old adolescent has evidence of weight loss. The PNP notes on the chart documentation of frequent loose stools for 2 weeks. The differential diagnosis includes hyperthyroidism. What additional clinical finding would suggest this diagnosis?
A) Dry mouth
B) Hypotension
C) Coarse hair
D) Heat intolerance
d

The PNP evaluates an infant born to a mother known to have autoimmune thyroid disease. Congenital hypothyroidism in infants is associated with which of the following?
A) Excessive sleepiness
B) Failure to thrive
C) Tachycardia
D) Diarrhea
a

Which of the following findings suggests a need for further evaluation for glomerular disease?
A) Blood pressure of 135/85
B) +1 protein and red blood cell (RBC) casts in the urine
C) Bright red urine
D) A urine culture of 100,000 bacteria/mL
b

An 8-year-old child is brought to the urgent care clinic. No chart is available for the history. The child’s symptoms include tea-colored urine and periorbital edema, and the blood pressure is 142/90. There is no dysuria or frequency. Poststreptococcal acute glomerulonephritis (PSAGN) is suspected. Which of the following statements is correct?
A) PSAGN typically begins 28 days after a streptococcal infection
B) Appropriate antibiotic treatment prevents nephritogenic group A beta-hemolytic streptococcal (GABHS) infection
C) The urine test reveals no RBC casts or proteinuria
D) Treatment in the acute phase may require salt restriction, diuretics, and antipyretics; the expected outcome is good
d

An 8-year-old child with spina bifida is examined in the clinic. The mother states that the child was doing well until attending a party at school today. Physical examination reveals a cooperative child with mild, clear rhinorrhea; red, watering eyes; generalized hives; edema of the face and eyes; and no acute respiratory symptoms. What would be the most appropriate question to ask when looking for the possible cause of an allergic reaction in this child?
A) “What foods did you eat?”
B) “Have you been exposed to anything new?”
C) “Were there any balloons at the party?”
D) “Did a bee or insect sting you?”
c

A 12-year-old child is seen in the school-based clinic because of a red, swollen, tender area on the upper eyelid. The PNP prescribes:
A) A topical steroid to be applied to the inflamed eyelid
B) Warm compresses to be applied two to three times a day and a topical antibiotic
C) A systemic antibiotic for 2 weeks
D) A systemic antihistamine and a steroid cream
b

A 14-year-old adolescent has an annular rash (erythema migrans) on the legs with some clearing in the center of the lesions. The rash has appeared and disappeared over the last few weeks. The adolescent also complains of fever, myalgia, and headache. Which of the following subjective data would be most helpful in making a diagnosis for this condition?
A) The rash is pruritic and feels warm
B) The adolescent recently went backpacking in wooded area
C) A sore throat developed 3 days ago
D) The adolescent went on a hiking trip 3 days ago in Southern California
b

A 2-month-old infant has symptoms of a severe cough followed by vomiting. The infant had an upper respiratory tract infection 2 weeks before the onset of coughing. The PNP suspects pertussis and orders a chest x-ray film and culture of the nasopharynx. The PNP refers the infant for hospitalization and begins treatment with:
A) Erythromycin
B) Trimethoprim/sulfamethoxazole
C) Corticosteroids and beta-agonist aerosol
D) Amoxicillin
a

A 15-year-old adolescent is brought to the clinic with jaundice, anorexia, and nausea. The adolescent is concerned that she may be pregnant because she has nausea and has had unprotected sex. The PNP suspects that the diagnosis may be:
A) Infectious mononucleosis
B) Hepatitis B
C) Chronic fatigue syndrome
D) Viral gastritis
b

A complete blood count (CBC) is obtained for a 6-year-old child with a rash and systemic symptoms of fever and general malaise. The results are all within normal limits except the eosinophil count, which is elevated. This finding supports the diagnosis of:
A) Allergic reaction
B) Bacterial infection
C) Viral exanthem
D) Rheumatic fever
a

A child is brought to the clinic after being hit in the head with a hockey stick. The child now has profuse rhinorrhea but is in no distress. The examination reveals a hemotympanum, which is indicative of:
A) A foreign body in the canal
B) Perforation of the tympanic membrane
C) A basilar skull fracture
D) Acute serosanguineous otitis media
c

A 2-year-old child who may have swallowed a watch battery is brought to the clinic. The child has no symptoms of ingestion of a foreign body. The initial action for the PNP is to:
A) Obtain an x-ray film of the chest and abdomen to locate the battery
B) Refer immediately for a surgery consult and possible removal of the battery
C) Send the parents home with instructions to watch for the battery in the stool
D) Send the parents home with instructions to return if the child experiences abdominal pain
a

A child is brought to the office with irritability, agitation, muscle pain, and cramping in the large leg muscles. On examination a target lesion consisting of an erythematous ring around a pale center is noted. The child has been playing in the garage and does not recall seeing or being bitten by an insect. The child’s face is flushed, and there is some diaphoresis. The PNP suspects:
A) An allergic reaction to flea bites
B) A black widow spider bite
C) A brown recluse spider bite
D) Ingestion of weed pods
b

A 3-year-old child is brought to the clinic with noisy respirations. Which of the following signs or symptoms would be an indication for immediate referral and hospitalization?
A) Mild restlessness
B) Stridor at rest
C) Fever of 102°F
D) Slightly elevated white blood cell count
b

A 10-year-old child is brought to the office with coughing, abdominal pain, splinting on the right side, fever, headache, and chills. The child appears to be in severe distress. There are decreased breath sounds, crackles, and dullness to percussion in the area of the right middle and right lower lobe. The diagnosis of pneumonia is made. The PNP orders an anteroposterior and lateral view of the chest and:
A) Sputum stain
B) Blood culture
C) Complete blood cell count and blood culture
D) Blood gases and pulse oximetry
c

A 12-year-old child comes to the school-based clinic complaining of unilateral chest pain that radiates to the back and abdomen. On physical examination, tenderness over the costochondral junction is noted. The chest pain is most likely caused by:
A) A fractured rib
B) Gastric reflux
C) Costochondritis
D) Pneumonia
c

A 10-month-old infant is noted to have cranial and facial asymmetry during a well-child visit. The PNP should:
A) Observe the infant, and reevaluate cranial shape at the 1-year well visit
B) Refer immediately to a neurologist
C) Obtain a cranial radiologic film, CT scan of the skull, or both
D) Perform a Denver Developmental Screening Test
c

A 15-year-old adolescent who has joint pain related to juvenile rheumatoid arthritis has been taking NSAIDs to relieve the pain. The adolescent came to the school-based clinic with epigastric and abdominal pain. The PNP suspects that the pain is related to the NSAIDs and manages the symptoms by:
A) Modifying the adolescent’s diet
B) Substituting salicylates for NSAIDs
C) Treating for H. pylori
D) Administering a H2 receptor antagonist
d

A 9-year-old child has circumscribed, coalescent, generalized, erythematous raised lesions of various sizes. The PNP recognizes that the most common cause of hives is:
A) Sun exposure
B) Cold exposure
C) Ingestion of foods and food additives
D) Insect bites
c

A 10-month-old infant is brought to the clinic because of an acute illness. On examination the PNP notes a large, bulging umbilical mass. The mass is easily reducible although prominent when the child cries. The PNP:
A) Refers the infant to a surgery clinic for evaluation of the hernia and treats the acute illness
B) Treats the acute illness, comments on the hernia, and measures the abdominal opening
C) Discusses with the parents the need to apply a bellyband to “hold in” the hernia
D) Explains to the parents that surgical repair should be performed when the child is between age 2 and 3 years
b

A 2.5-year-old child has anal itching and has been awakening at night. The PNP diagnoses:
A) Sexual abuse
B) Pinworms
C) Hemorrhoids
D) Rectal fistula
b

The PNP is evaluating an 8-year-old child with an eye injury. About 1 hour ago the child was hit in the eye with a foreign body. When the eye is stained with fluorescein, a vertical linear abrasion is visible. The pupils are equal in size and reactive, and a positive red reflex is noted bilaterally. There are no abnormal extraocular movements. No foreign body is visible. An appropriate plan of care for this patient would include the following:
A) Close both eyes, and apply eye patches
B) Instill ophthalmic steroid drops three times a day, but do not apply an eye patch
C) Apply antibiotic ointment, but do not apply an eye patch
D) Instill antibiotic drops, and apply patches over both eyes
c

A 4-year-old child is brought to the clinic with ulcers on the tongue and oral mucosa. There are vesicles on the palms, soles, and interdigital areas. The PNP diagnoses:
A) Rubeola
B) Infectious mononucleosis
C) Herpangina
D) Hand-foot-and-mouth disease
d

The PNP discusses the use of trimethoprim/sulfamethoxazole (Bactrim) in infants 2 months old and younger with the PNP student. The antibiotic should not be prescribed in the infant because:
A) The infant has low renal clearance
B) The infant has rapid renal clearance
C) The drug causes decreased gastric emptying
D) The infant has an immature liver and protein binding ability
d

A 16-year-old adolescent comes to the school-based clinic with chest pain. The most common cause of cardiac pain in children and adolescents is:
A) Myocarditis
B) Tachycardia
C) Mitral valve prolapse
D) Preventricular contractions
c

The most sensitive physical finding for the diagnosis of testicular torsion is:
A) The alleviation of pain when the testis is elevated
B) Pain lasting for hours or days
C) Abnormal results of urine culture
D) Absence of the cremasteric reflex
d

A 14-year-old adolescent has pain in the knee. The pain increases with activity and is relieved with rest. The PNP diagnoses Osgood-Schlatter disease and orders:
A) An x-ray examination, application of hot packs to the knee, and rest
B) Application of hot packs to the knee, aspirin, and rest
C) A reduction in activity, application of ice to the knee, and ibuprofen
D) Application of ice to the knee and continued participation in sports
c

The PNP in the Emergency Departmetnt is evaluating a 3-year-old child with a history of sudden onset of high fever and stridor. The child refuses to move and is sitting upright, leaning forward with the mouth open, and drooling. The safest and most helpful diagnostic test to order now would be:
A) Urine culture
B) A lateral neck x-ray film
C) A chest x-ray film
D) A throat culture
b

In evaluating a 7-year-old child for ADHD, the PNP questions the parents about reported behavior at school. Of the following symptoms, which would be most descriptive of ADHD?
A) The child will not listen to direction or when spoken to, submits messy papers with careless mistakes, runs about or climbs excessively, has difficulty sustaining attention, and exhibits intrusive behavior
B) The child has a short attention span, has difficulty listening, has difficulty organizing tasks, and interrupts others who are talking
C) The child seems to want attention, has a short attention span for schoolwork, talks out of turn, fidgets a lot, and interrupts others
D) The child wants attention, makes careless mistakes in schoolwork, has difficulty taking turns, and fidgets a lot
a

A 6-year-old child with asthma is brought to the clinic because the parents have noticed daily wheezing for the past week, especially during the night. The PNP recognizes that this may indicate the child has:
A) Intermittent asthma
B) Mild persistent asthma
C) Moderate persistent asthma
D) Severe persistent asthma
c

A 7-year-old child with cerebral palsy is brought to the clinic. The child is unable to walk but is able to use a wheelchair. The child uses the arms and hands to eat and perform self-care tasks. The PNP diagnoses this child with:
A) Spastic diplegia
B) Spastic quadriplegia
C) Athetoid cerebral palsy
D) Spastic hemiplegia
a

The parents of a 3-year-old child have noticed that the child experiences episodes of abrupt, brief loss of consciousness many times during the day. The PNP suspects that the child is having:
A) Status epilepticus
B) Absence seizures
C) Atonic seizures
D) Myoclonic seizures
b

A 3-month-old infant is brought to the clinic for a well-child visit. The mother is concerned because the infant has not been feeding as well as usual during the past few days. On examination of the mouth the PNP notes white, cottage cheese-like patches on the inner aspect of the cheeks. The PNP plans to treat the lesions with:
A) Acyclovir
B) Erythromycin suspension
C) Nystatin oral suspension
D) Hydrogen peroxide soaks
c

A mother brings a 1-year-old child with Down’s syndrome to the clinic. She is concerned that the child has been congested for 10 days. The child was born at home and has never been seen in this practice. The mother reports that the child is doing well, has no problems, and is still being breastfed. The child has never been examined by a doctor. The PNP’s initial plan of care for the child is:
A) Treat the congestion, and start immunizations
B) Start immunizations, and refer the parents for genetic counseling
C) Rule out an ear infection, and refer the child to a physician for a hearing evaluation
D) Rule out a heart defect, and refer the child to a physician
a

The PNP has completed vision screens on the children who attend a day care and preschool center. Which of the following children should be referred for further testing?
A) A 3-year-old child (tested with a Sjögren hand chart) with the following results: right eye, 20/40; left eye, 20/20
B) A 4-year-old child (tested with an HOTV chart) with the following results: right eye, 20/40; left eye, 20/40
C) A 4-year-old child (tested with a Snellen E chart) with the following results: right eye, 20/20; left eye, 20/30
D) A 3-year-old child (tested with an Allen chart) with the following results: right eye, 20/30; left eye, 20/30
a

An infant brought to the clinic is diagnosed with failure to thrive (FTT), persistent growth deficiency of the head, and delayed development. Some unusual facial abnormalities are noted. The mother has a history of consuming large amounts of alcohol. The PNP suspects fetal alcohol syndrome (FAS) and refers the infant:
A) To a First Steps Program or Birth to Three Program
B) For consultation with a neurologist
C) For a genetic workup
D) For consultation with an endocrinologist
a

The PNP is assessing a neonate in the nursery. The cremasteric reflex is absent, and the right testicle cannot be palpated in the scrotum. The PNP should:
A) Order a sonogram
B) Consult with the physician
C) Refer the neonate to the urology clinic
D) Not allow the neonate to be discharged from the hospital
b

The PNP is in the nursery to examine a neonate. The results of the neonatal screen are noted in the chart. The T4 level is low, and the TSH level is elevated. The PNP should:
A) Consult with the physician, and repeat the tests
B) Discharge the neonate, and schedule a follow-up visit in the office in 1 week
C) Begin the neonate on thyroxine based on the test results
D) Examine the neonate for signs of hypothyroidism
a

A 7.5-year-old Hispanic child has prepubertal pigmented pubic hair. No other secondary sex characteristics are noted. The mother is concerned that the child will soon begin menstruating. What advice would the PNP give the mother?
A) “Premature sexual development is common in Hispanic girls, and no treatment is required.”
B) “Premature adrenarche can be a normal variant in Hispanic girls, and no treatment is required, but we will monitor the changes at each annual visit.”
C) “Central precocious puberty is a normal variant in 10% to 15% of girls.”
D) “Start talking to your daughter about the menstrual cycle because menarche will occur soon.”
b

The mother of a 5-year-old child questions the PNP about precocious puberty. The PNP relates to the mother that complete or central precocious puberty:
A) Is more common in boys
B) Is usually idiopathic in girls
C) Is often caused by central nervous system (CNS) tumors in girls
D) Affects one area of sexual development only
b

Three months ago, a 7-year-old child with type 1 diabetes was hospitalized for elevated blood glucose levels and thyroxine (T4) and ketones in the urine. The child’s blood glucose level has been elevated for the past three mornings and within normal limits during the rest of the day. What advice would the PNP give the parents?
A) “Increase the evening dose of regular insulin, which works from bedtime to the following morning.”
B) “Increase the evening dose of NPH, which works from bedtime to the following morning.”
C) “Decrease the evening dose of regular insulin, which works from bedtime to the following morning.”
D) “Decrease the evening dose of NPH, which works from bedtime to the following morning.”
b

A 7-year-old child has a blood glucose level of 414 mg/dL. Ketones are present in the urine. The child is admitted to the hospital for treatment of type 1 diabetes. In the hospital a registered dietitian meets with the parents to explain how food influences blood glucose levels. The goal is a well-balanced diet that provides adequate nutrition for appropriate growth and development. The PNP reinforces the dietitian’s advice by telling the parents that the child’s diet should consist of:
A) 10% to 15% fat, 30% to 35% protein, and 45% to 60% carbohydrates
B) 10% to 20% fat, 10% to 20% protein, and 55% to 60% carbohydrates
C) 5% to 10% fat, 25% to 30% carbohydrates, and 45% to 60% protein
D) 30% to 35% fat, 10% to 15% carbohydrates, and 45% to 60% protein
b

A 10-year-old diabetic has blurred vision, fruity breath odor, and a rapid pulse. The blood glucose level is 380 mg/dL, and the urine glucose level is 2%. A test strip was positive for ketones. The child complains of cough and a sore throat. The PNP reminds the child that ketoacidosis is a serious complication of diabetes and that ketone testing should be done:
A) When the blood glucose test result is 180 mg/dL or more
B) When the blood glucose level is 80 mg/dL or less
C) Every day so that it is evident when the blood glucose level is elevated
D) During any illness, even a cold
d

Hypothyroidism has just been diagnosed in a 5-year-old child, and thyroid replacement therapy has been initiated. Anticipatory guidance for the child and family should include:
A) Emphasizing the importance of compliance and periodic monitoring of the child’s response to therapy
B) Initiating appropriate referrals for the child regarding possible mental retardation caused by hypothyroidism
C) Helping the family accept the child’s short stature, which cannot be prevented but can be coped with in a healthy manner
D) Referring the child to a dermatologist for treatment of mixed edematous skin changes
a

An 11-year-old girl is brought to the office for an annual well-child visit. When discussing the onset of puberty with the preadolescent and the mother, which information would the PNP provide?
A) “Pubic hair develops before breast buds.”
B) “Girls have their greatest linear growth after menses.”
C) “The average time from breast buds to menarche is 1.5 to 2.5 years.”
D) “The average age of onset for menarche is 10 to 12 years.”
c

An understanding of the pathophysiology of UTIs is essential when educating families about the prevention of recurrent infections. What should a family be told about decreasing the risk for UTI in their child?
A) Encourage daily baths
B) Prevent and monitor for constipation
C) Encourage drinking grapefruit and orange juice
D) Suggest the child use the bathroom three to four times a day
b

A father asks how his 10-year-old daughter could have developed pyelonephritis. The PNP explains that cystitis or pyelonephritis in childhood is most probably caused by:
A) Catheterization or fecal soilage
B) Hematogenous spread or bacteremia
C) Drugs or foreign bodies
D) Ascension of bacteria into the urinary tract
d

The mother of a 2-year-old child is concerned about the child’s fear of the dark. The PNP responds that:
A) Fears of this type are a normal developmental process
B) Young children with fears tend to have phobias as adults
C) Fears that interfere with social development resolve as the child develops
D) Parents should not allow the child to withdraw from a fearful situation
a

The mother of a 2-year-old child is concerned about the child’s thumb sucking during the day when not playing and at night while sleeping. The PNP suggests that:
A) This is not a problem unless the thumb sucking continues past age 3 years
B) The mother should obtain an orthodontic device for the child’s mouth to discourage the sucking behavior
C) The parents should send the child to time-out when caught thumb sucking during the day
D) The parents should ignore the behavior because it is not harmful at this age
d

A 3-year-old child is brought to the clinic for a well-child visit. The mother requests help with toilet training. The child refuses to use the toilet to have a bowel movement and squats, wearing a diaper, in a secluded corner to pass a stool. The parents offer encouragement through rewards and the “big kid” approach to using the toilet, but nothing seems to help. The PNP should respond that:
A) Refusal to use the toilet after age 2.5 years is the result of caregivers (parents, day care workers, etc.) putting too much emphasis on toilet training
B) The child should be forced to sit on the toilet every 2 hours for 5 to 10 minutes during the day
C) All responsibilities, including clean up (with appropriate assistance), should be turned over to the child after wetting or soiling pants
D) Autonomy struggles are common at this age; most children eventually train themselves
d

The mother of a 2-year-old child requests information regarding toilet training readiness. The PNP tells the mother that the most important factor in determining readiness to toilet train is that the child:
A) Is not bothered by wet or soiled pants
B) Is strong willed and shows an interest in the potty
C) Is able to communicate needs and follow directions
D) Wakes up from naps with soaked diapers
c

The PNP counsels the mother of an infant about possible sibling rivalry a 2-year-old sibling may exhibit. To decrease sibling rivalry the PNP suggests that the parents should:
A) Tell the older child that he or she is no longer a baby and can wait until his or her needs can be met
B) Not allow the older child to touch the infant under any circumstances
C) Not tolerate any regression on the part of the older child
D) Set aside extra time to spend with the older child
d

A mother brings her 2-year-old child to the clinic for a well-child visit. The mother’s chief complaint is the child’s rebellious behavior. The mother says that she has tried time-out, yelling, and spanking but nothing has worked. The child has never slept through the night and still gets a bottle in the middle of the night. In the examination room the PNP observes as the child repeatedly gets into the mother’s purse when she is not paying attention, despite the mother telling the child “no.” Based on the history and observation, the PNP suggests that:
A) The child needs to have limits and boundaries set consistently and the mother could benefit from parenting classes
B) The child is normal; this rebelliousness is only a phase and will improve with time
C) The child exhibits behavior atypical of 2-year-old children and immediate referral to a child psychologist is necessary
D) The child shows signs of attention deficit-hyperactivity disorder, and referral to a specialist may be necessary.
a

A new parent asks why a light is shined in the infant’s eyes. The PNP tells the parent that the infant must be checked for a red reflex in each eye to rule out:
A) Glaucoma
B) Strabismus
C) Cataracts
D) Amblyopia
c

The mother of a 3-year-old child questions the PNP regarding areas in North America where roundworms are a prevalent parasitic infection. Ascaris lumbricoides (roundworm) is most prevalent in what part of the United States?
A) Urban Northeast
B) Suburban Northwest
C) Rural South
D) Desert areas of the Southwest
c

Pinworm infestation is common in children. The PNP discusses the signs and symptoms of pinworm infestation with a mother. Which of the following parasites is responsible for this disorder?
A) Enterobius vermicularis
B) Giardia lamblia
C) Ascaris lumbricoides
D) Trichinella spiralis
a

The PNP is discussing with the father of a child in day care the issues related to treating head lice. The PNP stresses the following:
A) Lice are highly contagious, so all family members should be checked
B) Repeat permethrin cream rinse in 5 days
C) Re-treat in 2 weeks if itching persists
D) Prescribe lindane 1% for all cases of lice
a

The mother of a 1-year-old child requests information about the child’s speech and language development. The PNP explains that vocabulary acquisition may be slow during certain periods and that a vocabulary spurt is most common between ages:
A) 10 and 12 months
B) 12 and 14 months
C) 16 and 24 months
D) 2 and 3 years
c

The mother is concerned that her 3-year-old child has begun stuttering over the past 4 to 6 weeks. The PNP should:
A) Refer the child to a speech therapist and audiologist
B) Instruct the mother to remind the child to speak more slowly and clearly when beginning to stutter
C) Explain that dysfluency is normal at this age but should be reevaluated if it is persistent or worsens
D) Instruct the mother not to make eye contact with the child while the child is speaking to decrease anxiety
c

At a physical examination for Head Start, a mother reports being concerned about her 4-year-old child’s speech. She worries that the child’s speech is unclear and that the child is not saying enough words. The PNP explains to the mother that the child should:
A) Have a vocabulary of approximately 1500 words
B) Have speech that is 100% intelligible to strangers
C) Speak in sentences of three to four words
D) Not have problems with dysfluency (stuttering)
a

A 2-year-old child is screened for lead poisoning. The blood level is 16 mg/dL. The PNP discusses nutritional needs, including a diet high in:
A) Protein and calcium
B) Vitamin C and iron
C) Fat and Zinc
D) Calcium and iron
d

In counseling a parent regarding possible lead poisoning in the child, the PNP states that the most common source of lead is:
A) Drinking water
B) Soil
C) Lead-based paint
D) Batteries
c

The 19-year-old mother of an infant in the PNP’s care has just found out that she is expecting her second child. The mother’s first prenatal visit is scheduled soon. The mother smokes two packs of cigarettes a day. What action should the PNP take?
A) Inform the mother about the risks to herself and to her children from secondhand smoke
B) Sign the mother up for smoking cessation classes
C) Let the obstetrician/gynecologist assess and manage this issue
D) Write the mother a prescription for nicotine replacement therapy
a

A 17-year-old adolescent is anticipating marriage after completing high school. The adolescent does not have sickle cell disease and is not a carrier of the trait, but the intended spouse has sickle cell trait. The adolescent asks if it is possible that any of their future children will have sickle cell disease. The adolescent is told that there is a 50% chance that:
A) Their children will have sickle cell disease
B) Their children will have sickle cell trait
C) Only their sons will have sickle cell disease
D) Only their sons will have sickle cell trait
b

A 12-year-old child is brought to the office for a yearly physical. The child states that he has recently noticed an enlargement of his testes and scrotum. When counseling the boy as to what to expect next in pubertal development the PNP tells him:
A) The penis will grow in length
B) The penis will grow in width
C) Facial hair will appear
D) Changes in voice will occur
a

An 11-year-old child is concerned about breast size. About 6 months ago when her breast development began, she noticed that her breasts were different sizes. The PNP responds:
A) “This is abnormal and you need to see a specialist.”
B) “Menarche will appear soon.”
C) “A mammogram is needed.”
D) “The breasts will become closer to the same size within a few years.”
d

The PNP has prescribed salmeterol (Serevent) by inhalation for an asthmatic. The family should be warned about which of the following side effects that occur with this medication?
A) Tachycardia and headache
B) Weight gain and acne
C) Nausea and vomiting
D) Dysrhythmia and dry mouth
a

The most appropriate advice the PNP can give the mother of a 10-month-old who weighs 21 lb concerning the use of car seats is:
A) “A child weighing more than 20 pounds may face forward in the car seat.”
B) “A booster seat should be used when your child has outgrown a convertible car seat but is still too small to fit properly in a vehicle safety belt.”
C) “Keep using a rear-facing car seat until age 1 year.”
D) “It is safe to keep your child in a car seat in the front seat of the car as long as there is not a passenger-side airbag.”
c

The mother of a 2-year-old child calls the office with concerns about temper tantrums. She asks if screaming, falling on the floor, and kicking are normal behaviors. The PNP explains
A) “The child is expressing a need for independence.”
B) “The child is trying to get attention.”
C) “The child is probably spoiled and needs a spanking.”
D) “The child cannot yet express frustrations verbally.”
d

A 3-year-old is seen in the clinic for a well-child checkup. The mother states she is concerned because the child is a thumb sucker and worries this will cause dental problems. The PNP explains:
A) There is no need for concern until the child reaches age 4 to 5 years
B) An immediate dental referral is needed to prevent further damage
C) Thumb sucking is not a problem until permanent dentition is completed
D) A dental referral is needed within the next 6 months
a

A 3-week-old infant is being breastfed by a new mother. The mother is unsure if the infant is getting enough breast milk. Besides documenting weight gain, what other signs can the mother observe as an indication of adequate breast milk intake? The infant:
A) Sleeps 4 to 5 hours between feedings
B) Cries vigorously before feedings
C) Has 4 stools and 6 wet diapers per day
D) Sucks eagerly on a pacifier and chews on hands after feeding
c

The PNP is returning a phone call to a parent who has questions regarding the use of over-the-counter medications for a 2-year-old child. The mother reports that the child has been running a temperature of 101.2°F with no other symptoms of illness. The mother has been treating the fever by alternating doses of acetaminophen and ibuprofen every 3 to 4 hours. The child is playful and has a normal appetite. Appropriate counseling for this mother should include:
A) Continue giving the medicines every 3 to 4 hours at the appropriate dosage for the child’s weight until the fever subsides
B) Increase the interval of the medications to every 6 hours
C) This amount and frequency of antipyretics may be inappropriate and harmful in a child whose only symptom is a low-grade fever
D) Stop the medications, because a temperature of 101.2°F in a child with no other symptoms does not necessitate the use of antipyretics
c

The PNP is performing a well-child examination on an 8-year-old child with a previous diagnosis of precocious puberty. The patient is Tanner stage III for breast development and pubic hair growth, with height and weight in the 95th percentiles. The patient has missed 6 months of her hormone therapy. The mother states that the child often complains of abdominal cramping, and the mother has noticed an increased vaginal discharge. The remainder of the examination is normal. Addressing the abdominal pain should include:
A) Evaluation for a UTI
B) Guidance regarding the appropriate dietary intake of fruits, fiber, and water to prevent constipation
C) Anticipatory guidance regarding the importance of complying with drug therapy and the likelihood of the onset of menses
D) A referral to a gastroenterologist for evaluation
c

A 7-year-old child is brought to the clinic for a rash that has been present for 24 hours. The child was “a little droopy about 3 days ago” but had no fever or other symptoms. The immunizations are up-to-date. The child now has a “slapped cheek” appearance, with a lacy, erythematous rash on the torso and upper arms. The rash blanches. The PNP discusses with the mother the diagnosis of:
A) A coxsackievirus and explains that the rash will disappear within a week and no treatment is necessary
B) Measles, which is highly contagious, and explains that the child should be taken home and kept away from siblings
C) Human parvovirus B19, or fifth disease, for which there is no treatment
D) Roseola and explains that time is the best treatment and the rash will fade in about a week
c

A 12-year-old girl has questions about the potential for her to get much taller. She aspires to be a professional basketball player. On examination she is in Tanner stage IV. Menarche was 1.5 years ago. In considering answers to her questions, it is important to realize which of the following?
A) She will not reach her peak height velocity until she is in Tanner stage V
B) Menarche occurs an average of 2 years after thelarche
C) Girls reach peak height velocity immediately before menarche
D) Boys reach peak height velocity at the average age of 15 years
c

A mother asks about the term Moro reflex. The PNP explains that the reflex is present in a neonate and disappears or phases out at about age:
A) 3 to 5 months
B) 6 to 8 months
C) 7 to 9 months
D) 10 to 12 months
a

A 10-year-old child was seen in the clinic 5 days before for a routine sports physical examination. The child’s blood cholesterol level was 186 mg/dL. The most appropriate intervention by the PNP would be to:
A) Screen other members of the family for hyperlipidemia
B) Implement a diet plan of less than 20% of fat in total daily calories
C) Counsel the child and family regarding a healthy diet and exercise and perform another blood cholesterol test in 3 to 5 years
D) Repeat the total cholesterol test and obtain lipoprotein analysis
d

Which of the following do not meet the criteria for a selective screening for hyperlipidemia?
A) A 16-year-old adolescent who is adopted, who smokes, and who has no known family history
B) An 8-year-old child with a small restrictive ventricular septal defect
C) A 10-year-old child whose father had a balloon angioplasty at age 49 years
D) A 12-year-old child with BMI of 28
b

When assessing the heart of a healthy 4-year-old child, the PNP would expect to find:
A) A visible apical pulse or point of maximal impulse
B) Sinus tachycardia
C) S2 heart sounds louder than S1 heart sounds at the apex
D) Long, low-pitched heart sounds
a

A 15-year-old girl is being evaluated for possible delayed puberty. There is a documented history of Tanner stage 2 breast development and Tanner stage 1 pubic development at age 9.5 years. What current findings would support the diagnosis of delayed puberty?
A) Family history of abnormal puberty
B) Palpable breast buds with areolar enlargement
C) Dark, coarse, curly pubic hair spreading over the mons
D) Weight at 5th percentile
b

An 8.5-year-old child is brought to the clinic for the first time. On physical examination a penile length of 6.2 cm is noted. The differential diagnosis includes precocious puberty. What additional clinical finding is associated with precocious puberty?
A) A testicular length of 2.1 cm
B) Multiple small café au lait spots (neurofibromatosis)
C) A growth spurt of 5 cm/y
D) A blood glucose level of 97 mg/dL
b

A 5-day-old neonate, diagnosed with trisomy 21, was noted to have an elevated thyroid-stimulating hormone (TSH) level. The neonatal screening was performed at age 3 days. What should the PNP do next?
A) Reassure the mother that an elevated TSH at age 3 days is common and that no further treatment is necessary
B) Repeat the test because it was not performed at the appropriate time
C) Repeat the test when the child is age 3 weeks because this is the appropriate time
D) Repeat the test and inform the mother that children with Down’s syndrome have a high rate of hypothyroid disease
d

The PNP is meeting with the parents of a 7-year-old child with type 1 diabetes to discuss blood glucose levels. A suggested blood glucose level for a 7-year-old child with diabetes is between:
A) 80 and 120 mg/dL
B) 100 and 200 mg/dL
C) 80 and 180 mg/dL
D) 70 and 150 mg/dL
c

A 14-year-old Asian-American male is brought to the office for a well-child visit. The adolescent measures 145 cm (57 inches) in height and weighs 46 kg (102 lb). Physical examination reveals underdeveloped genitalia (i.e., Tanner stage 1, testes down, no hernia). Which diagnostic tests would the PNP order initially?
A) Thyroid function tests, a bone age determination, and a complete blood cell count
B) Tests to determine plasma luteinizing hormone (LH) and follicle-stimulating hormone (FSH) levels and bone age
C) Radiographic examination of the spine, tests to determine the plasma FSH level, and thyroid function studies
D) Tests to determine the sedimentation rate, FSH, and bone age
b

The PNP evaluates an infant born to a mother known to have autoimmune thyroid disease. Congenital hypothyroidism in infants is associated with which of the following?
A) Excessive sleepiness
B) Failure to thrive
C) Tachycardia
D) Diarrhea
a

In caring for a noncompliant 16-year-old adolescent with diabetes, the PNP can best assess the diabetic control by:
A) Measuring the insulin level
B) Obtaining a glucose tolerance test
C) Monitoring serum glucose levels for 3 consecutive days
D) Measuring the hemoglobin A1c level
d

The PNP is caring for a 10-day-old neonate who was born by vaginal delivery at a birthing center. The neonate was sent home at age 24 hours and received the initial thyroid function tests. The tests revealed a mildly low T4 and an elevated TSH level. The infant has been feeding without difficulty, has regained birth weight plus 3 oz, and appears alert during the examination. The PNP’s management of the child should include:
A) Beginning levothyroxine therapy and referring the family to an endocrinologist
B) Repeating the test, including serum T4, TSH, thyroxine-binding globulin, and T3
C) Referring the parents for genetic counseling
D) Beginning fludrocortisone therapy and repeating the initial tests
b

A 13-year-old Tanner stage III/IV female adolescent who competitively runs cross-country track is scheduled for an annual sports physical examination. The adolescent complains of mild abdominal cramps and backache. When asked about voiding difficulties, the adolescent reports frequency symptoms. The dipstick screening reveals yellow urine, +1 blood, and no protein, leukocytes, and nitrites on the reagent strip. This morning the adolescent started having some dark vaginal discharge. The physical examination is otherwise within normal limits. The blood pressure was 110/78. Which of the following would be appropriate management?
A) Obtain a culture and microscopic urinalysis, and start the patient on antibiotics, confirming improvement in 24 to 48 hours; perform repeat urinalysis or culture in 14 days
B) Reassure the adolescent and the family that transient hematuria can be related to vigorous exercise, urinary tract infection (UTI), or menarche
C) Obtain a urinalysis and complete blood cell count, and order tests to determine electrolyte levels, blood urea nitrogen, creatinine level, and creatinine clearance
D) Obtain urinalysis and culture, and explain that these problems may be transient and related to exercise
d

In the management of a child with a neurogenic bladder resulting from myelomeningocele, the most critical long-term goal is:
A) Controlling incontinence
B) Preventing kidney damage
C) Preparing the child for bladder augmentation
D) Preventing bladder spasms
b

Separation anxiety is a common developmental stage in which the child exhibits fears and unusual behavior when a parent is absent. This behavior is most often expected at:
A) Age 6 to 9 months
B) Age 9 to 12 months
C) Age 12 to 15 months
D) Age 15 to 18 months
a

A 5-year-old child is at the clinic for a routine well-child visit before beginning kindergarten. The mother reports the child’s list of chores as follows: clean own room, make own bed, wash and dry own clothing, take out the trash, and help wash the dishes. If the chores are not completed, the child is grounded from other activities, such as playing with friends or watching television. The mother states that the child frequently must be grounded. How should the PNP respond?
A) Tell the mother to make a chart so the child can remember to do the chores and record their completion and to continue grounding the child from activities when necessary
B) Suggest use of natural consequences (e.g., having no clean clothes to wear if the laundry does not get done) as an additional way to discipline
C) Perform a complete physical assessment of the child, as well as a parenting assessment
D) Tell the mother she is expecting too much from the child and should reduce the number of chores
c

One of a young child’s greatest fears is:
A) Separation from parents or caregivers
B) Not getting enough food
C) Not being accepted by peers
D) Having limits set by caregivers
a

A 4-year-old child is examined in the clinic. The parents report that the child has limited language skills and does not speak in three-word sentences. The PNP is interested in further assessing the child and chooses the following:
A) Denver Articulating Screening Examination (DASE)
B) Denver II
C) Clinical Adaptive Test/Clinical Linguistics Auditory Milestone Scale (CAT/CLAMS)
D) Vineland Social Maturity
a

A 4-year-old child is brought to the office for a preschool physical examination. The PNP asks the child to sit on the floor in a cross-legged position and then get up without using the hands. The child is unable to do so and rolls onto all four extremities before standing. The PNP is concerned that the child may have a:
A) Neuromuscular disorder
B) Developmental problem
C) Cognitive deficit
D) Neurologic deficit
a

A 6-month-old infant is brought to the clinic for a well-child visit. On physical examination, increased tone and lower leg scissoring are noted. The PNP recognizes this as a probable sign of:
A) Muscular dystrophy
B) Down’s syndrome
C) Cerebral palsy
D) Fragile X syndrome
c

A 16-year-old female adolescent with Down’s syndrome, who is healthy and has had no major medical problems, comes to the clinic for a well-child visit. The adolescent has been “mainstreamed” into a regular classroom and is doing well. The physical examination reveals Tanner stage III breast development and pubic hair. Today the PNP should:
A) Provide nutritional counseling, order cervical spine x-ray films, and refer the adolescent to a family support group
B) Perform hearing and vision screens, and refer the adolescent to a long-term planning counselor
C) Order cervical spine x-ray films, a vision screen, and echocardiography
D) Order thyroid and hearing screens, and provide sexuality education
d

The first step in the process of developing an IEP for a child with disabilities is to:
A) Provide government funds to the child and family to supplement income
B) Perform a comprehensive assessment of the child by a multidisciplinary team
C) Perform a complete history and physical examination to identify organic causes
D) Perform a battery of psychologic tests that are valid and reliable
b

A father accompanies his 4-year-old child to the clinic. Significant past history reveals that the child had lipomyelomeningocele. The father states that there are no specific complaints, but during the history he reports that the child has complained of back pain over the last 3 months. Further evaluation reveals that the frequency of bowel movements has decreased from once daily to three times a week and that the child is “wetter” between catheterizations. The most likely cause is:
A) Acute muscle strain
B) Tethered cord
C) Urinary tract infection
D) A diet low in fiber, fruits, and vegetables
b

A 2-month-old infant with spina bifida and shunted hydrocephalus has a history of increased gagging with feedings, spitting up, intermittent stridor, and failure to thrive. The most likely explanation is:
A) Tethered cord
B) Shunt malfunction
C) Arnold-Chiari deformity type II
D) Esophageal structure
c

A 2-week-old neonate is brought to the office for the first time. The mother states that the doctor in the hospital suspected Down’s syndrome, and she begins to cry. On physical examination the infant has characteristics that may indicate Down’s syndrome. The phenotypic features the PNP recognizes as commonly associated with Down’s syndrome include:
A) Large ears and small mouth, short fingers, and jaundice
B) Small ears, short neck, and hypotonia
C) Flat nasal bridge and normocephaly
D) Protuberant tongue, microcephaly, and jaundice
b

The mother brings a 4-month-old infant to the office for a well-child visit. The infant received a hepatitis B (Hep B) immunization in the hospital at birth and another Hep B in the office at age 1 month but has received no other immunizations. The infant lives with the mother and grandfather. The grandfather is asthmatic and is taking steroids. What immunizations should the infant be given today?
A) Hep B, DTaP, and Hib
B) DTaP, Hib, and OPV
C) DTaP, Hib, Prevnar, and IPV
D) Hep B, DTaP, Hib, and IPV
c

The school counselor refers a 7-year-old child, who is not known to have received any immunizations, to the clinic. The child has no history of varicella. What immunizations are required at this visit?
A) Hep B, MMR, Var, Td, and IPV
B) Hep B, Var, Td, and IPV
C) Hep B, Td, and IPV
D) Hep B, MMR, Var, DTaP, and IPV
a

The PNP is the primary care provider for a preterm infant (born at 31 weeks’ gestation). The infant had no complications other than a minor ventricular hemorrhage at birth. The infant is 4 months’ chronologic age and progressing well. What type of respiratory syncytial virus (RSV) prophylaxis should the PNP consider at today’s visit?
A) RSV prophylaxis in six monthly doses, if the local RSV season is approaching
B) Nothing, because the infant did not have bronchopulmonary dysplasia at birth
C) Nothing, if the RSV season has already begun
D) RSV prophylaxis in three monthly doses if the local RSV season is approaching
a

A 4-year-old is brought to the office for a preschool physical examination. The child was hospitalized for Kawasaki’s syndrome and treated with intravenous immune globulin (IVIG) 9 months ago. According to the immunization records, the child requires DTaP, polio, and MMR immunizations at this time. What immunizations should the PNP administer today?
A) DTaP, IPV, and MMR
B) Only MMR
C) None (immunizations should be given at the school physical examination next year)
D) DTaP and IPV (the parent should be instructed to schedule an appointment in 2 months for the MMR)
a

A 12-month-old child is brought to the clinic. On physical examination the PNP notes that the child’s right eye drifts to the nasal side of the eye. The PNP explains to the mother that a complication of this condition may be:
A) Myopia
B) Astigmatism
C) Amblyopia
D) Strabismus
c

What is the appropriate INITIAL intervention if the parent refuses immunizations?
a. notification of child protective services of vaccine refusal
b. discussion of the community benefit immunization
c. validation of the parental concerns regarding individual immunizations
d. provision of the “refusal to immunize” form for parental signature
c

Which adolescent is considered the MOST likely to abuse illicit drugs?
a. a 12yo caucasian male who admits to drinking beer once a month
b. a 15yo african am male who admits to trying a variety of alcoholic beverages
c. a 16yo caucasian male, who dropped out of school, was suspended twice for smoking and has smoked marijuana twice
d. a 16 yo african am, male who is an honor student but admits to smoking marijuana occasionally
c

The most appropriate analgesia for a 4yo with post operative tonsillectomy pain would include
a. ibuprofen
b. acetaminophen and oxycodone (Roxicet)
c. Ketorolac
d. aspirin and oxycodone (Percodan).
b

A previously healthy 10mo has vomiting, intermittent periods of intense crying, and the passage or red stool. Which would be considered FIRST in child?
a. refer to emergency dept
b. obtain an abdominal x-ray
c. obtain a complete blood count
d. schedule surgical appointment for tomorrow.
a.

Herpes Zoster is characterized by clustered vesicles and
a. is transmitted via respiratory secretions
b. is reactivated on exposure to a siblings chicken pox
c. results frequently in post herpatic neuralgia
d. often develops along the trigeminal nerver
d

Which of the following should be included when educating a family regarding the care of children with atopic dermatitis?
a. apply a barrier ointment to the child’s face before feeding
b. bathe children daily and encourage low humidity in the house
c. expect symptom improvement within 1 week after starting probiotics
d. investigate and eliminate possible food triggers
a

A parent reports that her 18mo cries during the entire aerosol asthma treatment with albuterol and budesonide. What is an alternative managment?
a. prescribing oral albuterol
b. using MDI and Spacer
c. using nebulizer when child is sleeping.
d. adding montelukast
a

In addition to hypoglycemia, the MOST common complications found in infants born to mothers with diabetes or gestational diabetes include
a. LGA, neural tube defects, anemia, and renal vein thrombosis
b. SGA, cardiomegaly, polycythemia and hyperthermia
c. LGA, cardiomegaly, renal vein thrombosis and polycythemia
d. SGA, hepatosplenomegaly, hypotonia, and anemia
c

The parents of an adolescent with pectus excavatum and asthma are interested in surgery for their child, in the hopes of improving his asthma. Which would be MOST accurate response?
a. the only purpose of surgery is cosmetic
b. studies of post-operative cardiac and pulmonary functioning following pectus repair show inconsistent results.
c. reduction of lung volume and degree of symptoms correlates well with extent of the deformity
d. improvement of asthma after surgery is dependent on the family history
b

Compared to single component vaccines, combination vaccines
a. are associated with significant incidence of adverse effects
b. expose children to many more antigens than previous vaccines
c. are more likely to overload an infant’s immune system
d. can be used if the child previously received component of the vaccine.
d

One of the major psychosocial tasks of infancy is:
a. development of secure attachment
b. separation-individuation
c. symbiosis
d. regulation
a

When examining 10 yo Bob’s teeth, you note that the upper incisors slightly overlap the lower incisors. The second and lower first molars are absent. Your assessment is:
a. malocclusion
b. delayed mandibular dentition
c. normal dentition
d. hyperdontia
b

Which one of the following best describes behavior associated with Piaget’s concrete operations?
a. Learning primarily by trial and error
b. Interpreting events in relationship to self
c. categorizing information
d. drawing logical conclusions
c

A 5.5yo asian child has fallen off of her growth curve. The best intervention would be to:
a. Suggest infant breakfast drinks
b. Incorporate traditional foods in diet
c. educate family on need to increase calories & nutrients
d. Refer to endocrine clinic for evaluation
b

What is considered minimal weight gain in a normal newborn after discharge from the hospital?
a. 10gms/day
b. 20gms/day
c. 30gms/day
d. 40 gms.day
c

The most likely weight of a 1yo child whose weight at birth was 6.5lbs would be what?
a. 19-20lbs
b. 13-14lbs
c. 25-26lbs
d. impossible to estimate
a

You would expect a school age child to:
a. grow 1.5in per year
b. grow 0.5in per year
c. Gain about 6lb per year
d. Gain about 3lb per year
c

Johnny, 5 yo, is in your clinic for his initial well check. You note he has not received any immunizations. Which of the following is not necessary at his age?
a. MMR
b. DTaP
c. Hib
d. Varicella
c

As the NP, you are assessing an 8mo infant in an immunization clinic. You know that by 8mo the child should have had which of the following immunizations?
a. Hep B (1st & 2nd), all primary DTaP series, two doses of polio
b. All DTaP, polio series, MMR
c. DTaP (1st & 2nd), MMR (1st), all Hep B series
d. Varicella, DTaP (1st), Hep B (1st)
a

Mom informs you that she and her 6mo infant are traveling to a place where measles is endemic. Your best response to her is:
a. She should not take her baby with her
b. Reassure her that her baby has passive immunity
c. Give baby a MMR or monovalent measles injection
d. Give baby gamma globulin prior to the trip
c

Tommy is in for his 4mo well check. He was born preterm with a birth weight of 2.0 kg. The appropriate immunizations to give him today would be:
a. DTaP, Hib, IPV, HepB-3
b. DTaP, Hib, IPV, PCV 7
c. DTaP, Hib, Hep B-3, PCV 7
d. Hold off on immunizations until he reaches his term due date
b

In considering client situations, which one requires the use of an inactivated (not live) vaccine?
a. history of nonspecific allergies
b. immunocompromised client
c. concurrent antimicrobial therapy
d. mild acute illness
b

Which of the following ongoing assessments have no specific indication for individuals which Turner’s syndrome?
a. Cardiac monitoring
b. vision screening
c. tanner staging
d. thyroid screening
b

While examining a 2.5 week old infant, you notice irritability when you lift the infant and an asymmetrical Moro reflex. A spasm of the sternocleidomastoid on the left side is also present. These findings suggest:
a. Torticollis
b. Genu varum
c. Fracture clavicle
d. Pierre-Robin syndrome
c

Newborn screening for hypothyroidsm is done by measuring:
a. thyroid stimulating hormone (TSH)
b. Thyroxine (T4) and TSH
c. Triiodothyronine (T3)
d. T4 binding globulin (TBG)
b

Infants who have been identified as IUGR are prone to developing hypoglycemia due to:
a. decreased metabolic rate
b. low levels of glycogen stored
c. become acidotic
d. a prone to develop sepsis
b

You are performing a newborn physical exam and you notice white specks around the circumference of the iris. You will also have to assess for:
a. hypertonicity
b. cherry red spot on the macula
c. heart murmur
d. cleft palate
c

What is the correct sequence of motor development in average infants?
a. Reach & miss, sit alone, creep, climb stairs, stand alone
b. creep, sit alone, climb stairs, stand alone, reach & miss
c. creep, sit alone, climb stairs, reach and miss, stand alone
d. sit alone, creep, climb stairs, reach and miss, stand alone
a

3 mo rachel cannot remain seated upright without support. At what age will she acquire the seated position with support?
a. 7mo
b. 9mo
c. 12mo
d. 14mo
a

During a well-child visit you teach parents that an infant may first transfer an object from hand to hand at:
a. 2mo
b. 4mo
c. 7mo
d. 9mo
c

A child just starting to reach for objects placed in front of him is probably around:
a. 1mo
b. 4mo
c. 7mo
d. 9mo
b

Children do not generally exhibit a fear of strangers until after about?
a. 2mo
b. 4mo
c. 6mo
d. 2yo
c

According to Erik Erickson, the “cornerstone of vital personality” is the development of:
a. Symbiosis
b. Autonomy
c. Independence
d. basic trust
d

A mother of 10mo old infant asks about the progression of solid foods into her baby’s diet. From the choices below, you advise her to introduce which food last to her baby’s diet?
a. egg yolk
b. egg white
c. fruits
d. vegetables
b

A 9mo old infant boy is seen with a 2 day hx of diarrhea. He had 3-4 wet diapers in the past 24hrs and his anterior fontanel is slightly depressed. Capillary refill is WNL. What % of dehydration does this infant have?
a. 3%
b. 5%
c. 8%
d. 10%
c

Molly is a 12yo girl who has diffuse stomach pain and acute onset of diarrhea described as frequent urge to defecate. She is passing large amounts of flatus, small amounts of stool, and has rectal tenderness with digital exam. Which lab test will confirm your diagnosis?
a. serum albumin & ESR
b. abdominal ultrasound
c. stool for ova & parasites
d. Upper GI
b

Vomitus that is bilious suggest:
a. obstruction proximal to the pylorus
b. obstruction below the ampulla of Vater
c. Pyloric stenosis
d. peptic ulcer disease
b

An 18yo female college freshman, Bailey, presents to college health with an 8hr hx of abdominal pain that began in the periumbilical area and then localized to the RLQ. She has been on OCPs for teh past 6mo. Bailey denies a medical hx of UTIs, pregnancy, STDs, or chronic illness. Upon her physical exam, she is guarding on the RLQ with generalized tenderness and no masses. Her temp is 99.6 and her vitals are normal.
Based on this H&P, which of the following is the most likely diagnosis for Bailey?
a. Mittelschmerz
b. Ectopic pregnancy
c. Appendicitis
d. Gastroenteritis
c

An 18yo female college freshman, Bailey, presents to college health with an 8hr hx of abdominal pain that began in the periumbilical area and then localized to the RLQ. She has been on OCPs for teh past 6mo. Bailey denies a medical hx of UTIs, pregnancy, STDs, or chronic illness. Upon her physical exam, she is guarding on the RLQ with generalized tenderness and no masses. Her temp is 99.6 and her vitals are normal.
Which of the following is the least helpful in the eval of abdominal pain in this client?
a. urinalysis
b. CBC
c. Temperature
d. Pregnancy test
c

An 18yo female college freshman, Bailey, presents to college health with an 8hr hx of abdominal pain that began in the periumbilical area and then localized to the RLQ. She has been on OCPs for teh past 6mo. Bailey denies a medical hx of UTIs, pregnancy, STDs, or chronic illness. Upon her physical exam, she is guarding on the RLQ with generalized tenderness and no masses. Her temp is 99.6 and her vitals are normal.
Which of the following is the most suggestive of Bailey’s diagnosis?
a. Leukocytosis
b. Hematemesis
c. + psoas sign
d. Vomiting before the onset of pain
c

You receive a call from a mother of a 2yo girl with a temp of 101F and diarrhea for 5 days with 6-8 liquid stools/day. How would you advise this mother?
a. Go to ER now
b. Come to the office tomorrow morning
c. NPO
d. Loperamide 10ml after each loose stool
b

An 18yo male comes to see you with a painless mass in the right side of his scrotum. The most likely diagnosis is:
a. epididymitis
b. testicular torsion
c. incarcerated hernia
d. hydrocele
d

Kali is HIV + and has just delivered a 3.8kg male infant. Which of the following should be included in her postpartum counseling?
a. suggest use of commercially prepared formula
b. encourage HIV testing of the infant at birth & at 2mo
c. avoid day care until infant has had 2 negative HIV tests
d. instruct that oral polio vaccine is recommened
a

Higher-functioning adolescents with autism are at increased risk for:
a. significant phobias
b. panic attacks
c. substance abuse
d. obsessive-compulsive disorder
d

Which of the following is an example of a fine motor skill achieved by preschool aged children?
a. being able to alternate feet
b. walking on a balance beam
c. holding a glass with one hand
d. balancing on one foot
c

according to Erikson, the major psychological task between 1-2yrs of age is the development of:
a. trust
b. identity
c. industry
d. autonomy
d

A mother of a 16mo old toddler announces that she is giving her child skim milk. You tell her she should switch to whole milk because:
a. skim milk is not easily digested
b. Skim milk does not have enough calcium
c. Skim milk has too little protein
d. Skim milk does not contain enough essential fatty acids
d

What is the most sophisticated and advanced type of play?
a. associate
b. parallel
c. cooperative
d. onlooker
c

In boys, Tanner III is distinguished from Tanner stage II by:
a. Penile growth in width
b. Penile growth in length
c. Fine, downy hair growth at the base of penis
d. Adult-like pubic hair that does not extend to the thigh
b

During an annual physical exam, you notice that Lisa has developed breast buds. You tell her she can expect which of the following in 2 years:
a. Pubic hair growth
b. Peak height velocity
c. Onset of menses
d. axillary hair
c

The time of ovulation is usually:
a. about 14 days before onset of the next menstrual period
b. about 14 days after the onset of the previous menstrual period
c. during the menstrual period
d. about 7 days after the onset of the menstrual period
a

Jess has recently noticed that a change in his voice, the shape of his body, and pubic and facial hair growth. What is he noticing is the development of:
a. primary sexual characteristics
b. secondary sexual characteristics
c. deciduous sexual characteristics
d. laten sexual characteristics
b

According to Erikson, the major psychosocial task of adolescence is the achievement of:
a. autonomy
b. industry
c. identity
d. generativity
c

Which component of identity is likely to develop first?
a. physical
b. vocational
c. moral
d. ideological
a

Which of the following is responsible for sexual maturation of males?
a. estrogen
b. testosterone
c. progesterone
d. adrogen
b

Which of the following is the correct sequence for adolescent female development?
a. growth acceleration, breast development, pubic hair, menarche, axillary hair
b. growth acceleration, pubic hair, breast development, axillary hair, menarche
c. breast development, growth acceleration, axillary hair, pubic hair, menarche
d. axillary hair, breast development, pubic hair development, menarche, growth acceleration
a

Adolescent girls are especially prone to developing which of these disorders:
a. hirsutism
b. hypothyroidism
c. folate deficiency anemia
d. iron deficiency anema
d

When discussing issues concerning confidentiality, the NP should tell the adolescent (>18yo) or when under the state age of majority that:
a. the parents must have access to all information in the health hx
b. parents cannot access information without adolescent’s permission
c. parents must be present during all encounters
d. it is at the discretion of the provider to decide whether to release information
b

During auscultation of a 3yo’s chest, you note an irregular heartbeat of 90bpm that slows when resp rate slows & accelerates when the child breathes faster. The rest of the exam is normal. What is appropriate response to this finding from the choices below?
a. record the finding in the child’s chart
b. order a chest x-ray
c. order an echocardiogram
d. refer the child to a cardiologist
a

Which of the following drugs is important in symptom management and the prevention of complications in Kawasaki’s disease?
a. aspirin
b. corticosteroids
c. acetaminophen
d. penicillin
a

While performing a well child check exam on a 3yo you discover a heart murmur. In your eval, you remember the following about innocent murmurs:
a. they are best heard in diastole
b. you must radiate to the axilla
c. the intensity is no greater than I or II/VI
d. There is no variation with changes in position
c

Following tympanostomy tube insertion, it is important that the tubes remain patent. Which of the following methods may be used to determine patency?
a. visual inspection
b. impedance tympanometry
c. valsalva maneuver
d. instillation of an ototopical suspension
a

9yo Tom is in the clinic for a “cold” that lasted for 10 days. He has no complaints of breathing problems. A physical exam reveals temp of 100F, edematous cervical lymph nodes, mucopurulent drainage from the middle meatus, erythematous pharynx & malodorous breath. Tom’s management should include:
a. culture of nasal drainage
b. radiograph of the sinuses
c. measurement of erythrocyte sedimentation rate
d. use of an antibiotic
d

After sinusitis dx, two days later, Tom is back in the clinic because now he has a “swollen eye”. Redness, inflammation, and impaired extraocular movement of his right eye are observed. Which action is most appropriate form the choices below:
a. reassure mom that this is common & benign
b. treat for bacterial conjunctivitis
c. order anti-inflammatory eye drops
d. refer tom immediately
d

7.5yo Jimmy presents to your clinic with inspiratory stridor, drooling, and a temp of 105F. He insists on sitting up during the exam. What is your diagnosis?
a. foreign body aspiration
b. reactive airway disease
c. croup
d. epiglottitis
d

In a child with chronic sinusitis, the most accurate method of identifying abnormalities is:
a. dark room transillumination of the sinuses
b. percussion of the paranasal sinuses
c. AP, lateral, and occipitomental sinus radiographs
d. CT scan of the sinuses
d

An 8yo afebrile child presents with a ST. Upon examination, you note that his tonsils are 4+ without exudate. Differential diagnosis includes:
a. normal
b. strep throat
c. hodgkin disease
d. phayngitis
c

A 2.5yo child presents with a dried pea stuck in his nose passed the first turbinate. An appropriate intervention would be:
a. Refer to ENT
b. NaCl flush
c. Attempt removal with forceps
d. Close ipsilateral nares and encourage sneezing
a

Which of the following is NOT a goal for appropriate asthma management?
a. limited activity & exercise
b. prevent recurrent exacerbations
c. prevent chronic troublesome symptoms
d. Maintain near normal pulmonary functions
a

9yo Jon has recently been diagnosed with mild intermittent asthma. Which of the following is not a routine part of his management?
a. spirometry eval
b. metered dose inhaler technique demonstration
c. environmental triggers & control methods review
d. allergy skin testing
d

Tammy has mild persistent asthma. Appropriate daily medications for this 4yo preschooler should include:
a. an inhaled low dose corticosteroid
b. short acting beta2 agonists
c. an oral systemic corticosteroid
d. a cough suppressant
a

Sally, a 9yo, with moderate persistent asthma, is still having mild symptoms. She uses a short acting inhaled beta 2 agonist 3x/day and cromolyn sodium. You alter this treatment to include which of the following?
a. broad spectrum antibiotic
b. addition of systemic corticosteroid x5 days
c. replace cromolyn with inhaled corticosteroid, and decrease the beta agonist to prn when symptoms subside
d. addition of an inhaled anticholinergic
c

During a visit to your clinic, Mitch, 2yo, presents large muscular calves & demonstrates difficulty rising from sitting position. Which lab test should you order below?
a. serum calcium
b. serum magnesium
c. serum phosphorus
d. serum creatine kinase
d

Which of the following tests is not an appropriate test for development dysplasia of the hip in an 18mo infant?
a. allis sign
b. skinfold symmetry
c. galeazzi sign
d. ortolani maneuver
d

Tina, 12yo, presents with a lateral curvature of her spine. Which of the following tests would diagnose scoliosis?
a. positive gower sign
b. negative gower sign
c. positive adam’s sign
d. negative adam’s sign
c

Johnny, a 16yo high school student is in for a sports physical. He is in good health, but is concerned about a “bad” headache he had several weeks ago. He is afraid he might have a brain tumor. You tell him the most common type of headache with onset in adolescents is:
a. sinus headache
b. vascular headache
c. tension headache
d. migraine headache
c

A CBC is ordered for a 10yo female. Results indicate decr MCHC & decr MCV. Differential diagnosis should include:
a. sickle cell anemia
b. vitamin B12 deficiency anemia
c. Pernicious anemia
d. iron deficiency anemia
d

lab results reveal a hypochromic, microcytic anemia in a 3yo child. Your differential diagnosis must include:
a. lead poisoning
b. pernicious anemia
c. hemophilia
d. folic acid deficiency
a

11yo Mark is diagnosed with constitutional growth delay. Appropriate management includes:
a. starting him on low dose testosterone therapy now
b. counseling regarding delayed onset of puberty
c. thyroxine replacement
d. nutritional counseling
b

Which of the following observations demonstrates the function of the oculomotor nerve?
a. the child frowns during the examination
b. the child offers appropriate verbal responses to the nurse practitioner during the examination
c. the child visually tracks the parent’s movements around the room during the examination
d. the child sticks out the tongue during the examination
c

A 6yo child is being evaluated after sustaining seizures. Which of these tests would be most relevant in this screening?
a. determination of the levels of anticonvulsant drugs
b. electroencephalography
c. Denver II screening test
d. Electromyography
b

A 13yo adolescent comes to the school based clinic with a headache that began this morning before school. The headache is described as unilateral and increasing in intensity. What type of HA is it?
a. cluster
b. tension
c. migraine
d. sinus
c

Which of the following types of headaches requires immediate referral to a physician?
a. acute recurrent
b. chronic nonprogressive
c. acute
d. chronic progressive
d

A newborn is brought to the office for a 2-week well-baby exam. The PNP notes that the infant’s head circumference is 2 SDs below the norm. A neurologist evaluated the child in the newborn nursery but detected the abnormal circumference without significant findings. The baby is otherwise growing well. What is the most appropriate recommendation?
a. F/U in 2 weeks, including measurement of the head circumference
b. refer the patient to a neurologist.
c. order skull x-ray films & MRI
d. do nothing.
a

The PNP is examining a 6mo old infant with what is believed to be craniosynostosis. The suture lines reveal a prominent bony ridge over the occipital and coronal sutures. The PNP should:
a. refer the infant to a neurologist
b. remeasure the head circumference in 2mo
c. discuss with the parents the need for PT
d. order chromosomal studies
a

Which of the following is the most appropriate first choice management strategy for a 30mo old child with learning difficulties and ADD/ADHD?
a. referral for appropriate education
b. referral to an ophthalmologist
c. behavior modification
d. prescription for Ritalin
c

A 20mo old child is reported to have been “shaking” for a short period. The parent reports that the child had a fever of 102F before the episode but seems to have returned to normal. Which intervention should be undertaken first?
a. refer child to the ED
b. refer child to neuro
c. conduct an exam of child
d. have a parent bring the child back for F/U in 1 mo
c

Which of the following is the most important factor in the diagnosis of syncope?
a. history
b. physical exam
c. screening tests
d. routine care
a

Which of the following is the most common cause of tic disorder?
a. hereditary
b. birth injury
c. head trauma
d. metabolic disorders
a

A 6-year-old boy is brought to your office for a routine
well-child examination. Significant findings in the child’s
history, reported by his mother, include unspecified “birth trauma” and a 9-year-old brother who has “difficulty in
school.” The mother also reports that her son cannot “sit still” or play with toys for long periods and notes that he recently had to change schools when the family moved. Physical examination reveals a well-nourished, appropriately dressed 6-year-old boy and no remarkable physical findings. The boy is unable to follow simple commands; he grabs your stethoscope from your neck and then runs around the examination room with it.
On the basis of this child’s history and physical, which of the following is the most appropriate initial action?
a. refer for CT or MRI of head
b. have mother bring the child back for F/U in 1mo
c. refer to psych
d. do nothing
a

A 6-year-old boy is brought to your office for a routine
well-child examination. Significant findings in the child’s
history, reported by his mother, include unspecified “birth trauma” and a 9-year-old brother who has “difficulty in
school.” The mother also reports that her son cannot “sit still” or play with toys for long periods and notes that he recently had to change schools when the family moved. Physical examination reveals a well-nourished, appropriately dressed 6-year-old boy and no remarkable physical findings. The boy is unable to follow simple commands; he grabs your stethoscope from your neck and then runs around the examination room with it. CT & MRI of child’s head reveal nothing out of the ordinary. What is the next step in determining the child’s behavior problems?
a. refer to specialist
b. refer to special education
c. determine whether child meets criteria for ADD/ADHD
d. recommend behavior modification
c

On the basis of your exam you determine a child has ADHD: distractibility, impulsivity, lack of attention, accompanied by hyperactivity. Which of the following conditions is a differential diagnosis?
a. seizures
b. social disturbance
c. dyslexia
d. cerebral palsy
b

Education for parents whose child has sickle cell disease should include that the majority of pain crises are triggered by which of the following?
a. no identifying cause
b. temperature changes
c. cigarette smoke
d. stressful situations
a

A 3 year old, who attends daycare, has diarrhea that began with three days of low grade fever and foul smelling watery stools. One month later, he continues to have loose stools and now has abdominal distention. The most probable causative organism is
a. rotavirus
b. Shigella toxicum
c. Giardia lamblia
d. Staphylococcal enterococcus
c

A 5 year old complains of a painful left eye after being accidentally scratched by a sibling two hours ago. Fluorescein exam shows a small central corneal abrasion. The most appropriate management during the first 24 hours is
a. frequent application of topical antibiotic
b. observation of the injured eye
c. frequent application of topical non-steroidal anti-inflammatory drops
d.occlusive patching of the injured eye
a

Which POST competition guidance is most helpful for best nutrition practices for the athlete?
a. additional protein consumption is needed for tissue repair
b. protein supplements will increase muscle size if taken after competition
c. fat loading is most important within the first 1-2 hours
d. replacement of carbohydrates should optimally occur within 2 hours
d

Which of the following best demonstrates the oral behavior of a 12 month old in relation to readiness for feeding?
a. begins chewing meat with rotary mouth movements
b. controls bite of soft cookie
c. controls liquids taken from a cup
d. sucks in anticipation of the spoon
b

Which of the following is associated with alcohol use in adolescence?
a. family history of alcoholism
b. authoritarian parenting style
c. passive temperament
d. high socioeconomic status
a

The intervention currently advised for a child with 4 episodes of acute otitis media in the past 5 months is
a. prophylactic antibiotics
b. myringotomy and tympanostomy
c. adenoidectomy and tonsillectomy
d. observation
b

Significant progression of a 20 degree scoliotic curve is most likely to occur in a female who is
a. 10 years, Tanner 3, Risser sign of 3
b. 11 years, Tanner 2, Risser sign of 2
c. 13 years, Tanner 4, Risser sign of 4
d. 15 years, Tanner 4, Risser sign of 5
b

The most common side effects of Retin-A cream which include erythema, dryness, and burning can be minimized by applying the medication
a. sparingly
b. while face is wet
c. in the morning
d. in the evening
a

Nutrition assessment of the child on a strict vegan diet should include regular growth monitoring, diet analysis, and laboratory assessment of
a. vitamin b12, zinc, and iron status
b. vitamin b12, calcium, and electrolytes
c. electrolytes, iron, and vitamin b6
d. vitamin b6, zinc, and calcium
a

A 13 year old has repeatedly exhibited behaviors of purposeful cruelty to younger children, property destruction, and school truancy. These behaviors fit BEST the diagnostic critera for
a. major depressive disorder
b. oppositional defiant disorder
c. conduct disorder
d. dysthymic disorder
c

Credentialing of the pediatric nurse practitioner requires which of the following?
a. recognition by third party payers
b. verification of license
c. authorization to perform specific services
d. certification of specialty
b

A 4 year old with close contact 10 days ago to a school classmate, now hospitalized with meningococcemia, currently has no symptoms and a normal physical exam. No other cases have been reported in the community. Appropriate management would include
a. oral rifampin prophylaxis
b. nasopharyngeal culture
c. meningococcal vaccine
d. reassurance
d

Which of the following sign or symptom is seen in students most often associated with anorexia nervosa (AN)?
a. salivary gland enlargement
b. hypothermia
c. hypertension
d. elevated erythrocyte sedimentation rate (ESR)
b

According to the National Asthma Education and Prevention Program (NAEPP) Expert Panel 3 Update, potential long-term adverse effects of chronic inhaled corticosteroids use in children includes
a. decreased bone mineral density
b. hypothalamic-pituitary-axis (HPA) suppression
c. vertical growth delay in the first year of treatment
d. ocular toxicity
c

A toddler is unable to use the right arm normally after the parent pulled the arm to prevent the child from falling. Which finding would confirm the diagnosis of subluxation of the radial head?
a. severe swelling and bruising of the elbow
b. elbow flexed with pronated forearm
c. point tenderness at ulnar aspect of elbow
d. obvious deformity of the forearm
b

Which immunization statement applies to a 3 year old who completed chemotherapy 2 months ago for acute lymphocytic leukemia (ALL)?
a. the child should not receive immunizations containing live viruses
b. serum immunity should be evaluated prior to administration of vaccine
c. a repeat of all recommended immunizations is necessary
d. a repeat of live virus vaccines is warranted
a

The ECG in a school-age child presenting with syncope shows a prolonged QT interval. The initial clinical evaluation should always include
a. urine drug screen
b. chest X-ray
c. electrolyte panel
d. liver enzymes
c

A 1 week diagnosis has a papular rash, with some clustered lesions, which was first noticed by the parents three days ago. Few areas are vesicular. The infant is otherwise healthy. What is the likely diagnosis?
a. neonatal pustular melanosis
b. millia
c. erythema toxicum
d. neonatal acne
c

According to the National Institutes of Health (NIH) guidelines for management of moderate-persistent asthma in a school-aged child, which daily plan is the BEST INITIAL recommendation?
a. high-dose inhaled corticosteroids
b. low-dose inhaled corticosteroids plus long-acting beta-2-agonist
c. leukotriene inhibitor
d. methylxanthine and a mast cell inhibitor
b

Which is the MOST accurate to convey when counseling a family about the effects of congenital cytomegalovirus (CMV) infection on hearing?
a. half of symptomatic infants will have conductive hearing loss
b. hearing loss may be progressive after the newborn period
c. asymptomatic infants are free of hearing loss
d. infection acquired from breast milk is associated with hearing loss
b

Which of the following is the best INITIAL counseling for a family of a 5 year old with nocturnal enuresis?
a. wake the child frequently during the night so urination is effective
b. symptoms are often a developmental lag and will be outgrown
c. medication therapy has the best long term response rate
d. an alarm system is the best choice as it is usually covered by health insurance
b

In addition to a developmental evaluation, which baseline studies should be included for an infant diagnosed with 22q11 deletion syndrome?
a. echocardiography, thyroid function and renal ultrasound
b. calcium levels, abdominal ultrasound and immunologic screening
c. echocardiography, renal ultrasound, and immunologic screening
d. thyroid function, calcium levels, and abdominal ultrasound
c

Which diagnostic criteria is MOST consistent with a finding of acute otitis media (AOM)?
a. erythema
b. decreased mobility
c. absent landmarks
d. increased vascularity
b

A 4 month old with congenital nasolacrimal duct obstruction (CNLDO) had several episodes of dacryocystitis and is currently hospitalized for cellulitis. What is the appropriate follow up?
a. continued frequent massage over the lacrimal duct
b. daily us of an ophthalmic antibiotic ointment
c. surgical referral for potential tear duct probing
d. oral antibiotic prophylaxis
c

An afebrile 3 year old with no history of trauma presents with a one day history of a limp. Physical exam is negative except for pain with weight bearing. The child has a history of a viral illness one week ago. What is the MOST likely diagnosis?
a. septic arthritis
b. osteomyelitis
c. fracture
d. transient synovitis
d

A 5 year old presents with a 2 cm abscess on the thigh with yellow discharge and no systemic symptoms. What is the first step in the management of this patient?
a.empiric antibiotic treatment
b. obtain culture and await results
c. incision and drainage of the abscess
d. warm soaks and topical bacitracin
c

The parent of a 7 year old is concerned about the child’s dislike of school. It is late fall, and the child has now been absent for 20 days. The FIRST challenge for the parent is to
a. obtain psychological counseling
b. send the child to school everyday
c. determine what the physical illness may be
d. have the child catch up on missed school work
b

Which of the following symptoms is MOST compatible with the diagnosis of sinusitis in a child with persistent sinus symptoms?
a. rhinorrhea and day cough
b. night cough and fever
c. rhinorrhea and night cough
d. fever and headache
a

A teen whose diet consists of frequently eating at fast food restaurants is most likely to have deficits of which nutrients?
a. iron and calcium
b. phosphorous and iron
c. calcium and niacin
d. phosphorous and niacin
a

A 15 year old presents with an infected pierced naval, revealing moderate swelling, erythema, and exudate. The piercing, a gold ring, is one month old and has been maintained with antibacterial soap and water cleansing. The teen reports swimming in a lake one week ago. Which of the following is the MOST appropriate?
a. removal of jewelry from naval
b. oral antibiotic therapy
c. topical therapy
d. cleansing with peroxide
b

The MOST accurate test on a vaginal swab to identify Chlamydia trachomatis in a 3-year-old girl with vaginal discharge is
a. enzyme immunoassay (EIA)
b. culture
c. DNA probe
d. nucleic acid amplification test (NAAT)
b

A previously healthy 4 year old who does not attend day care has acute sinusitis. The most appropriate therapy is
a. amoxicillin
b. amoxicillin/clavulanate potassium
c. ceftriaxone
d. azithromycin
a

A previously healthy 4 year old has had the following immunizations:
6 MONTHS: DTaP/HIB, IPV, Hep B
15 MONTHS: DTap/HIB, IPV, Hep B, MMR ,Varicella
At this visit the most appropriate recommendation is
a. DTAP/HIB, IPV, Hep B, PCV 7
b. DTaP, HIB, Hep B, MMR
c. DTaP, IPV, Hep B, MMR
d. DTaP, IPV, PCV 7
c

Which diagnostic study warrants the MOST attention when evaluating a child with autism?
a. audiologic evaluation
b. EEG to rule out seizure disorder
c. state-mandated metabolic screening
d. brain imaging
a

In acute, uncomplicated sinusitis
a. a 5-day course of high-dose amoxicillin is as efficacious as longer courses of standard-dose antibiotic treatment
b. the course of antibiotic therapy should last a minimum of 7 days after the resolution of symptoms
c. clindamycin is the drug of choice in children with a PCN allergy
d. decongestants are recommended as adjunctive therapy
b

Which test is the MOST sensitive EARLY indicator for musculoskeletal inflammatory disorders?
a. complete blood count with differential (CBC)
b. C-reactive protein (CRP)
c. erythrocyte sedimentation rate (ESR)
d. anti-nuclear antibodies (ANA)
b

In a 2 month old with visible rib fractures on radiograph, the NEXT most critical evaluation to obtain is a
a. CT scan of the head
b. long bone series
c. coagulation profile
d. retinal ophthalmologic exam
a

Which of the follwoing statements regarding alternating doses of ibuprofen and acetaminophen for fever is most accurate?
a. alternating drugs increases risk of dosage errors and toxicity
b. alternating allows for lower total dosage of both drugs
c. alternating drugs results in more rapid resolution of the fever than using either drug alone
d. is appropriate only for children over the age of six months
a

A nurse practitioner is considering a position with a pediatric pulmonologist in a clinic which specializes in the treatment of cystic fibrosis. There are patients in the practice who are over the age of 21. In considering this position, what would be the BEST action to take?
a. turn down the job offer for one in which only pediatric patients will be seen
b. contact the local state board of nursing regarding regulations related to patient populations
c. agree to take the job with the understanding that cystic fibrosis is a pediatric illness, despite the chronological age of the patient
d. contact the certification agency to determine their rulings regarding patients over the age of 21
b

Which is the MOST APPROPRIATE management for a 14-year-old sexually active patient who tests positive for Neisseria gonorrhea and Chlaymdia trachomatis?
a. azithromycin, 1 g orally, single dose
b. ceftriaxone 250 mg IM + azithromycin 1 g orally, single dose
c. ofloxacin 400 mg orally, single dose, to both patient and partner
d. metronidazole 2 g, single dose and doxycycline 100 mg BID x 7 days
b

Of the pathogens causing otitis media, which is MOST LIKELY to require antibiotic therapy?
a. Haemophilus influenzae
b. Steptococcus pneumoniae
c. Moraxella catarrhalis
d. Pseudomonas aeruginosa
b

An 18 month old has a history of multiple episodes of otitis media with recurrent prolonged, febrile seizures lasting longer than 10 minutes. Which is the MOST approriate management?
a. rectal midazolam at onsent of seizure
b. daily anticonvulsants
c. prophylactic antibiotics
d. rectal diazepam at onsent of seizure
d

Which best describes the meaning of sun protection factor (SPF) 6 labeled on sunscreen?
a. application of the sun screen will protect the child from UVA rays for 6 hours
b. the sun screen prolongs the time to sun burn by a factor of 6
c. the sun screen will continue to work 60 minutes in water
d. application of the sun screen will protect the child from UVB rays for 6 hours
b

The parents of a 6 year old reports that the child has been allergic to most fruits since early life. Which history question should be asked FIRST?
a. what were the child’s symptoms?
b. how many different fruits is the child allergic to?
c. has the child had any previous testing?
d. how old was the child with the first reaction?
a

The parent a of 2 week old states that even though the infant was fed Enfamil LIPIL with iron in the hospital, he is now receiving ProSobee LIPIL formula due to concern about allergies in an older child. What is the BEST response?
a. the infant should be switched back to the Enfamil because the ProSobee does not contain an adequate amount of iron
b. the infant should be maintained on Enfamil until there are obvious signs of allergy such as vomiting and bloody diarrhea
c. both formulas contain equal amounts of nutrients including iron
d. both formulas contain carbohydrates in the form of lactose
c

A 10 year old presents with a 0.2-0.3 inch swollen mass centrally located on his left upper eyelid. There is no pain and only mild erythema. The MOST LIKELY diagnosis is
a. external hordeolum
b. coloboma of the eyelid
c. blepharitis
d. chlazion
d

A school-age athletic male presents with a recurrent episode of hematuria. A urinalysis shows: nitrite and leukocyte esterase negative; greater than 50 RBC per high power field; and protein greater than 100 mg/L. The MOST likely diagnosis is
a. myoglobinuria
b. glomerulonephritis
c. drug/medication use
d. urinary tract infection
b

Which scenario represents an example of child ASSENT in health care research?
a. a 5 year old who passively shrugs when asked for permission to participate in a clinical trial
b. a 6 year old who is shown pictures of the research opportunity and agrees with nodding of the head
c. a 9 year old who verbally refuses than consents to participate the following day
d. a 10 year old who is asked by the parents to participate and is silent when approached
c

The MOST APPROPRIATE management of an infant or young child with recurrent wheezing is
a. montelukast
b. budesonide
c. cromolyn
d. proventil
b

The parents of an 18 month old are both overweight. When counseling them about prevention of over weight in their child, suggestions should include
a. measurement of the BMI every 6 months
b. rewards of candy for playing actively
c. thirty minutes of structured play each day
d. stringent parental food controls
c

Which alternative therapy is most likely to be explored by parents of a child with attention deficit hyperactivity disorder (ADHD)?
a. eliminations of food additives
b. use of multi-vitamin therapy
c. elimination of dietary sugar
d. visual training
a

Which criteria is essential for the diagnosis of obsessive-compulsive disorder (OCD) for children?
a. behaviors occur up to an hour a day
b. behaviors are purposeless
c. presence of repetitive thoughts, images, or processes
d. child recognizes the actions are excessive
c

Of the following, the MOST effective method to prevent childhood gun injuries and death is
a. parents preventing children’s access to guns
b. educating children with the Eddie the Eagle program
c. educating children with the STAR Program
d. children’s awareness of gun presence in the home.
a

An example of a pleiotropic genetic disorder is:
A) Marfan’s syndrome
B) Cri du chat syndrome
C) Vitamin D-resistant rickets
D) Tay-Sachs disease
a

A deficiency of which of the following vitamins may cause night blindness, xerophthalmia, or keratomalacia?
A) Vitamin A
B) Vitamin K
C) Vitamin D
D) Vitamin E
a

A 6-month-old infant is brought to the office for a well-child visit. The history obtained from the mother reveals that the infant has had “cold symptoms” for 1 week. The mother reports that the infant’s eating and sleeping habits have not changed and that the infant has not experienced any other symptoms. Physical examination reveals a temperature of 100.9°F (38.3°C) and right otitis media. The immunization record reads as follows:
Birth-Hep B
Age 1 month-Hep B
Age 2 months-DTaP, Hib, Prevnar, and IPV
What immunizations should the PNP administer today?
A) None (The mother should be instructed to bring the infant back for immunizations in 1 month)
B) Because the infant is febrile, only Hep B
C) Hep B, DTaP, Hib, Prevnar, and IPV
D) Hep B, DTaP, Hib, and OPV
c

A 6-year-old child in foster care is brought to the clinic by the foster parents. They are concerned that the child’s immunizations may not be current. The child’s history indicates no contraindications to any immunizations. The family has an official immunization record for the child that lists the following:
Age 2 months DTP, OPV, Hib, and Hep B
Age 4 months DTP, OPV, Hib, and Hep B
Age 6 months DTP, OPV, and Hib
Age 4 years DTP, OPV, Hib, and Hep B
The immunizations recommended for today are as follows:
A) MMR and Var
B) Td and MMR
C) DT, OPV, Hib, and MMR
D) DTaP, IPV, Hib, and MMR
a

The PNP sees a 6-week-old, HIV-exposed neonate in the clinic. The mother received zidovudine (AZT) starting in the second trimester of pregnancy and intravenous AZT during labor. The neonate was started on oral AZT after birth. As a prophylaxis for Pneumocystis carinii pneumonia (PCP) at this 6-week visit, it would be appropriate in conjunction with the HIV specialist managing the infant to:
A) Continue ZDV, and start PCP prophylaxis if the neonate is HIV positive
B) Continue ZDV, and start TMP/SMX
C) Discontinue ZDV, and start PCP prophylaxis if the neonate is HIV positive
D) Discontinue the ZDV, and start trimethoprim/sulfamethoxazole (TMP/SMX, Bactrim)
d

The father of a 10-day-old neonate calls the office concerned about the infant’s continually watery left eye. There is no redness or swelling of the eyelid. The PNP responds:
A) “The neonate probably has a blocked tear duct and should be examined.”
B) “The neonate needs to see a pediatric ophthalmologist.”
C) “The neonate may be having an allergic response to the medicine placed in the eye after birth.”
D) “I will call in a prescription.”
a

Which of the following abnormalities is not multifactorially inherited?
A) Diabetes mellitus
B) Neural tube defects
C) Cleft lip and palate
D) Tay-Sachs disease
d

Individuals living with an infant who has been diagnosed with pertussis should receive:
A) A cephalosporin
B) Pertussis immune globulin
C) Erythromycin prophylactic
D) The pertussis vaccine
c

The PNP understands that many factors complicate the inheritance patterns known as the Mendelian inheritance patterns. Neurofibromatosis can be used as an example of all of the following influencing factors except:
A) Reduced penetrance
B) Genomic imprinting
C) New mutation
D) Variable expression
b

A 3-year-old child is brought to the office for the first time. The child is adopted, and little is known about the prenatal and birth history. A diagnosis of fetal alcohol effects (FAE) is suspected. The physical findings that lead the PNP to suspect this diagnosis are:
A) Carious teeth, hyperactivity, toe walking, and macrocephaly
B) Balance problems, irritability, multiple scars on the arms and legs, and microcephaly
C) Growth retardation, thin or wide lips, flat midface, and finger anomalies
D) Abnormal hair pattern, cherubic lips, temper tantrums, and protuberant belly
c

Which of the following children does not need PCP prophylaxis?
A) An HIV-exposed newborn
B) A 9-month-old, HIV-positive infant with a CD4 lymphocyte count of 1600 (no immune suppression)
C) A 3-year-old, HIV-positive child with a CD4 lymphocyte count of 1000 (no immune suppression) and a history of PCP
D) A 3-year-old, HIV-positive child with a CD4 lymphocyte count of 600 (moderate immune suppression)
d

A 12-year-old child reveals to the PNP during a visit for acne that she may be pregnant. In this community the pregnancy rate among females aged 15 to 19 years is 3 in 100. What is the most important area to explore?
A) The possibility of sexual abuse
B) The expected date of confinement
C) Plans for telling her family
D) The source of prenatal care
a

Most states require that a battery of screening tests be performed on the blood of neonates, usually after the neonate has taken formula or breast milk. Many of the diseases being screened for are rare. The most common disease screened for is:
A) Phenylketonuria
B) Galactosemia
C) Maple syrup urine disease
D) Hypothyroidism
d

A 5-year-old child is brought to a community health clinic for a physical examination and immunizations before starting kindergarten. The father says he is unsure whether the child had chickenpox. The immunization record reads as follows:
Age 1 month Hep B
Age 2 months DTaP, Hib, Prevnar, and IPV
Age 4 months DTaP, Hep B, Hib, Prevnar, and IPV
Age 6 months DTaP, Hep B, Prevnar, and Hib
Age 15 months Hib, Prevnar, and MMR
Age 18 months DTaP and IPV
What would be the most appropriate action?
A) Order a serum varicella titer
B) Administer DTaP, IPV, MMR, and Var
C) Administer one dose of Var now and another dose in 1 month
D) Administer DTaP, IPV, and MMR at this visit and one dose of Var in 1 month
b

The PNP examines a 9-year-old child with a history of reactive airway disease since birth for the first time. In the past 2 weeks the child has had three exacerbations. After evaluating the child, the PNP determines that the mostappropriate medication is:
A) Oral steroids and albuterol
B) Pirbuterol acetate (Maxair) or Salmeterol
C) Albuterol (Ventolin, Proventil)
D) Beclomethasone (Beclovent) or Fluticason (Flovent)
a

The foster mother of a 16-year-old adolescent calls the office to make an appointment. She suspects that the adolescent is “doing drugs” and wants to secretly test for drugs. Which one of the following responses is most appropriate?
A) Explain that such drug testing is unethical
B) Tell the foster mother that drug use is rare in 16-year-old adolescents
C) Schedule an appointment for the adolescent to be tested
D) Schedule separate appointments for the foster mother and the adolescent
d

A 2-year-old child is currently receiving chemotherapy for neuroblastoma. An older sibling is due to receive a poliovirus vaccine before starting kindergarten. The appropriate action at this time is to:
A) Give IPV
B) Withhold IPV, and give it 1 year after the sibling completes chemotherapy
C) Withhold IPV, and give it as soon as the sibling completes chemotherapy
D) Give oral poliovirus (OPV)
a

A 7-year-old child has been seen in the clinic and emergency room several times in the past month for epistaxis. The nosebleeds last approximately 20 minutes and eventually stop with the application of pressure, but the child tends to begin bleeding again within a couple of days. While the PNP is obtaining a family history, the mother states that she experienced frequent nosebleeds as a child. When questioned, the mother states that she now has heavy menses. What is the mostlikely diagnosis for this child?
d

A 13-month-old child is brought to the office for a well-child visit. The child is healthy today and has no history of allergies or reactions to previous immunizations. The PNP reviews the immunization record and finds the following:
Age 5 months DTaP, IPV, Hib, Prevnar, and Hep B
The child should receive the following immunizations today:
A) DTaP, OPV, Hib, Hep B, MMR, and Var (the child should return in 2 months for DTaP, IPV, and Hib)
B) DTaP, IPV, Hib, and Hep B (the child should return in 1 month for MMR and Var)
C) DTaP, IPV, Hib, Prevnar, Hep B, MMR, and Var
D) MMR and Var (the child should return in 2 months for DTaP, IPV, and Hib)
c

When performing a preemployment physical on a 16-year-old adolescent in an urban health care center the PNP learns that the patient’s girlfriend was recently treated for a vaginal infection. Based on this information, the PNP should:
A) Contact the primary care provider of the adolescent’s sexual partner for further information
B) Obtain a urine specimen to rule out chlamydial infection
C) Treat immediately with doxycycline
D) Do no further workup because the patient is asymptomatic
b

The test most commonly used to elicit strabismus in infants and children older than 6 months is the:
A) Snellen Eye Chart
B) Tumbling E Game
C) Cover-uncover test
D) Allen Picture Cards
c

The PNP is managing care for a 5-year-old child with growth failure. After a complete history is taken and a physical examination is performed, the mostrelevant and cost-effective screening tests to obtain are:
A) Tests to determine total protein level, bone x-ray films to determine bone age, urinalysis, and CBC
B) Bone x-ray films to determine bone age, CBC, urinalysis, and tests to determine potassium and chloride levels
C) Bone x-ray films to determine bone age; CBC; T4, TSH, and liver profile tests
D) Tests to determine insulin-like growth factor (IGF-1) levels, CBC, and urinalysis
c

The PNP is examining an infant recently adopted from Western Europe. When should a foreign-born child undergo PPD testing?
A) At the first health examination after entering the United States
B) After the child develops signs and symptoms suggesting TB exposure
C) Never (if the child has had BCG, a chest x-ray examination should be ordered)
D) At the same time as infants born in the United States
a

The PNP is concerned that a child may have A. lumbricoides (roundworm). Which of the following symptoms would lead the PNP to this diagnosis?
A) Fever and runny nose
B) Nausea and vomiting
C) No symptoms
D) Vomiting and rash
c

A 17-year-old adolescent is seen in the “fast track” of the emergency room with symptoms of respiratory distress, headache, and euphoria. The PNP suspects:
A) Glue sniffing
B) Overdose of antihistamines
C) Alcohol ingestion
D) Acetaminophen overdose
a

A husband and wife, both aged 42 years, have adopted a 10-month-old infant from Eastern Europe. The infant has been in an orphanage since birth. The infant’s family history is unavailable. A physical examination of the infant was required before the infant entered the United States, and the results were documented as “normal.” The adoptive parents are bringing the child in for the first well-child visit with the PNP today. What initial screening tests should be performed?
A) CBC, urinalysis, hepatitis B screen, and screening for fetal alcohol effects (FAE) and fetal alcohol syndrome (FAS)
B) Hepatitis B screen, human immunodeficiency virus (HIV) screen, stool test for parasites, and screening for FAE and FAS
C) CBC, Mantoux test, and screening for FAE and FAS
D) CBC, hepatitis B screen, HIV screen, and screening for FAE and FAS
d

A 4-month-old infant is brought to the office for a well-child visit. While giving the infant’s history, the grandmother states that she dreaded coming to the office today because the infant “cried for hours” after the first set of immunizations. The immunization record reads as follows:
Birth Hep B
Age 2 months DTP/Hib, Hep B, Prevnar, and IPV
The following immunizations should be given today:
A) DTaP, Hib, and MMR
B) DT, Hib, Hep B, and IPV
C) DTaP, Hib, Prevnar, Hep B, and IPV
D) DTP/Hib and IPV
b

A 6-month-old infant is brought to the office for a well-child visit. The infant has had several upper respiratory tract infections but is healthy today. The mother states that the infant attends day care 3 days a week. The immunization record reads as follows:
Birth Hep B
4 months DTaP, IPV, Hib, and Hep B
What immunizations should the infant be given today?
A) DTaP, IPV, Hib, Prevnar, and Hep B
B) DTaP, IPV, Prevnar, and Hib
C) DTaP, OPV, Hib, and Rv
D) DTaP, IPV, Hib, and Rv
b

The PNP is examining a 15-year-old Vietnamese adolescent during a health maintenance visit. The adolescent immigrated to the United States 3 weeks ago. Which of the following screening tests is most important at this visit?
A) Mantoux skin test
B) Malaria smear
C) Microfilaria smear
D) Papanicolaou test
a

Once a child is started on oral iron therapy for the treatment of iron-deficiency anemia, what changes are evident in the laboratory studies?
A) There is an immediate rise in the reticulocyte count, and the hemoglobin level returns to normal in 2 weeks
B) There is a rise in the reticulocyte count beginning on the third day, and the hemoglobin level returns to normal in 6 to 10 weeks
C) There is a concurrent rise in the hemoglobin level and the reticulocyte count, with both returning to normal in 3 weeks
D) There is a rise in the reticulocyte count beginning in 2 weeks, and the hemoglobin level returns to normal in 3 to 6 weeks
b

A 12-month-old child is brought to the clinic for routine anemia screening. The hemoglobin level is 9.5 g/dL, and iron sulfate is prescribed at a dosage of:
A) 1 mg/kg/day
B) 10 mg/kg/day
C) 6 mg/kg/day
D) 8 mg/kg/day
c

What should the PNP consider when immunizing a child for measles, mumps, and rubella (MMR) in relationship to a TB skin test?
A) Give MMR first, and give the PPD in 1 to 2 weeks
B) Give PPD first, and 6 months later give MMR
C) PPD and MMR should never be given at the same time
D) Give both at the same time, or give MMR and wait 4 to 6 weeks to apply PPD
d

A child with hemophilia falls off a bike, hurting the knee. The child is brought to the emergency room because the mother notices swelling of the affected joint and the child is complaining of pain. What is the highest priority in this child’s care?
A) Applying ice to the affected knee
B) Obtaining a radiograph of the affected knee
C) Administering factor VIII concentrate
D) Splinting the affecting knee
c

A pregnant woman who has had no prenatal care comes to the hospital in active labor. After delivery, the enzyme-linked immunosorbent assay (ELISA) and Western blot laboratory studies performed upon admission reveal that the mother is HIV positive. The priority for the PNP caring for the neonate in the nursery is to start:
A) PCP prophylaxis with TMP/SMX
B) AZT the day of discharge
C) AZT within 24 hours of birth
D) Dapsone orally within 48 hours of birth
c

An 8-year-old child brought to the office by the parent has a cough, nasal congestion lasting 3 days, and an oral temperature of 99°F. After the PNP has reviewed the history and completed a physical assessment, a viral upper respiratory tract infection is diagnosed. The child has a history of varicella. A review of the immunization record reveals the following:
Age 2 months DTP, OPV, and Hib
Age 5 months DTP, OPV, and Hib
Age 7 months DTP, OPV, and Hib
Age 18 months DTP, Hib, and MMR
Age 5 years DTP and OPV
What is the suggested course of action regarding the child’s immunization status?
A) Start the Hep B series today, and have the child return in 1 month for the next Hep B
B) Start the Hep B series and give MMR; have the child return in 1 month for the next Hep B
C) Give no immunizations today because the child has a febrile illness
D) Give no immunizations because none are due until age 11 years
b

A 4-year-old child has had a productive cough with yellow-green sputum but no fever for 7 days. In developing a plan of treatment, the PNP understands the following:
A) The production and color of the sputum may indicate a bacterial infection
B) Prolonged cough illnesses are often allergic or viral in nature
C) A cough that continues up to 14 days after onset is probably a bacterial infection
D) Infection and reactive airway disease are common causes of productive cough in children
d

For which of the following individuals is there no indication for obtaining a chromosomal analysis?
A) A child with phenylketonuria (PKU)
B) A child with multiple congenital anomalies and mental retardation
C) A couple with a history of multiple miscarriages
D) A man with large testes and mental retardation
a

In what Tanner stage for girls would the PNP expect to see enlargement of the breasts and areolae, as well as curly, coarse pubic hair?
A) Tanner 4
B) Tanner 2
C) Tanner 3
D) Tanner 5
c

It is early fall, and an 18-month-old child with a history of asthma is brought to the office because of an acute exacerbation. The PNP reviews the child’s immunization record with the mother. The child’s breathing improves after a nebulizer treatment. While giving the history, the mother states that the child gets a rash after eating eggs. The immunization record reads as follows:
Birth Hep B
Age 2 months DTaP, IPV, Hib, Prevnar, and Hep B
Age 6 months DTaP, IPV, Hib, and Prevnar
Age 10 months DTaP, Hib, Prevnar, and Hep B
The following immunizations should be given today:
A) DTaP, IPV, Hib, MMR, Var, and flu vaccine
B) DTaP, IPV, Hib, Prevnar, MMR, and Var
C) None, because the child is acutely ill
D) DTaP, IPV, Hib, and Var
b

A 15-year-old adolescent is brought to the office for a routine examination. The adolescent is healthy and has no history of allergies or reaction to previous immunizations. There is no reliable history of varicella. The PNP reviews the adolescent’s immunization record and finds the following:
Age 3 months DTP and OPV
Age 5 months DTP and OPV
Age 9 months DTP and OPV
Age 18 months DTP, Hib, and MMR
Age 5 years DTP, OPV, and MMR
The PNP should administer the following immunizations today:
A) Td, Hep B, and Var (the adolescent should be asked to return in 1 month for another Hep B)
B) Td and Hep B (the adolescent should be asked to return in 1 month for another Hep B)
C) Hep B, IPV, and Var (the adolescent should be asked to return in 1 month for another Hep B)
D) Var and MMR (the adolescent should be asked to return in 1 month for another Var)
a

An example of a genetic disorder occurring because of genomic imprinting is:
A) Marfan’s syndrome
B) Prader-Willi syndrome
C) Trisomy 18
D) Down’s syndrome
b

A 13-year-old adolescent is brought to the office because of a partial-thickness burn on the leg caused by a campfire. The immunization record reads as follows:
Age 3 mo-DTP, OPV, & Hib
Age 7 mo-DTP, OPV, & Hib
Age 10 mo-DTP, OPV, & Hib
Age 20 mo- DTP, Hib, & MMR
Age 5 years-DTP and OPV
The following immunizations should be administered today:
A) Td, IPV, and Var (if there is no history of disease)
B) None (because a tetanus-containing vaccine was administered within the last 10 years)
C) Td, Hep B, and MMR
D) Td, MMR, Hep B, and Var (if there is no history of disease)
d

A single, 15-year-old mother who attends high school brings a 3-month-old infant to the clinic. The infant seems to be healthy and thriving, but the mother is vague when reporting the infant’s feeding and sleeping habits. To get more accurate information about the specifics of the infant’s routine and care, the PNP should:
A) Make a referral to a home nursing agency
B) Ask about the day care arrangements
C) Ask the mother if she has a learning disability
D) Make a referral to a social service agency
b

A 14-month-old child is brought to the office for a well-child visit. The child is healthy today and has no history of allergies or reactions to previous immunizations. The family lives with a grandmother who is currently undergoing chemotherapy for cancer. The PNP reviews the child’s immunization record and finds the following:
Birth-Hep B
Age 2mo-DTaP, IPV, Hib, & Hep B
Age 4mo-DTaP, IPV, and Hib
Age 6mo-DTaP and Hib
Age 9mo-Hep B
The PNP should give the child the following immunizations today:
A) DTaP, IPV, Hib, and Var
B) DTaP, IPV, Hib, MMR, and Var
C) DTaP, IPV, Hib, and MMR
D) DTaP, IPV, and Hib
b

Diagnostic studies considered part of normal screening for the child with developmental delays of unknown etiology include chromosomal studies or karyotyping, deoxyribonucleic acid (DNA) studies to detect fragile X syndrome, and:
A) Measurement of urine and plasma amino acid levels
B) Microscopic urinalysis
C) Cystometrography
D) Measurement of serum antinuclear antibody levels
a

A 2-year-old child is brought to the clinic for a well-child visit. The mother expresses concern about the child’s language development. Which of the following limitations would be an indication that this child has a language delay?
A) The child can name two pictures
B) The child has a vocabulary of 20 words
C) The child’s speech is halfway understandable
D) The child can put two words together
b

When evaluating a 12-year-old child with a 10-day history of rhinitis and facial pain, the testing most effective in determining sinusitis is:
A) Other upper respiratory tract conditions can result in abnormal radiographic findings, so sinus aspirate is the only positive test
B) Radiographic studies are most accurate in the early stages of a respiratory infection
C) X-ray opacities in the sinuses confirm bacterial sinusitis
D) Some mucosal thickening can be seen only on computed tomography or magnetic resonance imaging, so these studies should be ordered in place of plain films
a

What is the probability of a couple having a child with cystic fibrosis (CF) if the mother carries the △F508 gene and the father does not?
A) 0%
B) 100%
C) 25%
D) 50%
a

A 15-month-old child who began treatment for leukemia 3 weeks ago is brought to the clinic by the parents for a previously scheduled well-child visit. The immunizations that are due today include an inactivated poliovirus (IPV) vaccine and the measles, mumps, rubella (MMR) vaccine. What immunizations should be administered?
A) Administer IPV today and delay MMR until 1 year after treatment has been completed
B) Delay both vaccines until treatment for leukemia is completed
C) Administer vaccines as scheduled
D) Delay both vaccines until 1 year after treatment for leukemia has been completed
a

Some children are at increased high risk for complications of influenza. Of the following who is at greatest risk for complications and should have priority for influenza vaccination?
A) Tetralogy of Fallot
B) Chronic otitis media
C) Stevens-Johnson syndrome
D) Erythema multiforme major
a

Which hepatitis virus vaccines are available and approved for use in children?
A) Hepatitis A and C
B) Hepatitis B and C
C) Hepatitis A, B, and C
D) Hepatitis A and B
d

A 10-month-old infant is assessed for ocular alignment with the corneal light reflex test. The PNP is testing for:
A) Anisometropia
B) Strabismus
C) Functional loss of vision
D) Hyperopia
b

A 10-year-old child is brought to the clinic for a follow-up visit after an exacerbation of asthma 3 days ago. The child has symptoms more than twice a week but less than once a day. The attacks sometimes affect activity. At least twice a month the child wakes up at night with wheezing. FEV1/PEFRs are 80% or more of predicted value, with a variability of 20% to 30%. This patient’s asthma would be classified as:
A) Mild persistent
B) Mild intermittent
C) Severe persistent
D) Moderate persistent
d

The PNP is seeing a 1-month-old infant for routine follow-up. The infant received both the Hep B and the hepatitis B immune globulin (HBIG) vaccines because the mother is hepatitis B surface antigen (HBsAg) positive. At this visit the PNP should:
A) Obtain a Hep B core antigen
B) Obtain an anti-HBe
C) Readminister only the HBIG
D) Give the second dose of Hep B
d

A low-birth-weight neonate is examined in the clinic. The mother is Native American and smoked heavily during the pregnancy. The PNP recognizes that the Native American population has an increased incidence of:
A) Sudden infant death syndrome (SIDS)
B) Reactive airway disease
C) Developmental disability
D) Neonatal sepsis
a

A 5-year-old child was born in Mexico and received bacille Calmette-Guérin (BCG) vaccine as an infant. A PPD is administered as part of a routine office visit. After 48 hours, there is a 10-mm reaction. How should the PPD be interpreted?
A) Interpret the PPD as if the child had not received BCG
B) This child should not have undergone PPD testing because the result will always be positive
C) Interpret the PPD as a normal reaction
D) This child should not have undergone PPD testing because the result will always be negative
a

What is the American Academy of Pediatrics’ recommendation for routine skin testing for TB in a child with no risk factors who lives in a low prevalence area of the country?
A) Test at age 3 months, 1 year, and yearly thereafter
B) PPD should be performed yearly
C) Routine skin testing is not indicated in this group
D) Test at age 1 year, entrance to kindergarten, and high school
c

A 12-month-old infant is brought to the office for a well-child visit. The infant is healthy today, has tested positive for HIV, and has no history of allergies or reactions to previous immunizations. The mother is HIV positive. The child’s immunization history is as follows:
Birth Hep B
Age 3 months DTaP, Hib, Prevnar, Hep B, and IPV
Age 6 months DTaP, Hib, Prevnar, and IPV
Age 9 months DTaP, Hib, Prevnar, and Hep B
The PNP should administer the following immunizations today:
A) IPV and Var
B) MMR and Var
C) IPV, HIB, Prevnar
D) MMR, Prevnar, HIB
c

A 3-year-old child with sickle cell disease is brought to the clinic for a well-child visit. What is an appropriate intervention to decrease the child’s risk of infection?
A) Obtain a CBC with differential and platelet count every 6 months
B) Obtain a yearly Hgb electrophoresis and administer the pneumococcal vaccine annually
C) Prescribe penicillin 20 mg/kg and administer Hib at this visit
D) Continue penicillin prophylaxis 20 mg/kg and administer the pneumococcal vaccine at this visit
d

A 14-year-old adolescent comes to the school-based clinic for a physical examination. During the interview the adolescent claims to be a member of a gang. What response would be appropriate in the initial discussion with this adolescent?
A) “Can you tell me if you are currently using drugs?”
B) “Can you tell me who else belongs to your gang?”
C) “Tell me more about your relationship with your family and close friends.”
D) “Have you ever been arrested?”
c

A 7-year-old child with no record of prior immunizations received initial immunizations including Td, IPV, MMR, Var, and Hep B today. When should the child return for the second set of immunizations?
A) In 6 months
B) In 2 weeks
C) In 1 year
D) In 2 months
d

An 8-year-old child who was born and raised in Vermont is healthy and has no known risk factors for tuberculosis (TB). A PPD skin test results in 7-mm induration and redness. How should this be interpreted?
A) As a negative skin test result
B) As a positive skin test result
C) As a negative skin test result (but testing should be repeated in 3 months)
D) As an unequivocal skin test result (and testing should be repeated now)
a

Diagnostic studies included in the routine screening of the child with developmental delays of unknown etiology include chromosomal karyotyping, deoxyribonucleic studies (for fragile X syndrome), and:
A) Cystometrography
B) Tests to detect urine and plasma amino acids
C) Microscopic urinalysis
D) Tests to detect serum antinuclear antibodies (ANAs)
b

A 7-year-old child with factor IX-deficient hemophilia falls off the slide at recess and hits his head on the ground. Since returning home from school, the child has complained of headache and dizziness. What actions should the PNP take first?
A) Order magnetic resonance imaging, and then perform a complete neurologic examination
B) Administer factor IX concentrate
C) Perform a complete neurological evaluation to rule out CNS injury
D) Order a computed tomography (CT) scan of the head
b

A 13-month-old child is brought to the clinic for a routine visit. The child is healthy today and has no history of allergies or previous reactions to immunizations. The PNP checks the immunization record and finds the following:
Age 2 months DTaP, IPV, Hib, Prevnar, and Hep B
Age 5 months DTaP, IPV, Hib, Prevnar, and Hep B
Age 8 months DTaP and Hib
The PNP should give the child the following immunizations today:
A) DTaP and IPV
B) MMR and Var (a return visit should be scheduled for 1 month later so that Hep B can be administered)
C) Hep B, MMR, IPV, HIB, Prevnar, and Var
D) Hep B and MMR (a return visit should be scheduled for 1 month later so that Var can be administered)
c

A 15-year-old adolescent comes to the school-based clinic with the complaint of nausea in the morning and breast tenderness. Menses are 5 days late. It is not abnormal for her to be late. A pregnancy test is performed and is positive. The PNP advises the adolescent to first:
A) Discuss the situation with her parents
B) Discuss the results with the adolescent to determine what this means to her
C) Consult with an obstetric physician or adolescent clinic
D) Repeat the pregnancy test in a week
b

A 6-month-old infant who is new to the practice is brought to the office for a well-child visit. The infant is healthy today and has no history of allergies or reactions to previous immunizations. The PNP reviews the immunization record and finds the following:
Birth Hep B
Age 2 months DTaP, IPV, Hib, Prevnar, and Hep B
Age 4 months DTaP, IPV, Prevnar, and Hib
Today the infant should receive:
A) DTaP, Hib, Prevnar, and Rv
B) Hib, Hep B, and Rv
C) DTaP, Hib, Prevnar, and Hep B
D) DTaP, Hib, and IPV
c

A 5-month-old infant is brought to the office for a routine well-child visit. The infant is healthy today and has no history of allergies or reactions to previous immunizations. The PNP checks the immunization record and finds the following:
Birth Hep B
Age 1 month Hep B
Age 2 months DTaP, IPV, and Hib
Today the infant should receive the following immunizations:
A) DTaP, IPV, Hib, and Var
B) DTaP, IPV, Hib, Prevnar, and Hep B
C) DTaP, IPV, Hib, Prevnar
D) DTaP, OPV, and Hib
c

The PNP sees a 6-week-old, HIV-exposed neonate in the clinic. The mother received ZDV starting in the second trimester of pregnancy and intravenous ZDV during labor. The infant was started on oral ZDV after birth. The most accurate test to determine whether the infant is HIV positive is:
A) Ribonucleic acid (RNA) PCR
B) ELISA
C) Western blot
D) Deoxyribonucleic acid (DNA) polymerase chain reaction (PCR)
d

Which of the following genetic disorders can be detected by cytogenic testing?
A) Huntington’s chorea
B) Sickle cell anemia
C) Tay-Sachs disease
D) Down’s syndrome
d

A 14-year-old adolescent is brought to the office because of an asthma exacerbation. The PEFRs over the past 2 days have measured 50% to 80% of personal best. This measurement of maximal flow represents which zone on a peak flow meter?
A) Green
B) Yellow
C) Orange
D) Red
b

The mother of a 10-year-old girl is concerned that the child has been sexually molested. Upon genital examination of a prepubescent girl after penile penetration, the PNP would expect to find:
A) Permanent tears of the vaginal wall
B) An intact hymen
C) Normal-appearing genitalia
D) A wide-open vaginal orifice
c

An 18-month-old is seen in the emergency room with a second-degree “glove distribution” burn. A more detailed history is needed because of the concern for:
A) Staphylococcal scalded skin syndrome
B) Immersion injury caused by child abuse
C) Possible contact with a caustic material
D) Accidental burn from an iron
b

A 2-week-old infant comes to clinic accompanied by his 15-year-old mother. The primary goal for this visit is to discuss:
A) Available birth control methods
B) Establishing fatherhood through the legal system
C) Returning to school immediately
D) Specific parenting skills by using concrete examples
d

A 4-year-old is seen in the office for a preschool physical. The past history is significant for hospitalization for Kawasaki’s disease and treatment with intravenous immune globulin (IG) 2 months ago. The immunization record indicates that the child needs a DTaP, polio, and MMR at this time. What immunizations should the PNP administer today?
A) DTaP, OPV, and MMR at this visit, and instruct the parent to schedule a return visit in 8 weeks
B) MMR only at this visit
C) None, give immunizations at the school physical next year
D) DTaP and IPV, and instruct the parent to schedule an appointment in 1 month for the MMR
d

The PNP is evaluating a postterm (42 weeks’ gestation) neonate in the nursery. On the physical examination the PNP would expect to find:
A) Excessive vernix caseosa
B) Smooth soles of the feet
C) Cracked, peeling skin
D) An elevated weight/length ratio
c

At an 18-month-old child’s well-child visit, the father states that the child is a picky eater and has 4 oz of milk by bottle at nap time and 8 oz of milk by bottle at bedtime. The rest of the child’s fluids are taken by cup. Of the following, what is the most pertinent information for the PNP to obtain?
A) The total intake of milk for 24 hours
B) A 24-hour recall diet
C) A list of foods the child refuses
D) The family’s financial status
b

Routine pediatric office developmental testing is conducted to:
A) Diagnose children with mental retardation
B) Diagnose children with learning disabilities
C) Determine the child’s school readiness
D) Identify children who may require formal evaluation
d

A macrobiotic diet can best be described as a diet containing:
A) No animal foods, including milk and eggs
B) Whole grains, vegetables, fruits, and white meat of fish
C) Grains, fruits, vegetables, and milk but no eggs, meat, poultry, or seafood
D) Grains, fruits, vegetables, milk, eggs, fish, and fowl but no red meat
b

Precocious puberty is defined as sexual development beginning before the youngest acceptable age. Which of the following statements is true regarding precocious puberty?
A) The intellectual, emotional, psychosocial, and psychosexual development of these children far exceeds their sexual maturity rating
B) Short stature in adulthood may result from the rapid skeletal maturation attributable to the early secretion of sex hormones
C) The sexually precocious child rarely encounters psychosocial difficulties
D) Precocious puberty is not usually idiopathic in cause
b

Which of the following methods is best for diagnosing obesity?
A) Skinfold measurements
B) BMI
C) Weight for height measurements
D) Mother is obese
b

Which of the following statements is true regarding childhood obesity?
A) The most common problem associated with childhood obesity is diabetes
B) If one parent is obese and the other is of normal size, 9% of their children will become obese
C) The older the obese child is, the more likely it is that he or she will become an obese adult
D) Endocrine causes of obesity are common
c

The PNP sees a 4-week-old neonate for a well-child visit. The parents report that the infant seems to eat well but does not gain weight and has frequent, greasy-looking stools. The PNP suspects that the neonate might have:
A) Cystic fibrosis
B) Failure to thrive
C) Gastroenteritis
D) Intussusception
a

The first sign of puberty in girls is usually:
A) Pubic hair growth
B) Thelarche
C) Menarche
D) Acne
b

The PNP is examining a 12-year-old female child with enlarged breasts and areolas and curly, dark, coarse pubic hair. The PNP uses Tanner staging and classifies this child as:
A) Tanner stage II
B) Tanner stage III
C) Tanner stage IV
D) Tanner stage V
b

A 5-year-old child is brought to the clinic for immunizations. The child has never had immunizations. What immunizations should the child receive today?
A) Tetramune, IPV, MMR, and Varivax (and should return in 6 weeks)
B) DPT, IPV, and Varivax (and should return in 8 weeks)
C) DTaP, IPV, MMR, Varivax, and Hep B (and should return in 12 weeks)
D) DTaP, IPV, MMR, Varivax, and Hep B (and should return in 6 weeks)
d

A 2-week-old neonate is brought to the clinic for a well-child visit. The infant weighed 3.18 kg at birth; today the weight is 2.95 kg. The mother is breastfeeding about every 2 hours; the infant is latching on well and nursing 15 minutes per side. How many calories a day does this infant need for growth?
A) 500 Kcal
B) 375 Kcal
C) 345 Kcal
D) 300 Kcal
c

The first nurse practitioner program was designed to:
A) Provide emergency care to children in Colorado
B) Provide limited access to health care for low-income children in Colorado
C) Provide increased access to health care for children in Colorado
D) Manage severely ill children at Colorado General Hospital
c

National certification of PNPs has been offered since the 1970s. The following credentials may identify nurses who have national specialty certification:
A) Master of science in nursing (MSN)
B) Advance practice nurse (APN)
C) Certified (C)
D) Nurse practitioner (NP)
c

The most common form of reimbursement for services provided by PNPs is:
A) Fee charged at 85% of the physician’s payment
B) Fee charged at 90% of the cost under the physician’s billing
C) Fee for service
D) Same fee as physician, “equal pay for equal service”
c

In most cases the PNP is covered by employee professional liability for:
A) Off-duty situations, such as athletic physicals not sponsored by the agency
B) Off-duty personal liability
C) Off-duty coverage for personal injury, such as discussing a patient in public
D) Those acts covered as normal work-related services and duties
d

The PNP is employed by a physician and performs services under direct supervision of the physician; these services provided by the PNP are billed under the physician’s name and are called:
A) Incidence to service
B) Direct billing for service
C) Capitation reimbursement
D) Salary
a

In the health care system the major perceived barrier to use of an PNP’s services in a primary health care setting is:
A) Lack of consumer confidence
B) Lack of quality of service provided
C) Organized medicine
D) Organized nursing
c

Title V of the Social Security Act of 1935 was amended in 1981 and 1986 to expand coverage for children with special health care needs under:
A) Title XIX of the Social Security Act
B) The Supplemental Security Income (SSI) Program for the Aged, Blind, and Disabled
C) The U.S. Public Health Service
D) The Maternal and Child Health Services Block Grant
d

Legal authority for prescriptive privileges for PNPs is granted by the:
A) Food and Drug Administration
B) State Nurse Practice Act
C) Nurse practitioner certification boards
D) State pharmacy licensing boards
b

The School Lunch Program; Women, Infants, and Children (WIC) Program; and Food Stamp Program are administered by:
A) The Food and Drug Administration
B) The U.S. Department of Agriculture
C) Social Security Block Grants
D) The U.S. Department of Public Health
b

Standards for well child care/health promotion are set forth by the:
A) Task Force, Preventive Services, Department of Health and Human Services
B) American Academy of Family Physicians
C) American Academy of Pediatrics (AAP)
D) Healthy People 2000/2010 documents
c

A 6-year-old child is brought to the clinic with complaints of eye pain. The history reveals a sudden onset of irritation and pain in the left eye, which began at recess. The child refuses to open the eye. A corneal abrasion or foreign body is suspected. The diagnostic test of choice is:
A) Instillation of mydriatic drops and microscopic examination of the globe
B) Visual acuity testing with the Snellen Chart
C) Culture of any discharge from the eye
D) Instillation of fluorescein dye and examination with a Wood’s lamp

Feedback: INCORRECT
At the site of the corneal abrasion, there is increased uptake of fluorescein dye. The abrasion can then be visualized with a blue-filtered or Wood’s lamp. Corneal abrasions occur when superficial epithelium is broken. There is pain, and the child keeps the eyelid closed. Photophobia may be present.
d

Medical professionals are legally required to report abuse when:
A) Appropriate resources are available to investigate
B) Abuse is suspected
C) Forensic evidence is collected
D) Protocols are used to evaluate the report
b

When approaching adolescents about sensitive issues, such as drug use and sex, an effective interviewing approach is to:
A) Introduce the topics early in the history
B) Let the adolescent know the parents are concerned
C) Begin by asking about their friends’ activities
D) Avoid promising confidentiality
c

A 2-year-old child is brought to the office with a history of continual wheezing and persistent cough. The father reports that the child experienced a coughing and choking episode at a cookout about a week ago that resolved spontaneously. A few days later the child began to cough and wheeze. The PNP suspects the diagnosis of:
A) Vascular ring
B) Bronchiolitis
C) Foreign body aspiration
D) Reactive airway disease
c

A 16-year-old adolescent went jogging wearing a nylon sweat suit in 90°F heat. The mother found the confused adolescent at home. The PNP should initially:
A) Give Gatorade to replenish fluids, and send the adolescent to the emergency room
B) Start oral rehydration, remove the clothing, and elevate the legs
C) Evaluate the adolescent for neurologic deficits
D) Refer the adolescent to the emergency room immediately
b

The PNP receives a call from a mother with a crying 4-year-old child. The child, who was tired and oppositional, darted away from the mother, who took hold of the arm at the moment the child darted. The child immediately began to cry inconsolably. There has been no history of illness. The PNP suspects that the child:
A) Has a behavior problem and should be scheduled for an evaluation as soon as possible
B) Has sustained a stress fracture as a result of the incident and should be referred to an orthopedic specialist
C) Has dislocated the elbow and should be examined immediately
D) Had a major temper tantrum (the PNP plans to discuss appropriate management with the mother at the next visit)
c

In assessing the sinuses of children, it is important to understand the following about normal development:
A) The maxillary and ethmoid sinuses are present at birth
B) Only the ethmoid sinuses are present at birth
C) Sphenoid sinuses develop between age 18 and 24 months
D) Frontal and sphenoid sinuses begin to appear between age 3 and 4 years
a

A 7-year-old child is brought to the emergency room in respiratory distress. The PNP diagnoses the child with an acute asthma attack. It would be mostappropriate to administer which of the following medications:
A) A leukotriene-blocking agent
B) A steroid
C) A short-acting beta-agonist
D) A mast cell inhibitor
c

An ill-appearing 10-month-old infant is brought to the clinic with fever, tachypnea, and cough. No one else in the family is currently ill. Which of the following laboratory tests would be most useful in the initial evaluation of this infant?
A) Complete blood cell count
B) Blood culture
C) Nasopharyngeal culture
D) Chest radiography
d

A 3-week-old infant is is brought to the clinic with difficulty breathing and feeding. The mother reports that the infant’s cry has become hoarse. The infant is bottle-fed and is not offered honey with feedings. The most likely diagnosis is:
A) Botulism
B) Laryngeal web
C) Viral infection
D) Vocal nodules
b

A young child is brought to the clinic. The mother reports that the child has had nasal congestion and a runny nose with a green discharge for 3 weeks. Which physical finding would be particularly worrisome and indicate the need for further evaluation?
A) Unilateral nasal drainage
B) Sporadic, loose cough
C) A low-grade fever for 3 days
D) Fussiness
a

A 9-year-old child is brought to the clinic with facial pain, purulent nasal discharge, and tenderness over the maxillary sinus for the past 4 days. After completing the physical examination, the PNP should:
A) Order radiologic studies
B) Treat the condition empirically with amoxicillin
C) Order culture and sensitivity tests of the nasal discharge
D) Treat the condition with sinus irrigation
b

A 2.5-year-old is brought to the clinic because of vomiting, low-grade fever, and frequent watery stools for the past 24 hours. The PNP suggests:
A) Increasing fluid and a modified diet of bananas, rice, applesauce, and toast
B) Clear fluids and rest for 24 hours
C) Oral antidiarrheal agents such as Lomotil
D) A diet of bananas, rice, applesauce, toast, and milk
a

A 15-year-old comes to clinic with painful vesicular skin lesions. The most likely diagnosis is:
A) Impetigo
B) Herpes infection
C) Coxsackievirus infection
D) Molluscum contagiosum
b

The PNP suspects that a sexually active adolescent has pityriasis rosea. Based on the differential diagnosis, the PNP screens for:
A) Chlamydia
B) Secondary syphilis
C) Erysipelas
D) Molluscum contagiosum
b

A 14-year-old comes to the school-based clinic with chest pain. The most common cause of cardiac pain in children and adolescents is:
A) Myocarditis
B) Tachycardia
C) Mitral valve prolapse
D) valve prolapse
c

A dark-skinned 16-year-old presents in clinic with several areas where the skin pigment appears to be lost. These areas are not itchy. The PNP diagnoses tinea versicolor and prescribes:
A) Selenium sulfide shampoo and systemic ketoconazole
B) Silver sulfadiazine cream and an antihistamine
C) Steroid cream and erythromycin
D) Systemic ketoconazole and steroid cream
a

A 4-year-old has a 3-day history of sore throat, fever (101ºF to 103ºF), headache, and one episode of vomiting. The physical examination reveals +2 to +3 tonsils, palatal petechiae, erythema, and a clear nasal discharge. The PNP’s best course of action is to:
A) Discuss medications for seasonal allergies
B) Prescribe an antibiotic because the child has classic symptoms of a strep throat
C) Perform a throat culture before prescribing an antibiotic
D) Suggest only symptomatic treatment, because the causative virus needs to run its course
c

A well-child examination has been completed on a 1-year-old child who appeared slightly pale. The results of the routine CBC are as follows: hemoglobin, 7.8 mcg/dL; mean corpuscular volume, decreased; and red cell distribution width, elevated. The remainder of the CBC is normal. What would be the most likely cause of this child’s anemia?
A) Thalassemia
B) Megaloblastic anemia
C) Iron deficiency
D) Lead poisoning
c

The most appropriate treatment for axillary adenitis in a 3-year-old on the day after sustaining two small puncture wounds from the family dog would be:
A) Amoxicillin/clavulanate potassium
B) Dicloxacillin
C) Cephalexin
D) Rabies immune globulin
a

What causes physiologic jaundice (unconjugated hyperbilirubinemia)?
A) Decreased reabsorption of bilirubin from the intestine
B) Decreased survival of fetal red blood cells
C) Increased caloric intake
D) Decreased red blood cell volume
a

A father brings his 15-month-old to the clinic reporting that the child is constantly ill and presently has nasal congestion and a runny nose. The child has a history of four episodes of otitis media and associated cough that is loose and sporadic in nature. The father and the extended family are convinced that the child has allergies. What should the PNP do next to manage this child?
A) Refer the child for intradermal allergy testing
B) Investigate the child’s exposure to illness (including day care)
C) Determine whether the child is teething
D) Prescribe an antihistamine
b

A 9-year-old presents with irritability and a history of multiple seasonal allergies. The physical examination results are normal. The PNP suspects the cause of the irritability to be:
A) Vitamin C
B) Monosodium gluconate (MSG)
C) Antihistamines
D) Ibuprofen
c

An 8-year-old is brought to the clinic with “hard bumps” in the neck area and axillae for the past week. The child has been living with grandparents for the past 6 months and visits a nursing home frequently. The history is negative for other symptoms such as weight loss. Upon exam the nodes are enlarged but painless. The PNP is suspicious of:
A) Epstein-Barr virus (EBV)
B) Cytomegaly virus (CMV) infection
C) Histoplasmosis
D) Tuberculosis (TB)
d

While examining a 2-month-old, the PNP notices leukocoria. This is the most common physical finding associated with:
A) Neuroblastoma
B) Retinoblastoma
C) Retrolental fibroplasia
D) Congenital cataracts
b

Which of the following patients should be referred for further developmental evaluation?
a. Term 12-month-old child not walking independently whose older sibling walked at 10 months
b. Premature 7-month-old child born at 28 weeks gestation not sitting independently
c. Term 12-month-old infant not using single words
d. Term 6-month-old infant with poor head control
d

An infant with gastroesophageal reflux is being discharged from the hospital with a nasogastric tube for feeding. Which of the following would be most effective for teaching the patient’s family how to care for the nasogastric tube at home?
a. Arrange for nasogastric tube teaching after discharge with a home health care nurse
b. Have the family observe a nurse placing the feeding tube before discharge
c. Teach the primary caregiver how to care for the feeding tube before discharge and tell him/her to teach the rest of the patient’s home caregivers
d. Have the family observe, help with in-hospital feeding tube care, and then demonstrate independent skills with the feeding tube prior to hospital discharge
d

Which of the following is accurate about the emancipated minor?
a. The adolescent patient who disagrees with a parent/guardian about medical treatment can be treated as an emancipated minor
b. All 50 states have identical legal statutes with regard to the emancipated minor.
c. The emancipated minor may not legally consent to medical care involving reproductive health issues
d. The emancipated minor may legally consent to all types of medical care
d

Which of the following is most accurate regarding pain assessment in the pediatric patient?
a. Behavioral pain assessment measures are useful for measuring pain in infants or children with impaired communication skills
b. Most pediatric patients can use self-report pain scales (e.g., FACES, 0-10 scales) by 2 years of age
c. Premature neonates are neurologically less capable of feeling pain
d. Parents are an unreliable source of pain assessment in the cognitively impaired pediatric patient
a

An important assumption underlying the “family systems theory” is:
a. Problems in the family can be traced back to individual family members
b. Family dysfunction is best addressed by emphasizing past (rather than current) family dynamics.
c. The family is a closed system which does not interact with its environment
d. Changes in one part of the family affect all other parts of the family system
d

An important benefit of physical therapy for the patient with a moderate-to-severe ankle sprain is:
a. Therapeutic exercises assist with regaining full range of motion and stability of ankle joint
b. Rapid return to sports in the competitive athlete
c. Home ankle rehabilitation exercises are unnecessary
d. Lower incidence of compartment syndrome in association with ankle sprain
a

In evaluation of the healthy, toddler-age patient with asymptomatic microcytic anemia, the laboratory test(s) most likely to be diagnostic is:
a. Serum lead level
b. Serum folate level
c. Serum iron studies
d. Serum white blood cell count
c

The leading cause of death for children 1-18 years of age in the United States is:
a. Accidents
b Homicide
c. Cancer
d. Congenital anomalies
a

Which of the following statements is most accurate concerning the pediatric nurse practitioner’s scope of practice?
a. Prescribing medications is not within the pediatric nurse practitioner’s scope of practice
b. Pediatric nurse practitioners diagnose and treat most common childhood illnesses
c. Pediatric nurse practitioners may practice independently without physician oversight in all 50 states
d. Pediatric nurse practitioners may only provide routine health maintenance and immunizations
b

Which of the follow parental characteristics is associated with an increased risk of child maltreatment (abuse)?
a. Older parents
b. Immigrant parents
c. History of childhood abuse/neglect for the parent
d. Wealthy parents
c

Which is considered the strongest (or highest) level of evidence among the following choices?
a. Systematic review of randomized controlled trials
b. Expert opinion
c. Systematic review of case-controlled studies
d. Case series
a

The pediatric nurse practitioner performs the “cover test” on a 3-year-old patient to screen for strabismus. When the left eye is covered, the right eye moves outward to fix on a near object. The cover test has revealed which of the following conditions?
a. Pseudostrabismus
b. Left eye strabismus
c. Right eye strabismus
d. Normal ocular alignment
c

Which of the following is accurate about transmission of an autosomal recessive trait, such as the defective gene of cystic fibrosis?
a. 25% of children born to two carriers of the gene mutation will be affected by the disease
b. All male offspring born to two carriers of the gene mutation will be affected by the disease, while none of the female offspring will be affected
c. Half of the female offspring born to two carriers of the gene mutation will be carriers of the gene mutation and half of the male offspring will be affected by the disease
d. 50% of all offspring born to two carriers of the gene mutation will be affected by the disease
a

In which of the following age groups would inflammatory bowel disease (e.g., Crohn’s disease, ulcerative colitis) most likely present?
a. Adolescent (12-18 years)
b. Neonate (0-1 month)
c. Toddler (2-4 years)
d. School-age (5-10 years)
a

Family-centered care is characterized by which of the following?
a. Care team acts as the experts and instructs the patient and family in how to properly carry out the care plan
b. Collaboration between the care team and patient/family regarding decision-making and clinical care
c. Patient/family direct the clinical care team’s actions
d. All members of the patient’s family must agree on a specific care plan before it is implemented
b

The pediatric nurse practitioner is evaluating a well-appearing 18-month-old patient with diarrhea for 6 weeks. The family reports that the patient develops progressively looser stools throughout the day. Dietary history reveals that the patient drinks 12 ounces cow’s milk and 24 ounces apple juice daily. Growth and development are within normal limits for age. Stool studies are negative for enteric pathogens and blood. The most likely diagnosis is:
a. Malabsorption secondary to cystic fibrosis
b. Infectious diarrhea
c. Chronic nonspecific diarrhea/toddler’s diarrhea
d. Milk protein allergy
c

A 13-year-old female patient is being evaluated by the pediatric nurse practitioner for allergic rhinitis. During the visit, the patient discloses that she is considering have sexual intercourse with her boyfriend. Later in the visit, she mentions that she has been feeling “depressed” recently. Which of the following should be the pediatric nurse practitioner’s first priority in caring for this patient?
a. Evaluating the patient further to determine the best contraceptive method for this patient
b. Eliciting more details about the depression, including evaluation for suicidal thoughts or plans
c. Managing the symptoms of allergic rhinitis at this visit and scheduling a follow-up visit to address the other issues
d. Referring the patient to an adolescent medicine specialist
b

Informed consent is characterized by:
a. The duty to provide all relevant information to a patient about their condition and alternative treatments
b. The duty to respect a patient’s individual values and choices
c. The duty to benefit patients and protect their interests
d. The duty not to inflict harm on patients
a

Which of the following is the best approach for conducting a physical examination of the 2-3 year-old child?
a. Proceed systematically from head-to-toe
b. Perform the most uncomfortable or traumatic parts of the examination first
c. Have the parent leave the room before conducting the examination
d. Allow the patient to remain with the parent and perform the uncomfortable/traumatic parts of the examination last
d

The pediatric nurse practitioner reads a healthy 10-year-old patient’s screening PPD (tuberculin skin test) as positive (12 mm in duration) 48 hours after the PPD was placed. The pediatric nurse practitioner’s next action should be to:
a. Begin multi-drug therapy for tuberculosis
b. Repeat PPD testing in 6 months
c. Begin monotherapy with isoniazid (INH)
d. Obtain a chest x-ray
d

Which of the following approaches to cultural differences between healthcare providers and patients/families is most effective?
a. Encourage patients and families to conform to the “majority culture” when interacting with the health care system
b. Urge parents to follow the advice of healthcare providers over that of extended family members
c. Contact the appropriate child protective agency if a family is non-compliant with medical care due to cultural differences
d. Utilize knowledge of the family’s cultural beliefs and practices in meeting the patient’s medical needs
d

A pediatric nurse practitioner evaluates an established patient for fever. The nurse practitioner documents the chief complaint (fever), history of present illness (duration, height of fever, associated symptoms, and timing), and a review of systems involving 3 organ systems. Physical examination is documented and includes full vital signs, general appearance, and head/neck/lung/cardiac/abdominal/skin examinations. Final diagnosis is fever with lower respiratory tract infection. The pediatric nurse practitioner should bill for which level of service?
a. Detailed
b. Expanded problem-focused
c. Problem-focused
d. Comprehensive
b

Which of the following vaccines is contraindicated in the immunocompromised pediatric patient?
a. Diphtheria, Tetanus, and acellular Pertussis (DTaP)
b. Inactivated Influenza
c. Measles, Mumps, Rubella (MMR)
d. Hepatitis B
c

The pediatric nurse practitioner notes two irregular areas of blue-gray discoloration over the sacral area of a 4-week-old Hispanic infant. The most likely etiology of this finding is:
a. Inflicted injury (abuse)
b. Acrocyanosis
c. Mongolian spots (congenital dermal melanocytosis)
d. Cutis marmorata
c

All of the following circumstances are permitted uses or disclosures of an individual’s “protected health information” under the Health Insurance Portability and Accountability Act (HIPAA) except:
a. Disclosure to the individual who is the subject of the information
b. Individuals who may have contracted or been exposed to a reportable communicable disease
c. Disclosure to an individual’s employer requesting the individual’s mental health diagnosis and treatment records
d. Disclosure in accordance with workers’ compensation statues with regard to a work-related illness or injury
c

The pediatric nurse practitioner is seeing a 9-year-old patient for routine health maintenance whose 40-year-old father recently suffered an acute myocardial infarction. Which of the following screening tests would be most appropriate for the pediatric nurse practitioner to order next?
a. Hypercoagulability screening panel
b. Fasting lipoprotein levels
c. Serum troponin levels
d. Brain natriuretic peptide (BNP) levels
b

A 6-year-old female patient has had fever, cough, and abdominal pain for 4 days. Physical examination reveals temperature 103 degrees Fahrenheit, tachypnea, and decreased breath sounds at the left lung base. Abdominal exam is non-tender. Which diagnostic test does the pediatric nurse practitioner order next?
a. Urinalysis
b. Chest x-ray
c. Complete blood count (CBC)
d. Rapid influenza virus study
b

Which of the following best defines the term health literacy?
a. An individual’s formal health education level
b. An individual’s ability to read written information about health education
c. An individual’s capacity to obtain and understand basic health information needed to make appropriate health decisions
d. An individual’s motivation to make good personal health decisions
c

The pediatric nurse practitioner is performing a well-child examination on a developmentally normal infant who recognizes his parents, can sit independently for 1-2 minutes, rolls over from supine or prone starting position, drops one object when another is offered, and babbles one-syllable sounds. This patient is approximately:
a. 3 months old
b. 9 months old
c. 6 months old
d. 12 months old
c

The pediatric nurse practitioner suspects autism spectrum disorder in a 2-year-old patient. The nurse practitioner should:
a. Refer the patient for further neurodevelopmental evaluation if symptoms persist at the 3-year-old health maintenance visit
b. Promptly refer the patient for further detailed neurodevelopmental evaluation
c. Recommend that the patient receive no further immunizations
d. Assist the family in locating a long-term placement facility
b

Which of the following statements is most accurate regarding nutrition in the school-age child?
a. Most school-age children eat more than the recommended amount of whole grains
b. Most school-age children need to increase their average sodium intake for improved nutritional status
c. Meals consumed at fast food restaurants are usually more healthful than meals prepared at home
d. Consumption of excess sugars, starches and fats in the average school-age child contributes to childhood obesity
d

The siblings of a child who develops a serious illness, injury, or disability:
a. Should be disciplined for verbalizing feelings of resentment and anger toward the affected sibling or parents
b. Often develop behavior and adjustment problems due to disruptions in the family routine when a sibling is seriously ill or injured
c. Should not be involved in the care and treatment of the affected sibling
d. Should be removed from their usual activities in order to spend more time with the rest of the family
b

Which of the following represents the best organizational approach when changing one’s practice based on clinical evidence?
a. Establish clinical question, develop and implement care plan, evaluate effectiveness of care plan, collect and evaluate relevant research
b. Collect and evaluate relevant research, establish clinical question, develop and implement care plan, evaluate effectiveness of care plan
c. Establish clinical question, collect and evaluate relevant research, develop and implement care plan, evaluate effectiveness of care plan
d. Establish clinical question, collect and evaluate relevant research, develop and implement care plan
c

The pediatric nurse practitioner is seeing a 13-year-old female patient with Tanner stage 4 breast and pubic hair development who has not had menarche. The pediatric nurse practitioner suspects:
a. Precocious puberty
b. Normal puberty
c. Androgen insensitivity
d. Ovarian tumor
b

Which of the following is true about the children of parents who smoke cigarettes?
a. Elevated risk for Sudden Infant Death Syndrome (SIDS) in infants of mothers who smoke
b. Decreased likelihood that the child will smoke cigarettes as an adult
c. Smoking cigarettes outside eliminates the risk that second-hand smoke poses to children
d. No significant difference in the risk of asthma and lower respiratory tract infections when compared to children of non-smoking parents
a

A 9-year-old patient has several circular scaly patches on the scalp with overlying hair loss, consistent with tinea capitis. The pediatric nurse practitioner prescribes:
a. Topical antifungal cream
b. Topical steroid cream
c. Permethrin 5% cream
d. Oral griseofulvin and selenium sulfide shampoo
d

At a routine health maintenance visit, an 11-year-old patient with Tourette Syndrome informs the nurse practitioner that he is struggling both socially and academically at school. His parents have spoken with his teacher at school, but the problems have continued. He confides that he often goes to the school nurse’s office with vague complaints to avoid some of his regular school day. The pediatric nurse practitioner’s next action should be to:
a. Refer the patient to a neurologist so that medications can be started for symptom management
b. Refer the patient for psychological counseling
c. Establish communication with the school nurse to enlist his/her assistance in advocating for the patient at school
d. Tell the patient to try harder to control his symptoms while at school
c

The mother of a 4-year-old female patient reports that the child frequently awakens screaming and agitated about 1 hour after falling asleep and does not appear to recognize her mother. The child falls asleep again within 10 minutes and does not recall these events in the morning. The pediatric nurse practitioner should:
a. Tell the mother to speak loudly to the child during these episodes to fully awaken her
b. Refer the family for counseling
c. Refer the patient to a pediatric neurologist
d. Reassure the mother that these episodes will likely resolve as the child gets older
d

The pediatric nurse practitioner is evaluating a one-month-old infant male with a unilateral undescended testicle (cryptorchidism). The patient was born at term after an uncomplicated pregnancy and physical examination reveals no other abnormalities. The pediatric nurse practitioner should:
a. Refer the patient to pediatric surgery urgently for orchiopexy
b. Refer the patient to pediatric surgery if testis remains undescended at 6 months of age
c. Inform the parents that orchiopexy is indicated only if they are concerned about cosmetic appearance of an undescended testicle
d. Refer the patient to pediatric surgery only if the testis remains undescended at 24 months of age
b

Which of the following is not a typical clinical feature of Kawasaki disease?
a. Hand and foot edema
b. Bilateral non-exudative conjunctivitis
c. Risk of coronary artery aneurysms
d. Bacterial growth in peripheral blood cultures
d

The pediatric nurse practitioner notices that the mother of her 6-year-old patient looks down at the floor when denying concern for domestic violence. During the rest of the conversation, the mother maintains good eye contact with the nurse practitioner. The pediatric nurse practitioner should:
a. Ask an open-ended question about domestic violence with the mother in private
b. Ask the 6 year-old patient about violence at home
c. Tell the mother she appears to be lying about domestic violence
d. Assume the mother was distracted by something on the floor while answering the question
a

A sexually-active, 17-year-old female patient is being evaluated for vaginal pain, discharge, abnormal odor and pain during intercourse. Pelvic examination reveals a frothy yellow vaginal discharge and vulvar erythema. Vaginal wet mount reveals several polymorphonuclear lymphocytes (PMNs) and motile flagellated organisms. For this particular condition, the pediatric nurse practitioner prescribes:
a. Topical clotrimazole
b. Oral fluconazole
c. Oral metronidazole
d. Oral doxycycline
c

Which of the following is most accurate regarding pediatric head injuries?
a. Larger head size relative to body size puts the pediatric patient at increased risk for head injury
b. Motor vehicle accidents are an uncommon cause of pediatric head injury
c. Female gender is a risk factor for pediatric head injury
d. Falls are an uncommon cause of head injury in young children
a

Which of the following techniques is most effective in communicating with the shy, 4-5-year-old patient?
a. Stand up when speaking with them so that they are looking up at you
b. Communicate through the child’s transition object (e.g., doll) before addressing them directly
c. Establish physical contact early
d. Speak only with the patient’s adult family members, rather than addressing the child directly
b

Which of the following statements is most accurate regarding inhaled corticosteroids in the pediatric patient with asthma?
a. Oral thrush is a common complication of inhaled corticosteroid use
b. Spacers are not usually necessary for the proper administration of inhaled corticosteroids with a metered dose inhaler
c. Daily inhaled corticosteroids are most effective in patients with mild intermittent asthma
d. Inhaled corticosteroids are indicated only for treatment of acute asthma exacerbations
a

In conducting educational health programs for patients with chronic illness, the most effective model involves:
a. Individualized education aimed at encouraging patients to take charge of their own health
b. Disseminating accurate information about self-care and treatment options without considering each patient’s individual strengths or challenges
c. Eliminating patients from the program who don’t change their behavior based on the health information provided
d. Single-session programs to provide the maximum amount of information in the shortest time possible
a

Which of the following characteristics is most concerning in the school-age patient with a cardiac murmur?
a. History of maternal grandmother who died of congestive heart failure at 62 years of age
b. History of tachycardia after inhaled albuterol administration
c. Corrected QT interval of 400 m/sec on 12-lead EKG
d. Progressive exercise intolerance
d

The pediatric nurse practitioner evaluates an injured pediatric patient and suspects that the patient’s injuries were caused by abuse. The pediatric nurse practitioner should:
a. Report the suspected abuse only if the patient’s family is acting suspiciously
b. Report the suspected abuse to the appropriate state agency
c. Report the suspected abuse to the appropriate state agency only if the physician agrees
d. Document his/her concerns in the patient chart without reporting the suspected abuse
d

Which of the following is most accurate about childhood lead exposure?
a. Houses built after 1978 are most likely to have lead-based paint
b. Children are less likely than adults to be affected by lead poisoning because their bodies absorb a lower percentage of lead when exposed
c. Most pediatric lead poisoning occurs via normal hand-to-mouth behavior in the presence of environmental lead dust
d. Children in minority ethnic groups have a lower incidence of lead poisoning than Caucasian children
c

Which of the following factors associated with poor health outcomes would be least attributable to lack of health insurance?
a. Difficulty accessing health care services due to geographic distance
b. Non-compliance with prescribed medications due to cost
c. Delay in seeking care for an acute condition due to financial hardship concerns
d. Less utilization of preventive health care services
a

Most stage-based theories of development focus primarily:
a. the continuity of development
b. the discontinuity of development
c. persistance of inherent personality characteristics
d. the influence of context on development
b

The common practice using “time outs” with young children is direct application of:
a. operant conditioning
b. classical conditioning
c. separation-individuation
d. maturational reinforcement
a

good communication between families, schools, and primary care providers is an example of which ecological concept?
a. microsystem
b. mesosystem
c. exosystem
d. macrosystem
b

Which of the following findings would be most likely associated with asymmetric intrauterine growth retardation?
a. weight, length, & head circumference ranging from 3rd to 5th percentile
b. heavy maternal smoking throughout pregnancy
c. Weight at 3rd percentile and length at 25th
d. Gestational diabetes
c

Early reflexive responses that are not r/t survival include all but:
a. babinski
b. moro
c. swimming
d. rooting
d

The most likely weight of 1yo child whose weight at birth was 6.5lb would be:
a. 19-20lbs
b. 13-14lbs
c. 25-26lbs
d. Impossible to estimate
a

One of the major psychosocial tasks of infancy is:
a. development of secure attachment
b. separation-individuation
c. symbiosis
d. regulation
a

Which developmental theory best explains the multifactorial etiology of failure to thrive?
a. organismic-maturational theory
b. social learning theory
c. transactional theory
d. psychoanalytic theory
c

Most healthy infants are able to reach, grasp, & hold onto a rattle or other small toy by about:
a. 2mo
b. 6mo
c. 8mo
d. 10mo
b

The pincer grasp is a fine motor skill that involves the ability to pick up a small object such as a raisin or piece of cereal with the thumb and forefinger that usually is mastered around:
a. 4mo
b. 6mo
c. 9mo
d. 16mo
c

You would be concerned about the language development of a child who:
a. repeats simple phrases at 32mo
b. stutters when excited or tired at 42mo
c. has a vocabulary of 10 words at 12mo
d. pronounces words that are understandable at 36mo
b

The most common temperamental profile is:
a. easy
b. difficult
c. slow-to-warm-up
d. intermediate
a

The underlying emotion of an insecurely attached (avoidant) relationship is:
a. ambivalence
b. deprivation
c. anger
d. conditional love
c

The stage of cognitive development that Piaget described as characteristic of the way preschoolers think is the:
a. preoperational stage
b. mental combinations stage
c. tertiary circular function stage
d. sensorimotor stage
a

A preschool boy whose parents have separated and are beginning divorce procedures:
a. may think that he caused the divorce by misbehaving
b. should not be told of the impending divorce until the parents are sure of their decision
c. is likely to experience gender identity confusion
d. should be able to make a decision about which parent he prefers living with
a

Which behavior would you expect to decrease during the preschool years?
a. rough-and-tumble play
b. instrumental aggression
c. hostile aggression
d. cooperative play
b

A preschool child who says that the sky is blue because it is his favorite color is illustrating the concept of:
a. symbolic thinking
b. egocentrism
c. centration
d. imaginary audience
b

Which of the following strategies would not be appropriate to include as part of your management of a 9yo boy who is obese?
a. referral to nutritionist for weight reduction plan
b. increase physical exercise
c. behavior modification strategies to deal with stress and/or reinforce treatment plan
d. involve family in management program
a

Which of the following issues or concepts is relevant to the school-aged child?
a. operational thinking
b. initiative
c. concrete operations
d. separation-individuation
c

The 1st physical sign indicating the onset of female puberty is:
a. sparsely distributed fine, pale pubic hairs
b. breast buds
c. menarche
d. peak height velocity
b

Which of the following findings would be helpful in distinguishing obesity vs. large body frame in an adolescent who is concerned with her weight?
a. tricep skinfold measurement
b. weight-for-height ratio
c. body mass index
d. percent of ideal body weight
c

the most common form of child abuse seen in pediatric primary care is:
a. burns
b. fractures
c. soft tissue injury
d. shaken baby syndrome
c

a differential diagnosis for child abuse would include all of the following except:
a. birth marks
b. unintentional injury
c. inadequate parenting
d. Prader-Willi syndrome
d

Which of the following symptoms are not typical of a child with ADHD?
a. easily distracted
b. difficulty playing quietly
c. doesn’t follow directions
d. frequently angry & resentful
d

Which of the following clinical findings would not suggest an eating disorder with purging component?
a. sore throat
b. brittle nails
c. constipation
d. finger calluses
b

Which of the following situations does not necessarily warrant immediate mental health assessment and/or referral?
a. a 13yo girl who has been “down” for the last mo with varied somatic complaints
b. a 9yo boy whose parents recently separated and filed for a divorce and seems to be doing well
c. a 16yo girl who has a hx of long standing depression but seems to be doing well in school
d. a 15yo boy who expresses suicidal thoughts
b

Which adolescent would be at greatest risk for developing anorexia nervosa?
a. 12yo female who just had her first period
b. 14yo gymnast
c. 16yo male runner
d. 18yo female college student
b

Which of the following substances is associated with pupillary constriction?
a. amphetamines
b. LSD
c. Heroin
d. nicotine
c

A risk factor that is common to many psychosocial pediatric problems including failure to thrive, conduct or oppositional disorders, and childhood depression is:
a. maternal depression or other psychiatric disorder
b. substance abuse
c. prematurity
d. history of sexual abuse
a

Which of the following diagnoses is not more common among males?
a. ADHD
b. conduct disorders
c. suicide
d. FTT
d

The diagnostic criteria for autistic disorders includes which of the following?
a. speech delay, ataxia, mental retardation
b. impairments in social interactions, interpersonal communication, and staring spells
c. mental retardation, impairments in social interactions, & stereotypical restricted pattern of interests & activities
c. impairments in social interactions, in interpersonal communication, and stereotypical restricted pattern of interest and activities
d

In addition to specific academic skill deficits, learning disabilities are commonly associated with which of the following characteristics?
a. perceptual-motor impairments, normal motor function
b. perceptual-motor impairments, impulsiveness
c. perceptual-motor impairments, down syndrome
d. perceptual-motor impairments, lack of impulsiveness
b

Preventative health guidelines include references to:
a. immunizations, health screening, disease prophylaxis, education, and infection control
b. immunizations, counseling, health screening, disease prophylaxis, and education
c. health screening, disease prophylaxis, counseling, and CPR
d. health screening, disease prophylaxis, education, immunizations, and CPR
b

The nurse practitioner role was initially established to:
a. improve access to care and partially solve physician shortage
b. reduce the nursing shortage and improve access to care
c. improve working conditions of nurse while improving access to care
d. improving nursing’s image through expansion of the role
a

early nursing research focused on:
a. the response of policy makers to the nursing shortage
b. the effectiveness of the NP as a primary care giver
c. an effort to demonstrate quality and cost effectiveness of NP
d. the role of the NP as physician extender
c

Which of the following is not a major factor influencing healthcare delivery services?
a. provider
b. payers
c. insurers
d. agencies
d

All definitions of primary health care include:
a. the concept of universal access and accountability
b. the concept of universal access and AIDS prevention
c. the concept of universal access and focus on self-responsibility for health
d. the concept of universal access and a focus on reimbursement for services rendered
a

standards of practice are:
a. authoritative statements used to measure quality
b. used to measure outcome but are not authoritative
c. designed for legal purposes
d. not designed for legal purposes and cannot be used to measure quality
a

Quality improvement activities include:
a. patient satisfaction surveys only
b. peer review, patient satisfaction surveys, chart audits
c. defining four practice domains
d. systems to decrease risk of injury to patients
b

Most risk management programs are based on the assumption that:
a. many injuries to patients are preventable
b. most legal liability is a result of poor documentation
c. most injuries to patients are not preventable
d. malpractice insurance is generally unnecessary
a

If an APRN practices beyond his/her scope:
a. malpractice insurance will protect him/her from a charge of practicing medicine without a license
b. malpractice insurance will not protect him/her from a charge of practicing without a license
c. he or she is legally accountable to the certifying body
d. The collaborating physician is legally accountable to the certifying body
b

Standards of practice may be used to:
a. establish minimal levels of performance
b. establish reimbursement schemes for APRN
c. Mandate nursing practice across the nation
d. Mandate nursing practice in certain states
a

Scope of practice:
a. is identical across the states
b. is determined by the federal government
c. is mandated by the federal government
d. varies from state to state
d

Medicaid provides health insurance coverage to:
a. certain categories of people whose personal income falls below the federal poverty level
b. anyone whose personal income falls below the federal poverty level
c. newborns, pregnant women, and those over 65 whose personal income falls below the federal poverty level
d. those who are elderly
a

Medicaid reimbursement is available to an APRN:
a. practicing in federally designated areas
b. at a rate that is between 70-100% of the physician rate
c. only if the APRN is in collaborative practice with a physician
d. practicing in nursing homes only
b

Medicare reimbursement for services:
a. is not dependent on the patient’s income level
b. depends on the patient’s income level
c. is not available to APRN under any circumstances
d. is only available to APRN who is in collaboration practice with a physician
a

Medicare Part A covers:
a. Hospital, skilled nursing facility, and hospice care
b. all medically necessary services
c. skilled nursing facility care only
d. hospice care only
a

Medicare Part B covers:
a. all medically necessary services
b. inpatient hospital care
c. outpatient physician services only
d. skilled nursing facility and hospice care
a

To receive Medicare reimbursement, APRN must:
a. be nationally certified and maintain prescriptive privileges
b. maintain current license in the state in which they are practicing
c. practice in a designated medically underserved area
d. practice with a physician
b

The knowledge base of the APRN is based on:
a. medical content
b. theoretical content only
c. scientific content and theory
d. theory and research
c

The role of the APRN has traditionally focused on:
a. the delivery of primary health care to all people
b. the delivery of acute health care to all people
c. chronic care
d. the medical model
a

Legal authority for APRN practice is granted by:
a. federal law
b. regulations from the department of health and human services
c. state law and regulations
d. the board of medicine in most states
c

malpractice insurance:
a. protects an APRN from charges of practicing medicine without a license when they are practicing outside the legal scope of practice
b. does not protect an APRN from charges of practicing medicine without a license when they are practicing outside the legal scope of practice
c. does not pay for legal defense if the APRN is practicing beyond the legal scope of practice
d. is important, but should not be purchase if the facility in which the APRN is employed carries good coverage
b

Current prescriptive authority for APRN:
a. varies among states
b. is fairly consistent among the states
c. provides DEA numbers for APRN
d. allows APRN to move freely from state to state
a

Managed care is a term that describes:
a. an established system of healthcare delivery that is mandated by the federal government
b. a network of providers who contract to provide services for a specific group of enrollees
c. a system that does not recognize APRN as a primary provider
d. a network of hospitals and nursing homes that provide care to chronically ill people
b

Certification is:
a. procedure through which the government appraises and grants certification to the APRN
b. Granted by the individual states
c. Governed by each state’s Board of Nursing
d. A process in which a nongovernmental agency or group verifies that an APRN has met certain predetermined standards for specialty practice.
d

Licensure is:
a. federal process that is used to standardize healthcare facilities
b. is granted by a state government agency and grants permission to engage in the practice of a given profession
c. Cannot be used to prohibit anyone from practicing a given profession
d. is a federal process that is used to standardize educational programs
b

Reimbursement under managed care:
a. requires that the provider accept the financial risk for the care provided to a specific population of enrolled patients
b. requires that the managed care organization accept the financial risk for the care provided to a specific population of enrolled patients.
c. does not reward efficient care delivery
d. is not available to APRN
a

An integrated delivery system:
a. one that delivers high quality care but is often not cost effective
b. delivers a vertical integration of services with capitated payment
c. does not include rationing of resources
d. does not include a capitated payment scheme
b

The APRN Consensus Model is:
a. mandate from the NCSBN that defines advanced practice nursing
b. a proposed regulatory model for advanced practice nursing
c. a proposal for federal legislation for advanced practice nursing
d. approved only by the american nurses association
b

The term “incident to” refers to:
a. the occasions when an APRN practices independently but occasionally consults with a physician
b. the notion that the physician must be present in the office suite and immediately available to provide assistance in order for the APRN to bill for services rendered
c. the notion that a physician must examine the patient along with the APRN if Medicare is to be billed for services rendered
d. Medicaid only and is not pertinent to Medicare billing
b

“incident to” billing is specific to:
a. Medicare
b. Medicaid
c. Medicare & Medicaid
d. Private insurance companies
a

Which of the following is the most accurate description of the Dubowitz tool?
a. it evaluates the physical condition of the newborn at birth.
b. the criteria evaluated are influenced by labor & birth; therefore a second exam may be indicated to pick up any changes in the newborn
c. It is a gestational assessment tool that evaluates the infant’s newborn reflexes including moro reflex, tonic neck, grasping, rooting, and sucking
d. Estimations of gestational age are performed by examining physical characteristics and neuromuscular development of the newborn.
d

A full term newborn had a birth weight below the 10th percentile, with length and height in the 50th percentile. The maternal hx was + for anemia, preeclampsia, and 2 prior miscarriages. What is the most likely diagnosis?
a. Intrauterine growth retardation (IUGR) secondary to chronic HTN
b. anemia secondary to maternal deficiency
c. small for gestational age secondary to chromosomal abnormalities
d. Asymmetric IUGR
d

Which of the following is the most accurate statement regarding newborn screening?
a. sickle cell disease & cystic fibrosis screening tests are required in all 50 states.
b. screening is performed for disorders in which there are no cure
c. screening is performed using a simple screening method
d. Phenylketonuria (PKU), glactesemia, hypothyroidism screening tests are performed in all symptomatic newborns
c

A parent complains that their child awakens crying and agitated during the last half of night sleep. The child is awake and can be comforted. The parent denies this child is experiencing
a. sleep terrors
b. nightmares
c. somniloquy
d. sleep starts
b

The parents of a 2 year old are concerned that their child is having temper tantrums in public settings. What is the BEST option?
a. The temper tantrum is an indication that the child is tired and needs to go home.
b. Make sure to ensure safety while ignoring the child’s display of behavior.
c. You can pick up the child and take him to a quiet place for a time out.
d. In a public place, it is okay to give into the child’s desires to maintain peace.
b

The most common hematological presentation of leukemia in children is anemia along with which of the following?
a. neutropenia and thrombocytopenia
b. leukocytosis and monocytosis
c. neutropenia and blast cells on peripheral smear
d. monocytosis and blast cells on peripheral smear
a

Recommendations regarding the current management of croup with increased work of breathing include the use of
a. nebulized bronchodilators
b. oral corticosteroids
c. oral antibiotics
d. cool mist therapy
b

An 11 year old presents with a 3 day history of cough at night, tachypnea, and upper respiratory infection (URI) symptoms. Exam reveals loud inspiratory and expiratory wheezing. What should be the FIRST COURSE of action?
a. administer inhaled albuterol
b. oxygen by mask
c. obtain a chest x-ray
d. administer oral steroids
a

The most appropriate INITIAL management of a toddler with persistent post-infectious diarrhea is to
a. continue with a regular diet
b. start a high fat, low carbohydrate diet
c. switch to a lactose-free diet
d. start a clear liquid diet
c

A mildly ill 4 year old presents with exudative pharyngitis and erythematous nonblanching lesions on the legs. Rapid strep test is positive What is the MOST likely diagnosis?
a. scarlet fever
b. Henoch-Schonlein pupura (HSP)
c. systemic lupus erythematosus
d. immune thrombocytopenia purpura
b

Which clinical or diagnostic finding is the most significant for pertussis in an adolescent patient?
a. hyperinflation on chest x-ray
b. persistent night-time cough
c. lymphocytosis on CBC
d. productive cough
c

Which counseling for parents of a 2 year old with an elevated lead level, would be effective in decreasing lead exposure?
a. avoid kerosene heaters
b. dry dusting
c. sweeping
d. high efficiency particulate air (HEPA) filters for vacuuming
d

The purpose of administering acidophilus while taking antibiotics is to
a. decrease pH in the gastrointestinal tract
b. enhance the absorption of the antibiotic in the colon
c. treat diarrheal illnesses caused by Clostridium difficile
d. decrease diarrhea by adding additional fiber
a

Which of the following should be included in the plan of care for preterm infants?
a. use of a car seat without a shield harness
b. supplemental vitamin D at 4 months for breast fed infants
c. immunizations beginning at 2 months corrected age
d. evaluation of vision at 3 months
a

Which represents information obtained during a review of systems (ROS)?
a. mother of the child has asthma
b. height and weight
c. history of birthmarks or skin lesions
d. number of previous hospitalizations
c

A teenager sustains a puncture wound in the sole of a foot after stepping on a protruding nail. The most appropriate INITIAL management of this injury would be
a. debridement of epidermal edges
b. wound probing for retained material
c. initiation of prophylactic antibiotics
d. superficial irrigation of the wound
d

A 15 year old previously healthy teen is having school problems, feelings of sadness, insomnia, and disinterest in activities previously enjoyed. The parents reported the teen spends hours alone, avoiding social situations. When questioned, the teen denies any desire or plan to hurt himself or others. Which would be the MOST appropriate INITIAL management of this patient?
a. prescribe a tricyclic antidepressant
b. arrange for home schooling
c. refer for immediate inpatient evaluation
d. refer for outpatient psychotherapy
d

Nurse practitioners can best assist teens in reducing behaviors that present health risks while socializing with peers by
a. maintaining awareness of local social events for teens
b. incorporating conversations with teens regarding safe sex, substance use, and body art with regular health visits
c. discussing safe sex, substance use, and opportunities for body piercing and tattoos with teens in the presence of their parents
d. discouraging teens to participate in group activities without adult supervision
b

For the parents of an 8 month old, the MOST appropriate age-related anticipatory guidance related to safety should include which of the following?
a. cover electrical outlets, keep small objects out of reach, turn car seat to forward-facing position
b. keep medications out of reach, continue to use car seat in rear-facing position, cover electrical outlets
c. avoid finger foods that can be easily aspirated, teach stranger safety, place Mr. Yuk stickers on toxic materials
d. keep matches out of reach, place Mr. Yuk stickers on toxic materials, teach stranger safety
b

The most important PRIMARY care goal for a school aged child with a prosthetic heart valve is to
a. prohibit exercise and sports participation
b. maintain endocarditis prophylaxis
c. provide home schooling
d. maximize carbohydrate calories for growth
b

A term infant born by C-section, with birthweight of 5 lbs 5 oz (2.4 kg), develops tachypnea with a respiratory rate of 68 at 2 hours of life. The most likely diagnosis is
a. transient tachypnea of the newborn
b. pneumothorax incurred during delivery
c. respiratory distress syndrome
d. congenitally acquired pneumonia
a

16yo Sarah makes the following statements to you during a health visit. Which of the following pieces of information should not be kept confidential?
a. “I have been sexually active with 3 of my boyfriends”
b. I sometimes smoke marijuana
c. I want to get pregnant
d. Sometimes I feel like ending my life
d

In performing a physical examination on a 9mo old infant, which of the following developmental fears woud not be appropriate for you to consider?
a. Stranger anxiety
b. Pain
c. Separation from parents
d. Bodily harm
d

When performing a physical examination on a toddler, which of the following body parts would you examine last?
a. heart & lungs
b. abdomen & genitals
c. ears & throat
d. hips & extremities
c

Role play with equipment during the course of a physical exam would be most beneficial with with of the following age groups?
a. toddlers
b. preschoolers
c. young school aged children
d. older school aged children
b

Providing reassurance of “normalcy” during the course of an exam would be most important for:
a. preschool children
b. young school aged children
c. older school aged children
d. adolescents
d

Which of the following would not elevate the pulse of a child?
a. fever
b. anemia
c. hypothyroidism
d. exercise
c

The PNP recognizes which of the following signs as indicators that baby is not receiving sufficient breastmilk?
a. sleepiness, jaundice, decreased urine & stool
b. diarrhea, nausea, & vomiting
c. bulging, nausea, & vomiting
d. sleeplessness & excitability
a

Blood pressure should be measured at well child visits, beginning at age:
a. 2yo
b. 3yo
c. 4yo
d. 5yo
b

A wide pulse pressure that results from high systolic blood pressure is usually not due to which of the following?
a. fever
b. exercise
c. excitement
d. a patent ductus arteriosus
d

Head & chest circumferences should be equal at:
a. 6mo
b. 1yo
c. 2yo
d. 3yo
b

The anterior fontanel usually closes by:
a. 2mo
b. 6mo
c. 18mo
d. 24mo
c

Diffuse edema of the soft tissue of the scalp which usually crosses suture lines in the newborn is:
a. bossing
b. caput succedaneum
c. cephalohematoma
d. macrocephaly
b

An infant should no longer have a head lag when pulled from the supine to sitting position at what age?
a. 2mo
b. 4mo
c. 6mo
d. 9mo
c

“boggy” nasal mucous membranes with serous drainage upon examination usually suggests:
a. sinusitis
b. polyps
c. URI
d. allergic rhinitis
d

A white instead of red reflex upon eye exam of a 1yo child would suggest:
a. an accommodation error
b. retinoblastoma
c. papilledema
d. retinal detachment
b

A cobblestone appearance of the palpebral conjunctiva usually indicates?
a. bacterial infection
b. chemical irritation
c. viral infection
d. severe allergy
d

An eye that deviates in when covered but returns to midline when uncovered is an:
a. esophoria
b. exophoria
c. esotropia
d. exotropia
a

Pain produced by manipulation of the auricle or pressure on the tragus suggests:
a. acute otitis media
b. otitis externa
c. otitis media with effusion
d. mastoiditis
b

a hypernasal voice and snoring in a child is suggestive of:
a. polyps of the larynx
b. nasopharyngeal tumor
c. hypertrophied adenoids
d. cleft palate
c

Physiological splitting of the second heart sound during inspiration in a child:
a. is normal
b. should be evaluated with an EKG
c. suggests an ASD
d. should be referred to a cardiologist
a

Which of the following is not a characteristic of innocent heart murmurs in children?
a. systolic in timing
b. varies in loudness with positioning
c. usually transmitted to the neck
d. usually loudest at left sternal border or at 2nd or 3rd intercostal space
c

A grade II musical or vibratory murmur heard best at the left sternal border that changes with positioning is suggestive of a:
a. pulmonary ejection murmur
b. ventricular septal defect
c. venous hum
d. vibratory or Still’s murmur
d

Wheezing in a child may not be found in which of the following conditions:
a. asthma
b. bronchiolitis
c. pleural friction rub
d. cystic fibrosis
c

Gynecomastia in a male may not be a finding in which of the following?
a. normal pubertal development
b. steroid usage
c. hyperthyroidism
d. testicular tumor
c

Which of the following would usually not be considered a sign of a pituitary tumor in a an adolescent female?
a. dysfunctional uterine bleeding
b. galactorrhea
c. loss of peripheral vision
d. increase in headaches
a

Which of the following is not a specific exam test for a dislocated hip?
a. barlow’s test
b. ortolani’s test
c. trendelenburg test
d. gower’s test
d

In addition to the knee, which of the following should be examined in a child complaining of knee pain?
a. foot
b. ankle
c. hip
d. spine
c

Which of the following infant reflexes should not disappear by 6mo of age?
a. moro
b. rooting
c. tonic neck
d. plantar grasp
d

Spasticity in an infant may be an early sign of:
a. neurofibromatotis
b. hydrocephalus
c. cerebral palsy
d. muscular dystrophy
c

A shift to the left is present when which of the following are elevated?
a. neutorphils
b. bands or stabs
c. lymphocytes
d. eosinophils
b

Which of the following is usually elevated with viral infections?
a. neutrophils
b. eosinophils
c. lymphocytes
d. basophils
c

Decreased platelets may not be found in which of the following?
a. leukemia
b. anemia
c. ITP
d. medication usage (ampicillin, cephalothin)
b

Which of the following does not suggest a UTI?
a. increased protein
b. increased WBCs
c. increased RBCs
d. increased nitrites
a

A mantoux test in a child with no risk factors is considered positive with a reaction of:
a. at least 5mm induration
b. at least 8mm induration
c. at least 10mm induration
d. at least 15mm induration
d

The PNP teaches new parents that when breastfeeding is well established they can expect baby to have:
a. as many as 4 wet diapers each day
b. a stool every 3-4hrs
c. 1 wet diaper an hour
d. 5-6 wet diapers and 2-3 stools each day
d

Cholesterol screening should be done:
a. children 2yrs+ who have a parent with a total cholesterol level of 240mg/dL or greater
b. once all children at 6yrs of age
c. overweight children with a family history of premature cardiovascular disease
d. once for all children at 12yrs of age
a

For which of the following screening tests should children fast for 12hrs before the test is done?
a. total cholesterol
b. serum chemistry profile
c. serum lipid profile
d. hematocrit
c

Which of the following is the most important history taking question for sports evaluation?
a. has the child ever had head injury?
b. has the child ever fainted or lost consciousness during exercise?
c. does the child ever get short of breast with exercise?
d. has the child ever had prior surgeries?
b

Which of the following conditions would exclude a child from participating in contact collision sports?
a. fever
b. absence of a paired organ
c. atlantoaxial instability
d. prior head injury
a

Which of the following topics would not be appropriate to include when providing anticipatory guidance to the parent of a 4mo old infant?
a. introduction of solid foods
b. teething
c. negativism
d. introduction of a cup
c

Which is the correct order for introduction of solid foods to an infant?
a. fruits, cereal, vegetables, & meats
b. cereal, meats, vegetables, fruits
c. fruits, cereal, meats, and vegetables
d. cereal, vegetables, fruits, meats
d

Which of the following topics is not appropriate to include when providing anticipatory guidance to the parent of an 18mo old?
a. temper tantrums
b. toilet training
c. dental care
d. stranger anxiety
d

Appropriate anticipatory guidance for the parents of an 8yo girl should not include:
a. preparation for an increase in nervous mannerisms & restless activity
b. preparation for pubertal changes
c. information that friends begin to serve as allies against adults
d. information that their daughter will take idols & heros
a

A new mom calls the PNP on post partum day 5. She reports her newborn wants to nurse for 30min every 2hrs. Which of the following is your best responses?
a. This is a very healthy breastfeeding pattern. Be sure to rest when you can. You are doing a great job.
b. Your baby is too demanding. If you can continue to feed that often you will spoil your child.
c. You are not making enough milk and your baby will need to go to the ER to be evaluated.
d. Your baby has an oral fixation and you should offer a pacifier to relieve stress
a

A pelvic exam should not be performed on which of the following adolescents?
a. a 14yo who is sexually active
b. a 15yo who has just started menarche
c. a 17yo who is having irregular menses
d. an 18yo healthy female
b

Which of the following may cause microcephaly?
a. hypocalcemia
b. craniosynostonosis
c. skull fracture
d. seizure disorder
b

What finding may accompany macrocephaly?
a. pulsating anterior fontanel
b. sunken fontanel
c. premature closure of suture lines
d. widened suture lines
d

Obtaining a CT of the head would be indicated in which of the conditions?
a. macrocephaly
b. cephalohematoma
c. craniosynostosis
d. caput succedaneum
a

Which one of the following conditions increases the risk of developing hydrocephalus?
a. bilateral cephalohematomas
b. craniosynostosis
c. prematurity
d. familial macrocephaly
c

A conjunctivitis appearing in a 2 day old newborn is likely due to:
a. chemical irritation from eye drops
b. group B streptococcus
c. chlamydia
d. gonorrhea
a

Confirming the diagnosis of chlamydia conjunctivitis in a newborn would best be done by obtaining which one of the following?
a. cervical swab of the mother
b. urine PCR from the mother
c. culture of the eye discharge
d. culture of the conjunctival scrapings
d

Which one of the following eye findings would be considered on ophthalmic emergency?
a. unilateral vesicular lesions on the upper eyelid in a 3 week old
b. presence of chemosis in a 5 yo with bilateral upper eyelid edema
c. cobblestone-like appearances along the inner aspect of the upper eyelid in a 15yo
d. bilateral redness along eyelid margins with tiny ulcerated areas in a 16yo
a

The most appropriate management of a 5yo with a firm, nontender nodule in the mid-upper eyelid for 3 weeks would be:
a. cool compress
b. topical ophthalmic ointment
c. oral antibiotics
d. oral steriods
b

Daily eyelid cleansing with diluted baby shampoo and a cotton-tipped applicator would be appropriate in the treatment of which one of the following conditions?
a. dacryostenosis
b. chalzion
c. hordeolum
d. blepharitis
d

A 3yo has an edematous, mildly erythematous right upper eyelid for one day with a fever of 103F. An important eye assessment would be:
a. ocular mobility
b. conjunctival inflammation
c. pupillary reaction
d. optic disc papilledema
a

Concurrent otitis media & conjunctivitis is likely due to which organism?
a. streptococcus pneumoniae
b. haemophilus influenza
c. moraxella catarrhalis
d. staphylococcus aureus
b

All but which one of the following is consistent with glaucoma?
a. photophobia
b. epiphoria (increase tears)
c. blepharospasm
d. leukocoria (white red reflex)
d

All but which one of the following assessments is used to determine the presence of an strabismus?
a. Hirschberg test
b. Cover-uncover test
c. Extraocular movements
d. Pupillary response
d

A 3mo old has a mild asymmetrical corneal light reflex on physical exam. What is the next appropriate step?
a. Observe & reevaluate at the next well check
b. Refer immediately to ophthalmology
c. Begin atropine drops or eye patching
d. Protect eyes from sunlight
a

Prematurity increases the risk of developing which one of the following?
a. nystagmus
b. astigmatism
c. myopia
d. glaucoma
c

Fluorescein screening of the eye is used to detect a:
a. keratitis
b. foreign body
c. corneal abrasion
d. hyphema
c

Trauma to the eye increases the risk of developing all but which one of the following?
a. strabismus
b. glaucoma
c. cataracts
d. hyphema
a

Corneal abrasions can be managed with topical application of which one of the following?
a. anesthetic for pain control
b. steroids to prevent adhesions
c. antibiotics to prevent infection
d. atropine to prevent ciliary spasm
c

The greatest risk in a patient with a hyphema is which one of the following?
a. glaucoma
b. infection
c. rebleed
d. cataracts
c

A 16yo was hit in the eye 1 day ago and now has ecchymoses on the upper & lower lids with 5/10 eye pain. All but which one of the following would be appropriate to obtain at this time?
a. visual acuity
b. intraocular pressure
c. CT scan
d. fluorescein stein
b

A 10yo has marked ear pain, not wanting anyone to touch his ear. The canal is edematous & exudate is present. TM is normal. How would this be managed?
a. Topical fluoroquinolone
b. Oral steroids & topical neomycin
c. oral amoxicillin & topical anesthetic
d. oral amoxicillin & topical steroid
a

Patients with otitis externa should be instructed to do which one of the following?
a. keep ear dry until symptoms improve
b. limit swimming for remainder of summer
c. wear ear plugs at all times with swimming
d. use alcohol drops before swimming each day
a

All but which one of the following patients are at an increased risk of developing otitis media?
a. a 2yo with cleft palate repair at 1yr of age
b. 15mo old with Down syndrome
c. 9mo old with lactose intolerance
d. 3yo with IgA immune deficiency
c

A 15mo old failed treatment with amoxicillin for an otitis media. At his 2 week recheck his TM remained red with distorted landmarks and he persisted with nasal congestion, poor nighttime sleeping, & with a 101F fever for past 2 days. the next step would be to treat with:
a. 10 day course of Augmentin
b. 3 week course of cephalosporin
c. a higher dose of amoxicillin & topical antibiotics
d. Ceftriaxone & an antihistamine
a

A 2yo male with hx of chronic serous otitis media is noted to have pearly white opacity in the upper outer quadrant of his TM. He currently has no symptoms & appears to hear “okay”. The most likely diagnosis & appropriate management would be:
a. tympanosclerosis; no treatment necessary
b. persisten perforation; prescribe topical antibiotic drops
c. foreign body; perform an ear wash for removal
d. cholesteatoma; refer to otolaryngology
d

A 7yo has experienced recurrent nosebleeds in the past 2mo. What finding on the physical exam would suggest an underlying medical cause for epistaxis?
a. wheezing
b. grade II murmur
c. Petechiae
d. Tonsil hypertrophy
c

An 8yo has chronic intermittent nasal congestion. All but which one of the following would support allergic rhinitis?
a. pale boggy turbinates
b. darkened areas on lower eyelids
c. increased basophils on CBC
d. itchy, watery eyes
c

Acceptable management options for allergic rhinitis include all of the following except:
a. oral cetirizine
b. oral montelukast
c. nasal beclomethasone
d. nasal neosynephrine
d

Which foreign body in the nose needs immediate removal?
a. bean
b. bead
c. stone
d. battery
d

What complication of sinusitis are adolescent males more prone to?
a. intracranial abscess
b. potts puffy tumor
c. orbital cellulitis
d. dental infection
a

Patients with sinusitis should be instructed not to participate in what activity?
a. Swimming/diving
b. boxing/wrestling
c. weight lifting
d. cross country running
a

All of the following may predispose a patient to thrush except:
a. age
b. steroid therapy
c. antibiotics
d. poor oral hygiene
d

A 9mo old is noted to have a bifed uvula. This would increase his risk of developing which disorder?
a. otitis media
b. retropharyngeal abscess
c. sinusitis
d. dental malocclusion
a

A 10yo has a single painful ulcerated lesion on an erythematous base on the inner buccal mucosa. The most likely diagnosis & treatment would be:
a. herpes simplex stomatitis-oral acyclovir
b. herpangina-viscous xylocaine
c. apthous ulcer-triamcinalone in Orabase
d. Hand, food, mouth syndrome-antibiotic mouth wash
c

Initial exposure to the herpes virus may produce all of the following except:
a. fever & dehydration
b. submandibular lymph nodes
c. vesicular lesions on tonsils
d. friable & edematous gingivia
c

The organism that causes hand, foot, and mouth syndrome is what virus?
a. cytomegalovirus
b. parainfluenza
c. varicella-zoster
d. coxsackie
d

Which one of the following complications of strep pharyngitis cannot be prevented with antibiotics?
a. peritonsillar abcess
b. cervical adentitis
c. glomerulonephritis
d. acute rheumatic fever
c

In addition to penicillin, all of the following antibiotics can be used to treat strep pharyngitis except:
a. clindamycin
b. erythromycin
c. bactrim
d. ceftriaxone
c

Retropharyngeal abscess is typically seen in what age group & mainstay treatment includes:
a. neonates; hospitalization for IV antibiotics
b. 2-6yo; ICU admission and IV antibiotics
c. 6-12yo; outpatient oral antibiotics
d. adolescent: ENT drainage of abscess
b

Findings consistent of peritonsillar abscess include all of the following except:
a. muffled voice
b. unilateral enlargement of tonsil
c. trismus
d. exudate on tonsils
d

Appropriate lab tests to obtain in assessment of cervical adenitis include all of the following except:
a. throat culture
b. mono test
c. PPD test
d. blood culture
d

The incidence of epiglottis has decreased because of which vaccine?
a. Hib
b. Prevnar
c. Varicella
d. Meningococcal
a

Patients with epiglottis prefer to sit in what position?
a. sitting up and leaning forward
b. left lateral position
c. supine with neck hyperextended
d. 45 degree upright, resting back
a

All but which one of the following conditions requires urgent inpatient admission?
a. cervical adentitis
b. retropharyngeal abscess
c. epiglottis
d. orbital cellulitis
a

Conductive hearing loss can be caused by:
a. brain tumor
b. ototoxic drug exposure
c. loud noises
d. serous otitis
d

The most common congenital heart defect in children is:
a. tricuspid atresia
b. ventricular septal defect
c. aortic stenosis
d. pulmonary atresia
b

The mother of a 4mo old infant reports that he turned “blue” and seemed to have fast, labored breathing after vigorous crying soon after awakening. He “fell asleep” and his color and breathing seemed to improve. On physical exam, the mucous membranes of the lips & mouth apear mildy cyanotic. A systolic murmur is heard best at the left sternal border. Vital signs are normal with normal peripheral pulses. There is no hepatomegaly. A likely diagnosis is:
a. congestive heart failure
b. apnea
c. coarctation of the aorta
d. cyanotic spell related to tetralogy of fallot
d

Management of the infant with suspected heart disease and reported cyanotic spell should include:
a. prompt referral to a cardiologist
b. an apnea monitor
c. instructing the parent to keep a diary of these episodes
d. continuous administration of oxygen
a

chest pain in young children is usually:
a. a symptom of congenital heart disease
b. noncardiac in origin
c. a sign of hypercholesterolemia
d. a symptom of congestive heart failure
b

a common cause of congestive heart failure in the first year of life is:
a. pulmonary stenosis
b. ventricular septal defect
c. rheumatic fever
d. complete heart block
b

The least likely physical finding in a 2mo old with congestive heart failure is:
a. tachypnea
b. tachycardia
c. hepatomegaly
d. pedal edema
d

A vibratory systolic murmur is heard between the lower left sternal & the apex in a healthy 4yo at her preschool physical. The cardio exam is otherwise normal. A likely diagnosis is:
a. venous hum
b. still’s murmur
c. transposition of the great arteries
d. rheumatic heart disease
b

Characteristics of a venous hum include:
a. a systolic murmur
b. radiation over precordium
c. marked decrease or disappearance of murmur when child is supine
d. heard best at lower left sternal border
c

which of the following is true regarding innocent murmurs?
a. the murmur is often holosystolic
b. prompt referral to a cardiologist is indicated
c. a precordial thrill is present
d. the murmur is low intensity, grade 1-3
d

SBE prophylaxis is recommended for:
a. all children with congenital heart disease on a daily basis
b. all children with congenital heart disease before dental, GI, and GU procedures
c. children with repaired congenital heart disease with residual defect at the repair site
d. 5 years after repair of all congenital heart disease
c

A 12yo girl seen at a routine visit has a blood pressure of 140/90. She denies any symptoms. The initial management would include:
a. intravenous pyelogram
b. return for two repeat blood pressure measurements
c. no follow up needed-blood pressure probably related to anxiety
d. diuretic therapy
b

A 9yo boy presents with a fever of 102F and complaints of leg pains. His mother reports that he had upper respiratory infection with a sore throat approximately two weeks ago, which subsided without therapy. On physical exam, he has tender, swollen knees bilaterally. His heart rate is 120/min and blowing systolic murmur is heard at apex. No murmur was noted on previous well-child visit. The most likely diagnosis is:
a. Kawasaki disease
b. Rheumatic fever
c. Sickle cell anemia
d. Viral illness
b

The most useful test for evaluation of suspected acute rheumatic fever is:
a. antistreptolysin-O titer
b. electrocardiogram
c. hemoglobin electrophoresis
d. urinalysis
a

the initial attack of acute rheumatic fever is preceded by:
a. a viral illness
b. a group A streptococcal infection
c. exposure to mites
d. exposure to chicken pox
b

A 3 week old infant has one day history of irritability, pallor, and poor feeding. He is afebrile. On physical exam, his heart rate is 240/min while asleep. The most likely diagnosis is:
a. supraventricular tachycardia
b. premature ventricular contractions
c. sinus tachycardia
d. cyanotic heart defect
a

Congenital complete heart block may be associated with?
a. maternal lupus erythematosus
b. Wolff-Parkinson-White syndrome
c. maternal myocardial infarction
d. Kawasaki disease
a

The most common cause of myocarditis is:
a. bacterial
b. viral
c. drug reaction
d. radiation therapy
b

Which of the following is not an expected finding in a child with myocarditis?
a. persistent tachycardia
b. history of antecedent “flu-like” illness
c. a gallop rhythm
d. a significant heart murmur
d

Hypercholesterolemia in children over 2yo is defined as total cholesterol at or above:
a. 100mg/dL
b. 130mg/dL
c. 160mg/dL
d. 200mg/dL
d

A potential childhood risk factor for development of atherosclerotic or coronary heart disease as adults is:
a. obesity
b. tachycardia
c. heart murmur
d. aerobic exercise
a

Which of the following is not likely to cause secondary hypercholesterolemia?
a. nephrotic syndrome
b. hypertension
c. corticosteroids
d. obstructive liver disease
b

Which of the following is a common cause of acquired coronary artery disease during childhood?
a. Rheumatic fever
b. hypertension
c. Systemic lupus erythematous
d. Kawasaki disease
d

Kawasaki disease is most common in:
a. neonates
b. children less than 5yo
c. children over 6yo
d. females
b

A principal clinical feature of Kawasaki disease includes:
a. low grade fever for 24hrs & pruritic rash
b. conjunctivitis with exudate & facial rash
c. arthritis & chorea
d. fever persisting at least 5 days & acute erythema and/or edema of hands & feet
d

An essential test in the evaluation of a 2yo being managed for Kawasaki disease is:
a. an echocardiogram
b. electrolytes
c. cholesterol
d. streptococcal antibody titer
a

In regards to generalized anxiety disorder (GAD), evidence has shown that
a. cognitive behavior therapy leads to higher response rates than medication alone
b. less than 50% respond to psychotherapy alone
c. GAD is less responsive to psychotherapy than other psychiatric illnesses
d. medication therapy provides the basis for treatment
a

The MOST appropriate treatment for children with uncomplicated obstructive sleep apnea (OSA) is
a. positive airway pressure (Bi PAP)
b. oxygen therapy
c. tonsillectomy and adenoidectomy
d. nasal steroids
c

A 10 yo with no allergies, weighing 88 lb (40kg), presents with a fever to 102 degrees F (38.8 C), nausea and sore throat for 3 days. Physical exam reveals palatial petechiae. What is the BEST management?
a. throat culture and treatment with oral potassium penicillin V, 500 mg BID for 10 days
b. throat culture and treatment with amoxicillin, 250 mg QD for 10 days
c. treatment with benzathine penicillin G, 600,000 units IM one dose
d. treatment with oral azithromycin, 400 mg today and 200 mg QD for 4 additional days
a

Which of the following primary interventions assists in the prevention of adolescent depression?
a. recommending the allocation of social isolation time
b. anticipatory guidance that mild depression is normal
c. recommending help from professionals rather than friends
d. discussing changes associated with adolescence
d

The factor with the STRONGEST link to malignant melanoma is
a. family history of malignant melanoma
b. fair skin coloring
c. living near the equator
d. sun exposure early in life
d

A 19 month old had a five minute episode of generalized shaking following by brief unresponsiveness. Physical exam is unremarkable except for fever and bilateral otitis media (BOM). Initial appropriate management includes
a. treating the bilateral otitis media (BOM)
b. ordering an EEG
c. ordering a lumbar puncture
d. ordering serum electrolytes
a

In an adolescent with pelvic inflammatory disease (PID), which is an indication for hospitalization?
a. elevated C-reactive protein
b. oral temperature over 102d F
c. positive endocervical culture for N. gonorrhea
d. positive urine HCG
d

Foods containing high levels of iron appropriate to suggest for a 2 yo with iron deficiency anemia include
a. almonds, brown rice, peanut butter
b. brown rice, Cheerios, kidney beans
c. pancakes, peanut butter, Cheerios
d. pancakes, broccoli, almonds
b

Adolescents who receive teaching about condoms should also have information about
a. the brands least likely to break during intercourse
b. the application of petroleum jelly to prevent dry intercourse
c. the us of nonoxynol 9 as a lubricant and spermicide
d. the use of oil based lubricants as safe lubricants
c

A 12 year old has a 4-week history of a paroxysmal cough and occasional post-tussive emesis. Physical exam is essentially unremarkable except for upper respiratory infection(URI) signs. The MOST LIKELY causative organism is
a. Streptococcus pneumoniae
b. Bordetella pertussis
c. Haemophilus influenzae
d. Mycoplasma pneumoniae
b

Which institution legally defines the scope of practice for pediatric nurse practitioners?
a. graduate schools
b. certification board
c. state legislation
d. NP membership organiation
c

A 13 month old with iron deficiency anemia has just complete one month’s treatment with ferrous sulfate therapy. Management at this point includes
a. obtaining a complete blood count
b. continuing the medication for another 2 months
c. obtaining a serum iron level
d. replacing the ferrous sulfate with diet high in iron
b

Initial examination for the presence of unilateral choanal atresia in a newborn is BEST achieved by
a. alternately occlusion of each nares while assessing for air exchange in opposite side
b. attempting to pass a nasal gastric tube into each naris
c. using an otoscope to view the nasal turbinates for color and shape
d. observing for broadening of the nasal bridge
a

A 10 yo with a history of anaphylactic reactions to be stings is given a prescription for an EpiPen self-injectable (epinephrine 1:1000), 0.3 mg. Instructions to use include
a. keep the device refrigerated at all times to prolong its shelf life
b. administer only after consultation with a health care provider
c. regular review of usage and expiration date
d. administer the dose subcutaneously for most rapid absorption
c

A 12 yo presents with peri-umbilical pain which keeps the child from standing straight. Over the past few days, the child has also experienced mild diarrhea and fever to 102d F. Which should be done FIRST?
a. obtain a CBC and ESR
b. obtain an abdominal x-ray and CBC
c. refer to the emergency department and contact pediatric surgeon
d. obtain an abdominal CT scan and refer to the emergency department
c

A healthy 24 month old can be expected to have achieved which of the following language milestones?
a. counts three objects correctly
b. has a 30-50 word vocabulary
c. states full name
d. tells a story
b

A 7 year old fell, hitting the right wrist. X-rays of the wrist were negative, and the child was treated for a bad sprain and sent home with a sling. 10 days later the child is still complaining of pain in wrist with point tenderness. Which should be the NEXT step in management?
a. discontinue use of the sling
b. order a bone scan
c. repeat wrist radiographs
d. refer to orthopedic surgeon
c

When calculating oral replacement therapy for a child with moderate dehydration from gastroenteritis, which guideline should be followed?
a. intake should equal combined stool and urine output
b. stool losses should be replaced in addition to maintenance requirements
c. quickly replace fluid deficits
d. fluid intake should be equal to twice the stool output
b

Which factor is MOST suggestive of an organic cause of recurrent abdominal pain?
a. pain localized to the periumbilical area
b. pain accompanied by diaphoresis
c. pain associated with a change in bowel habits
d. patient’s description of the pain as sharp in nature
c

What is the MOST common side effect noted when oxybutynin chloride (Ditropan) is used in the management of dysfunctional voiding?
a. headache
b. dry mouth
c. palpitations
d. diarrhea
b

Which symptoms would prompt the ordering of a genetic test for Prader- Willi syndrome?
a. obesity, daytime hyperactivity, increased libido
b. large hands/feet, thin saliva
c. neonatal hypotonia, feeding difficulties
d. failure to thrive, hyperphagia, clitoromegaly
c

Emphasis on promoting healthy behavioral changes for the overweight child should include which of the following?
a. establishing family goals
b. maintaining family holiday meal rituals
c. weighing child daily
d. using weight loss as a sign of success
a

The MOST important rationale regarding the use of oral contraceptives for an adolescent with primary dysmenorrhea includes that
a. the use of oral contraceptives will regulate and lighten menstrual flow
b. hormones in oral contraceptives will suppress ovulation, reduce endometrial growth and total prostaglandin production
c. hormones in oral contraceptives will suppress ovulation, reduce menstrual flow, and increase total prostaglandin production
d. the use of oral contraceptives will provide pregnancy protection
b

Leukotriene modifiers in children with asthma
a. act solely on bronchial hyperactivity
b. are recommended for monotherapy with any persistent asthma
c. are approved for use as young as one year
d. have been efficacious in preventing exercise symptoms in adolescents
d

Which should always be included in the evaluation of a child with a suspected learning disorder?
a. full history and physical exam
b. Denver II developmental assessment
c. genetic and chromosomal testing
d. intelligence quotient measurement
a

Implications for nurse practitioners who counsel adolescents on health risks related to smoking are based on current research concluding that
a. early cigarette initiation age often results in adult nicotine dependency and is associated with other risk behaviors
b. cigarette use frequently decreases as the grade in school increases
c. adolescent males are more likely to report cigarette us in the past thirty days
d. pregnant adolescent females are less likely to smoke during pregnancy than adult women
a

Prophylaxis for secondary prevention following a definitive diagnosis of rheumatic fever includes
a. penicillin G IM every 28 days
b. penicillin G IM every 60 days
c. amoxicillin orally prior to dental procedures
d.erythromycin orally twice a week
a

What is the MOST important safety question to ask a 17 yo?
a. Are you aware of ways to avoid accidents?
b. Do you play violent video games more than 2 hours a day?
c. Do you always wear a seat belt when driving or riding in a car?
d. Is there a gun anywhere in your house?
c

A MOST concerning cardiac finding in the initial newborn assessment is
a. a soft, grade III/IV systolic murmur
b. a cuff blood pressure of 65/35
c. decreased femoral pulses
d. a regular heart rate of 95 while sleeping
c

A 3 yo has a history of severe asthma with six hospitalizations over the past 2 years. One parent states that the other parent continues to smoke around the child and requests you to report the spouse to CPS for neglect. Before filing such report, it is most important to determine that the
a. smoking parent has been adequately educated as to the danger of secondhand smoke
b. non smoking parent has been adequately educated as to the danger of secondhand smoke
c. child has been shown on RAST testing to be allergic to secondhand smoke
d. child has been reported in the past to experience wheezing when exposed to smoke
a

The American Academy of Pediatrics (AAP) recommends that child ASSENT to participate in clinical research requires evidence of which of the following?
a. full understanding of the risk and potential benefits of the treatment
b. rejection of non-research alternatives
c. acceptance of unfamiliar procedures
d. deveolpmentally appropriate understanding of the nature of his condition
d

What is the MOST COMMON condition seen with sudden exercise-associated cardiac death in young athletes?
a. inherited hypertrophic cardiomyopathy
b. intaventricular conduction abnormalities
c. prolonged QT syndrome
d. atrio-ventricular block
a

A 10 yo obese child whose family history is unknown has a total cholesterol (TC) of 205 when first screened for hyperlipidemia. The MOST appropriate course of action is to
a. repeat TC level in 2-4 weeks
b. repeat TC level after 3 months on a low-fat diet
c. perform a lipoprotein analysis now
d. perform a lipoprotein analysis after 3 months on a low-fat diet
c

Internationally adopted children from ANY country should receive testing for
a. tuberculosis, hep B and C, syphilis, HIV
b. measles, hep A and C, tuberculosis, malaria
c. syphilis, cytomegalovirus, hep A and C, HIV
d. measles, hep A and C, cytomegalovirus, malaria
a

In a 9 month old with sensorineural hearing loss (SNHL), which one of the following is most likely to determine the etiology with the highest cost:benefit ratio?
a. MRI
b. electrocardiogram
c. genetic evaluation
d. TORCH serology
c

A 14 yo girl and her parent express concern that she has not begun to have her period. Pubic hair development started about 2 years ago, and breast development began about 18 months ago. She has grown 2 inches in height in the past 6 months. Which would be the MOST accurate response?
a. Menses generally begins in girls within 1 year after breast development begins
b. The appearance of pubic hair before any breast development indicates need for endocrine evaluation.
c. As menses will probably begin in the next 6-12 months, a wait and see approach is appropriate
d. No current need for worry, as menses may not begin until the age of 18
c

A 14 yo old, cross country runner, has a 3-4 week history of knee pain, which increases with activity. On exam, there is notable pain and tenderness over the tibial tuberosity.
What would be the BEST management strategy?
a. modify activities that cause pain until the inflammation subsides
b. refer to orthopedic surgeon for evaluation
c. refer for physical therapy
d. obtain MRI to rule out malignant diesase
a

Shelby, a 4 week old, presents to your office in mid-January with one week history of nasal congestion and occasional cough. On the evening prior to this visit Shelby developed a temp of 102F, refused to breastfeed, had paroxysmal coughing, & noisy, labored breathing. On exam, you note an ill-appearing infant who is lethargic with tachypnea and intercostal retractions. Shelby does not attend daycare but has a 3yo sibling who is in daycare & just had a “cold”. Considering the clinical presentation, what is the most likely cause of Shelby’s illness?
a. Mycoplasma pneumonia
b. RSB bronchiolitis
c. Aspiration pneumonia
d. Streptococcal infection of the pharynx
b

Of the following children, which one should not have tuberculin skin testing?
a. Richard a 14yo, whose uncle was recently granted parole after 5 years in prison and is currently living with Richard’s family
b. Theresa a 2yo who was infected with RSV 3mo ago and is currently asymptomatic
c. Han, a 3mo old whose family emigrated to the US from Cambodia a month ago
d. Chris, an 18mo old whose mother is infected with HIV
b

Which of the following clinical presentations least warrants sweat chloride testing?
a. 10yo female sibling of a pt newly diagnosed with cystic fibrosis, sibling is without pulmonary problems and growth parameters are at 50% for age
b. 2yo male with recurrent pneumonia and growth parameters at 5% for age
c. 4yo female with nocturnal cough, which resolves after treatment with bronchodilators and short term steroids; growth parameters at 10% for age
d. 7yo female with nasal polyps, mildly hyperexpanded lungs, growth parameters at 25% for age
c

Of the following diagnostic findings, which one should be referred to a specialist immediately?
a. suspected foreign body aspiration
b. sweat chloride results 30mEq/L
c. pulmonary function tests of 85% predicted
d. chest radiograph with hyperexpansion
a

What is the most common agent for nonviral pneumonia from older preschool to young adulthood?
a. mycoplasma/chlamydia aureus
b. staphlycococcus aureus
c. ureaplasma
d. Haemophilus influenza
a

Which one of the following diagnoses would not be part of the differential for recurrent lobar pneumonia in a 2yo?
a. cystic fibrosis
b. Foreign body aspiration
c. atelectasis
d. bronchitis
d

The most common clinical presentation of pneumonia includes:
a. cough, fever, tachypnea, abdominal pain
b. hemoptysis, putrid breath, and weight loss
c. sudden chest pain, cyanosis
d. retractions, stridor
a

In addition to airway hyper-responsiveness and reversible airway obstruction, asthma is a chronic lunch disease characterized by:
a. Bronchiectasis
b. Inflammation
c. Pleural effusion
d. Pulmonary edema
b

The most common trigger for acute asthma episode in the very young child is:
a. Respiratory infections
b. exercise
c. tobacco smoke
d. Outdoor allergens
a

Luke has a mild persistent asthma. Appropriate daily medication should include:
a. an inhaled low-dose corticosteroid
b. short acting beta2 agonists
c. an oral systemic corticosteroid
d. a cough suppressant
a

Which of the following is not a good goal of appropriate asthma management?
a. limited activity and exercise
b. prevent recurrent exacerbations
c. prevent chronic troublesome symptoms
d. maintain near normal pulmonary functions
a

Deon is a 4yo male with a hx of atopic dermatitis and recurrent pneumonias, according to his mother. He presents with a persistent nighttime cough. His most likely diagnosis is:
a. asthma
b. foreign body aspiration
c. croup
d. cystic fibrosis
a

The most typical chest radiographic finding consistent with the diagnosis of asthma is:
a. normal chest film
b. diffuse airway edema
c. right upper lobe infiltrate
d. hyperinflation
d

When providing asthma education regarding the use of long-acting beta2 agonist, it is important to stress:
a. it should not be used as a quick relief medication
b. may be given every 30min TID for rescue therapy
c. may be most beneficial for exercise induced asthma
d. should never be taken while also using inhaled corticosteroids
a

Claire is an 8yo with moderate persistent asthma who is still having a daily cough. She reports three times a day use of short acting inhaled beta 2 agonist and cromolyn sodium at her clinic visit. Her management plan should be altered to include:
a. broad spectrum antibiotics and recheck in 2 weeks
b. addition of systemic corticosteroids for 5 days
c. replace cromolyn sodium with inhaled corticosteroids
d. addition of an inhaled anticholinergic
c

Major contributors to asthma morbidity and mortality are:
a. underdiagnosis & inappropriate treatment
b. an increase in indoor allergens
c. overuse of anti-inflammatory medications
d. an increase in air pollution
a

The primary treatment for bronchopulmonary dysplasia is:
a. pancreatic enzymes
b. surgical repair
c. adequate oxygenation
d. chest physiotherapy
c

The single most predictive factor in the development of bronchopulmonary dysplasia is:
a. birth weight
b. maternal age
c. maternal education level
d. respiratory infections
a

The classic radiograph finding in croup is:
a. hyperinflation
b. perihilar lymphadenopathy
c. thumb sign
d. steeple sign
d

Unilateral wheezing is a finding suggestive of:
a. croup
b. asthma
c. foreign body aspiration
d. cystic fibrosis
c

Which of the following is not characteristic of an apparent life-threatening event (ALTE):
a. change in muscle tone
b. fever
c. change in skin color
d. apnea
b

Following an ALTE, management and treatment are based on findings from:
a. a thorough history and physical exam
b. an electroencephalogram
c. chest radiograph
d. sleep study
a

The predominant characteristic of a young infant with bronchopulmonary dysplasia is:
a. prolonged fevers
b. hypoxemia on room air
c. recurrent pneumonias
d. chronic hypoinflation
b

J.D. is the postterm infant with lesions of varying morphology including wheals, vesicles, and pustules on her trunk. You suspect J.D. has:
a. cutis marmorata
b. Erythema toxicum neonatorum
c. Milia
d. contact dermatitis
b

In order to confirm your diagnosis of JD (erythema toxicum neonatorum), you order a Wright’s stained smear. If your diagnosis is correct, what are the expected results of the smear?
a. presence of eosinophils
b. presence of neutrophils
c. presence of keratinous material
d. presence of staphylococcus bacteria
a

In addition to monitoring the skin for any changes, what is the best management for JD (toxicum neonatorum)?
a. topical antibiotics on lesions
b. topical steroids on lesions
c. a moisturizer on lesions
d. No treatment necessary since JD’s condition will resolve spontaneously in 5-7 days
d

You examine C.C. a newborn, & observe numerous white papular lesions on the cheeks, forehead, and nose. You suspect either milia or neonatal acne. Which physical finding helps to confirm a diagnosis of milia?
a. Papular lesons are intermixed with pale yellow macules
b. papular lesions have an erythematous circular ring at the base
c. papular lesions are surrounded by lacy-blue area with erythematous mottling
d. papular lesions, yellow in color, are observed on the hard palate
d

Newborn KT is 3 weeks premature and you observe a macular erythematous lacy appearance to her skin when you undress her. KT has which condition?
a. cutis marmorata
b. erythema toxicum neonatorum
c. salmon patch
d. nevus flammeus
a

In addition to monitoring the skin for any changes (cutis marmorata), what is the best management for KT?
a. Keep KT warm
b. Decrease the environmental temperature
c. use a moisturizer on affected skin areas
d. Do nothing, as condition will resolve spontaneously in 5-7 days without intervention
a

Newborn WR has a vascular lesion that will not fade as she gets older. What is your diagnosis?
a. Salmon patch
b. Capillary hemangioma
c. Cafe au lait
d. Port-wine stain (nevus flammeus)
d

WR’s (port-wine stain/nevus flammeus) are concerned about her appearance and the psychological effect on their daughter as she becomes aware of her condition. In educating the parents, you tell them about several options. Which of the following is not an appropriate management or treatment consideration for WR?
a. application of topical steroids to the affected area to prevent pruritus
b. camouflage the affected areas with cosmetics
c. pulsed laser treatment of affected area
d. counseling for psychological concerns
a

Which condition is thought to be more apparent in darker-skinned individuals or during the summer months?
a. Tinea corporis
b. psoriasis
c. Pityriasis alba
d. Pityriasis rosea
c

JR, an 8yo boy has scaly, hyperpigmented lesions in a, “fir tree” distribution, predominately on his trunk. One lesion on the buttocks is larger than all other other lesions and measures 4cm in diameter. What is your likely diagnosis?
a. Psoriasis
b. Eczema
c. Pityriasis alba
d. Pityriasis rosea
d

What symptom is commonly experienced in pityriasis rosea?
a. pruritus
b. pain on site of lesions
c. nausea
d. HA
a

What management would you not recommend for pityriasis rosea?
a. cool bath or cool compress to lesions
b. topical steroids to lesions
c. oral antibiotics
d. monitored and controlled daily sunlight exposure
c

You have diagnosed DL with acute atopic dermatitis. Which of the following is not correct regarding the incidence of this condition?
a. DL is most likely an infant
b. DL has a greater chance of developing asthma later in childhood than the average individual
c. DL has a greater chance of developing malignant melanoma in adulthood than the average individual
d. DL has a condition associated with familial predisposition
c

Which of the following management measures or treatments would you not recommend for acute atopic dermatitis?
a. topical steroids to affected areas
b. wet compresses to affected skin areas
c. Maintain a dry, warm environment
d. eliminate all substances that dry the skin
c

In addition to having atopic dermatitis, you have diagnosed DL with a secondary bacterial infection at the site of several lesions. what is the best management for the infection?
a. topical antibiotics to affected areas
b. oral antibiotics
c. hot compresses to affected areas
d. Monitored and controlled daily sun exposure until lesions resolve
b

You see BD for the first time at age 6 weeks. BD has bright red, raised, rubbery lesion of irregular shape & 2cm in diameter on the occiput. What condition do you suspect BD has?
a. malignant melanoma
b. Port-wine stain
c. Capillary hemangioma
d. Burn
c

Which of the following is not a characteristic of the lesion BD has (capillary hemangioma)?
a. It was not present at birth, however BDs mother noticed site was blanched
b. it will continue to grow for the first 9-12 months of BD’s life
c. It will begin to gradually resolve when BD is between 12-15mo old
d. It is expected to completely resolve by the time BD is 10yo
c

You notice 10 macular tan lesions of varying sizes on DD and refer him for medical eval to rule out neurofibromatosis of Albright’s syndrome. What kind of lesion does DD have?
a. Malignant melanoma
b. Cafe au lait spots
c. mongolian spots
d. vitiligo
b

What is characteristic of the lesion that DD has (cafe au lait)?
a. more common in caucasians than dark skinned individuals
b. more common in males than females
c. lesions can be present at birth, however, more lesions may develop at any age
d. lesions usually fade spontaneously and completely resolve in adult life
c

You suspect at A.F. age 9yo; has either pityriasis alba or vitiligo. Which of the following would not confirm the diagnosis of pityriasis alba?
a. AF’s skin would be normally pigmented except for areas of depigmentation
b. AFs skin would have one or more scaly areas of hypopigmentation
c. AF complains of mild itching in areas of hypopigmentation
d. AFs lesions became more pronounced when she was exposed to sunlight
a

AF was diagnosed with pityriasis alba. Which of the following is proper management of AF’s condition?
a. bland moisturizers to reduce overdrying
b. topical steroids to the affected areas
c. expose affected areas to short periods of sunlight each day
d. Burow’s wet compress to affected areas
a

Patient education is a major part in the PNP’s role. What would you teach AF and her parent regarding the progress and prognosis of pityriasis alba?
a. AF will continue to develop lesions for the rest of her life
b. AFs condition should fade appreciably in 3-4months
c. AF’s condition is permanent and affected areas will not repigment
d. AF’s condition will resolve completely, however, the affected areas can become slightly reddened when exposed to sunlight
b

malignant melanoma is a form of much dreaded skin cancer. Which of the following is not characteristic of this condition?
a. occurs in all ethnic groups but more commonly in light skinned individuals
b. severe sunburn or excessive exposure to sunlight before the age of 10yrs predisposes developing melanoma later in childhood or in adult life
c. Spreads through the lymphatic system and invades other distant skin surfaces and organs.
d. spreads primarily by invading skin surfaces that surround the major lesion
d

Which of the following does not characterize the lesion of malignmant melanoma?
a. irregular assymmetrical nodule with blurred borders
b. raised with distinct symmetrical borders
c. uneven coloring in which blue, black, brown, tan, and red may all be present in the same lesion
d. bleeding, ulceration in later stages
b

patient education regarding prevention of malignant melanoma is essential. Which of the following is not considered best prevention education?
a. avoid sunlight, especially during the hours of 9am-1pm
b. avoid sun tanning lamps
c. use cover up clothing, hats, and sunglasses
d. use sun blocks that protect against UV exposure with 30SPF
a

You suspect MN as having chronic psoriasis. Which of the following is characteristic of her lesions if she has psoriasis vulgaris?
a. scaly erythematous patches and plaques 3-10mm in diameter
b. round or oval in shape
c. large scaly silver-white plaque 5-10cm in diameter
d. located mainly on her trunk
c

MN’s condition (psoriasis vulgaris) of psoriasis is common in approximately 33% of children. Which of the following is not correct regarding the etiology or incidence of this condition?
a. occurs more commonly in dark-skinned ethnic individuals
b. associated with constant rubbing or trauma to exposed affected areas such as elbows
c. associated with overproduction of epithelial cells
d. associated with epithelial cells that migrate to the skin surface much more quickly than normal
a

What would you not advise regarding the management or treatment of MN’s condition (psoriasis vulgaris)?
a. excise lesions
b. apply topical steroids
c. apply mineral oil & moisturizers
d. expose to monitored short periods of sunlight
a

You have diagnosed Jale as having contact dermatitis. Which symptom is most characteristic of his condition?
a. HA
b. difficulty breathing
c. pruritus at site of affected areas
d. pain at site of affected areas
c

Which of the following is not characteristic of Jale’s condition (contact dermatitis)?
a. He has hypersensitivity to a substance within his environment when direct contact is made
b. He may experience a delayed reaction of several days with re-exposure to an allergen
c. His dermatitis may be caused by direct contact with topical medications, soaps, cosmetics, fabrics, and plants
d. Typical response is redness and edema at the site of contact which may progress to papules and vesicles
b

What would you not recommend as management and treatment of Jale’s condition (contact dermatitis)?
a. Skin testing during the acute episode to determine if Jale has an allergy
b. Cool compress of Burow’s solution to affected areas
c. Topical steroids to affected areas for 5 days
d. Oral antihistamines
a

You diagnose Kelli, 7mo, with diaper dermatitis. Which of the following should not be included in the differential diagnosis?
a. Atopic dermatitis
b. Child abuse
c. Contact dermatitis
d. Pityriasis alba
d

What management measure would you not prescribe to treat Kelli’s condition (diaper dermatitis)?
a. Oral antihistamines
b. Lubricants such as petrolatum jelly to mildly affected areas
c. Low potency topical steroids to severely affected areas with erythema & papules
d. Topical antibiotics to severely affected areas with ulcerations
a

What would not be an appropriate recommendation to prevent Kelli from having subsequent episodes of diaper dermatitis?
a. Expose diaper area to air several times each day
b. Increase oral fluids using orange juice to dilute urine
c. Make diaper changes immediately after soiling
d. Use a double rinse of vinegar and water for home-laundered diapers
b

Seborrhea dermatitis is common in both infants and adolescents. Which of the following is not correct of this condition?
a. Can cause irritation pigment changes to include hyperpigmentation and hypopigmentation
b. Is associated with an overproduction of sebum in areas abundant with sebaceous glands
c. The condition in infants is known as “cradle cap” in which lesions have erythematous base with yellow crusted areas and greasy scales
d. The condition in adolescents is known as acne with comedomes, papular and pustular lesions
d

What is the best treatment of seborrhea in the infant?
a. Mineral oil to loosen crusts prior to washing affected areas with a nonperfumed baby shampoo
b. Topical antibiotics
c. Oral antibiotics in severe cases
d. Oral steroids for severe cases
a

You are evaluating FP, 3yo, who acutely sustained a burn when she pulled a pan of boiling water onto herself within the past hour. Since burns are classified according to the depth of the injury to the skin layers and the amount of area involved, how would you rate the 5% of her body surface is burned involving the epidermis and upper part of the dermis?
a. She has a minor first and second degree burn
b. She has a major second degree burn
c. She has a major full thickness burn
d. She has a major first and second degree burns
d

FP’s burn (major first & second degree; 5% of body surface) should appear:
a. Dry with mild edema and erythema
b. Dry whitish areas that blanch with pressure
c. Dry whitish to brownish areas with edema
d. Moist edema, erythema, and few vesicles
d

What is the best treatment for FP’s burn (major first & second degree; 5% of body surface)?
a. Warm compress to affected areas and mild analgesic for discomfort
b. Topical emollients to affected areas
c. Topical steroids to affected areas
d. Refer for urgent treatment in an ED
d

Jerry has been diagnosed as having folliculitis, an inflammatory condition involving the pilosebecaous follicle. What is the most common cause of this condition?
a. Microsporum canis tinea
b. Poxvirus
c. Staphlycoccus aureus
d. Streptococcus group a
c

Jerry has a condition that most commonly occurs on which body surface (folliculitis?
a. Neck & scalp
b. Upper arms
c. Chest & abdomen
d. Legs
a

You order a culture and the results confirm that Jerry’s condiditon (folliculitis) is caused by the most common organism for this condition. What treatment do you prescribe?
a. Oral penicillin
b. Dicloxacillin
c. Tinactin
d. Tretinoin
b

Sandra, 12yo, has several vesicles and honey colored crusted lesions on her face above the right nares. She has a hx of having had a scratch in the same area several days ago. What condition do you suspect?
a. Acne
b. Impetigo
c. Herpes simplex
d. Eczema
b

Judy, 15yo has been diagnosed as having acne. Which one of the following is not true of this condition?
a. Poor hygiene is the primary cause of acne
b. Associated wit increased androgenic hormonal activity
c. Females can have a “cycle” component to their acne
d. Severe acne having a later onset in puberty is more common among males
a

  1. Judy has a hx of remission and exacerbation of acne that has followed the pattern of menses for two years. However, the condition over the last 6 months has worsened to a moderate degree of severity and has been chronic & persistent. You prescribe antibiotic therapy. Which of the following antibiotics would you not consider?
    a. Topical clindamycin
    b. Oral erythromycin
    c. Oral minocyline
    d. Oral tetracycline
    b
  2. KC age 13yo has several firm, small (2mm), white or skin colored umbilicated papules on her neck. The lesions have been present for 3mo and have increased in number. What is your diagnosis?
    a. Acne
    b. Molluscum contagiosum
    c. Warts
    d. Cellulitis
    b
  3. What is the cause of KC’s condition (molluscum contagiosum)?
    a. Microsporum canis tinea
    b. Poxvirus
    c. Staphylococcus aureus
    d. Streptococcous group a
    b
  4. Which treatment would you not recommend for molluscum contagiosum?
    a. Curettage lesions
    b. Oral antibiotics
    c. Observation
    d. Topical imiquimod
    b
  5. Paul has 4 superficial lesions on his anterior lower abdomen of one week duration. The lesions are 4cm in diameter, scaly, irregular shaped plaques with skin colored centers and erythematous borders. The affected areas are slightly pruritic. What condition do you suspect Paul has?
    a. Psoriasis
    b. Eczema
    c. Tinea corporis
    d. Pityriasis rosea
    c
  6. You performed two tests to confirm your diagnosis of Paul’s condition (tinea corporis). The KOH scraping was + for the presence of hyphae. The Wood’s lamp did not fluoresce the lesions. You are sure that Paul’s condition was not caused by which organism?
    a. Epidermophyton floccosum
    b. Mircosporum canis
    c. Trichophyton tonsurans
    d. Trichophyton rubrum
    b
  7. You see Paul after 8 weeks of treatment with antifungal preparation (for tinea corporis). The original lesions have almost resolved, however the condition has worsened with the development of several other larger lesions on the abdomen and groin area. Which of the following would you not consider?
    a. Oral antifungal medication, griseofulvin
    b. Topical antibiotic preparation
    c. Continue with the topical antifungal applications
    d. Educate again regarding not sharing personal items
    b
  8. Dale, age 7yo, is complaining of pain & burning on his right leg where you observe two small red puncture marks surrounded by a blanched area with an erythematous border. He had been playing with his dog all morning outside in a grassy wooded area near his home and was wearing shorts. You suspect he has been bitten by which insect?
    a. Mosquito
    b. Bee
    c. Recluse spider
    d. Black widow spider
    d
  9. Which of the following is not true of insect stings from bees, wasps, and fire ants?
    a. Greater reaction of hypersensitivity occurs most often with the initial exposure than with subsequent exposures
    b. For mild reactions, cool compresses to the site of injury is the usual management
    c. Occurs more often during the spring and summer months
    d. Most stings occur in self-defense when the nonaggressive insect feels threatened or irritated
    a
  10. You diagnose WA with scabies. Which of the following is not characteristic of this condition?
    a. He has several erythematous papular, pustular, and crusted lesions on his face
    b. He has several excoriated scratched areas around the umbilicus and waist area
    c. He has several linear curved lines approximately 4mm in length with a papule at the proximal end linear line
    d. He complains of severe pruritus which is worse at night
    a
  11. Which of the following is not recommended as a management and treatment strategy and treatment for WA (with scabies)?
    a. Put nonwashable items in a plastic bag & store for 1 week
    b. Prescribe topical antifungal applications
    c. Prescribe topical antiparasitics
    d. Prescribe topical steroids and/or oral antihistamines for pruritus
    b
  12. Pediculosis is highly communicable, common condition in children. Which of the following is not correct of pediculosis humanus?
    a. Caused by an insect that does not fly or jump
    b. Gravid females lay ova in seams of clothing
    c. Likes hairy areas of body better than non-hairy body surfaces
    d. Same medication used for scabies may be used to effectively eradicate this condition
    b
  13. Hypersensitivity may occur to a variety of substances causing a variety of reactions. It is important to determine if the body’s hypersensitivity reaction will cause erythema multiforme condition. Which of the following is not typical of the erythema multiforme reaction?
    a. Target “bulls eye” lesion with a necrotic center surrounded by a pale macular middle area and then by an eyrthematous peripheral ring
    b. Itching at site of affected areas
    c. Pain at site of affected areas, especially in the oral cavity
    d. Lesions which all have the same morphology on the trunk
    d
  14. You see DY in your clinic and suspect she has a form of erythema multiforme. Erythema multiforme minor must be differentiated from erythema multiforme major. Which of the following is the most important confirming evidence for making a diagnosis of erythema multiforme major?
    a. Presence of deeper lesions within the dermis
    b. Presence of lesions on the exposed areas of the body
    c. Presence of pustules indicating a secondary infectious process
    d. Occurrence of prodromal systemic symptoms of fever, malaise, sore throat, HA, nausea, and/or vomiting
    d
  15. You suspect DY has erythema multiforme major. What treatment or management is most indicated?
    a. Prescribe topical antibiotics due to secondary infection
    b. Prescribe topical steroids to lesions for pruritus
    c. Refer for medical evaluation
    d. No treatment is indicated as condition will resolve spontaneously in one week
    c
  16. Urticaria is a hypersensitivity allergic reaction to a variety of substances and agents. You suspect WP has urticaria due to the typical morphology of lesions on her trunk and arms which are:
    a. Erythematous papules
    b. Vesicles
    c. Pustules
    d. Wheals
    d
  17. During WPs acute episode of urticaria which of the following is not considered an appropriate management or treatment measure?
    a. Oral antibiotics to prevent secondary infection
    b. Oral antihistamines for pruritus
    c. Topical steroids to affected areas to reduce the immune response
    d. Cool compress to affected areas
    a

In addition to screening by checklist of a 17 month old suspected of having autism, what other testing should be included?
a. occupational therapy evaluation
b. MRI of the brain
c. hearing screening
d. chromosomal analysis
c

An 18 month old presents with moderate symptoms of croup of 8 hours duration. Which is the MOST appropriate therapy?
a. oral amoxicillin 40 mg/kg divided TID for 10 days
b. helium-oxygen
c. racemic epinephrine by nebulization
d. oral dexamethasone 0.6 mg/kg
d

The parent of a 2-month-old, formula fed infant, reports that the child vomits after after almost every feeding. The infant’s birth weight was 7 lbs and he now weights 10 lbs., 4 oz. Physical exam reveals a healthy, thriving infant. The most likely reason for vomiting is
a. pyloric atenosis
b. overfeeding
c. lactose intolerance
d. constipation
b

In supporting parents of a school aged child with a learning disability, the most important PNP role is to:
a. understand the development of the IEP (Individualized Education Plan) and 504 Plan
b. have a collaborative relationship with the local psychiatrist who works with learning disabled children
c. identify the participants in the student study team or child guidance group
d. prescribe the most common medications used for ADHD
a

A previously healthy 18-month-old girl has completed 10 days of an antibiotic for a first febrile urinary tract infection (UTI), and is now asymptomatic. Which management option would be MOST appropriate for this child?
a. monitor for recurrent infections by urine dipstick testing until imaging results are known
b. obtain a voiding cystourethrogram (VCUG) only if renal ultrasound (RUS) is abnormal
c. obtain VCUG if a second febrile UIT occurs
d. administer daily antibiotic prophylaxis until results of RUS and VCUG are known
c

A 16-year-old volleyball player is diagnosed with long QT syndrome after a syncopal episode during practice. Management for this patient includes
a. avoiding further involvement in competitive sports
b. changing to swim team for varsity sport
c. beginning mexiletine as an antiarrhythimic
d. taking beta blockers before competition
a

The parents of a 5 yo are frustrated by their child’s need to select her own clothing, despite weather and safety. The BEST example of a parent statement to the child that wil promote self esteem
a. “You must wear shoes and socks, but you can choose between the red or black ones.”
b. “Go ahead and wear the sandals, but you will be cold.”
c. “Wear the sandals, but take your shoes and socks in case you change your mind.”
d. “It is too cold to wear sandals, stay in your room until you decide to put on shoes and socks.”
a

Typical symptoms of migraine headache in a 14 yo usually include recurrence and
a. unilateral forehead pain, nausea and photophobia
b. bilateral temporal pain, cyclical vomiting and vertigo
c. unilateral forehead pain, ataxia and photophobia
d. bilateral forehead pain, nausea and an aura
a

Based on recent evidence, which statement is MOST appropriate when providing education regarding the delivery of inhaled albuterol to children?
a. Longer nebulized therapy enhances the medication delivery
b. Crying enhances nebulized medication delivery to the airway in infants
c. Using a metered dose inhaler with a spacer and mask provides similar efficacy as nebulized form
d. Using a metered dose inhaler with a spacer is indicated for children over the age of three
c

Anticipatory guidance for a parent with a child whose venous lead level is 15 mcg/dL should include
a. retesting in 2 months along with an evaluation of the living environment
b. retesting required every month along with a neurodevelopmental evaluation
c. emergent retesting required and evaluation for intravenous chelation
d. an abdominal x-ray with bowel decontamination
a

Diagnostic laboratory testing indicated for metabolic disorders in the newborn period always includes
a. glycosylated hemoglobin
b. urine electrolytes
c. serum amino acids
d. serum AST, GGT, ALT
c

In switching from short-acting to long-acting stimulant medication, especially methylphenidate, which factor is a key consideration
a. the duration of action of long-acting medications
b. the effects of long-acting medication are more consistent than those of short=acting mediations
c. the total daily dose may need to be increased when a child is switched from a short to long-acting medication
d. the child should be given a weekend “off” prior to switching from one form of stimulant to another
c

In switching from short-acting to long-acting stimulant medication, especially methylphenidate, which factor is a key consideration?
a. the duration of action of long-acting medications is less variable than that of short-acting medications
b. the effects of long-acting medication are more consistent than those of short-acting medication
c. the total daily dose may need to be increased when a child is switched from a short to a long-acting medication
d. the child should be given a weekend “off” prior to switching from one form of stimulant to another
c

Developmental milestones for a 5 month old include
a. rolling from back to front, reaching for an objects and smiling spontaneously at familiar faces
b. sitting unsupported, transferring an object form hand to hand, smiling, and cooing
c. making sounds that mimic one syllable words, rocking on hands and knees, fine pincer grasp
d. crying when parent leaves the room, beginning to creep, holding onto a toy
a

Which MOST accurately reflects the expert consensus statement about the use of syrup of ipecac?
a. There is no evidence it improves outcomes of poisoning
b. It should be available in every household for emergency care
c. It is indicated in most poisonings unless there is a risk of aspiration
d. It has proven positive outcome when utilized at the scene of ingestion
a

A 4 month old with atopic dermatitis has dry patches on the cheeks and extensor surfaces of the arms and legs. Which is an important consideration in the management of this patient?
a. topical antihistamines are used to control pruritus
b. topical antibiotics are used to prevent infection
c. corticosteroid ointment preparations are preferable to creams
d. oral corticosteroid is the preferred treatment
c

Which BEST represents the description of Tanner stage 3 in an adolescent female?
a. areola and breast enlarges with no separation of contours, presence of pubic hair over the middle of the pubic bone
c. areola and breast enlarges with no separation of contours, appearance of fine, downy pubic hair along the labia
d. distinctive projection of the areola, appearance of fine, downy pubic hair along the labia
a

The MOST appropriate treatment for documented Lyme disease in a 5 yo, who weighs 55 lb (25 kg) is
a. erythromycin 250 mg given orally BID for 14 to 21 days
b. doxycycline 100 mg given orally BID for 14-21 days
c. amoxicillin, 400 mg given orally BID for 14-21 days
d. ceftriaxone 2500 mg given IM once
c

Children with Down syndrome have 15 to 20 times incidence of developing which of the following as compared to other children?
a. asthma
b. neuroblastoma
c. leukemia
d. diabetes
c

A 10 month old has had apparent abdominal pain for the past 4 hours. During episodes of pain, the infant screams inconsolably and draws his legs towards the abdomen, has vomited twice but has has no diarrhea. Between episodes the infant is calm and quiet. The MOST LIKELY diagnosis is
a. Meckel’s diverticulitis
b. intussusceptoin
c. acute appendicitis
d. acute gastroenteritis
b

A child with asthma symptoms which occur less than twice a week, with night symptoms less than twice a month and normal peak flow meter
a. daily use of a low dose inhaled corticosteroid
b. daily use of leukotriene receptor agonist
c. albuterol nebulizer or MDI for use with symptoms
d. medium dose inhaled corticosteroid, administered twice a day
c

Which of the following is the most appropriate information to use in providing anticipatory guidance for parents with a colicky baby?
a. colicky babies are hypersensitive to stimuli
b. teething can stimulate a colicky response
c. symptoms increase at 3-4 months of age
d. fever is often an accompanying symptom of colic
a

The parents of an alert, active 4 yo diagnosed with immune thrombocytopenia purpura (ITP) are frustrated because they feel “nothing was done” for their child. Physical exam reveals multiple purpura and petechiae, but no mucocutaneous bleeding. A repeat CBC shows platelets at 45,000/mm^3, mild anemia and no abnormalities of the red or white blood cells. The NEXT step should include
a. discussion regarding safety concerns
b. recommendation of hospitalization
c. initiation of steroid therapy
d. referral for IVIG infusion
a

The risk factor that is MOST predictive of suicide in an adolescent patient is
a. major mood disorder
b. previous suicide attempt
c. substance and alcohol abuse
d. family history of suicide
b

What is the BEST study to confirm the diagnosis of a 4 week old whose examination reveals a positive Barlow and Ortolani test?
a. arthrography
b. MRI
c. ultrasound
d. plain film radiograph
c

A 16 yo male presents with a three hour history of acute pain and swelling of the scrotum, severe enough to make examination difficult. What should be the FIRST course of action?
a. transillumination of the scrotum
b. Doppler study of the scrotum
c. referral to an urologist
d. obtain history of sexually transmitted disease (STD)
c

In which situation should treatment of the healthy child who is an asymptomatic carrier of Giardia lamblia be considered?
a. a child whose mother is pregnant
b. a child living in an endemic area
c. a 6 yo in a school setting
d. a 6 yo who has a 3 month old sibling
a

The most important evaluation of a 6 yo male with an acute, palpable varicocele is to
a. determine testicular size bilaterally
b. obtain scrotal ultrasound
c. review history for evidence of trauma
d. obtain abdominal/pelvic ultrasound
d

Counseling regarding the meningococcal disease for a prospective college freshman who plans to live in a dormitory should include that
a. the risk for meningococcal disease is just as high for commuting students
b. meningococcal vaccine is mandatory for all college students
c. meningococcal vaccine confers life long immunity
d. the risk for meningococcal disease is increased for freshmen college dormitory residents
d

A 4 week old has short palpebral fissures, hypoplastic philtrum and low nasal bridge with current symptoms of a poor suck and jitteriness,. Which of the following in-utero exposures is most likely?
a. cocaine
b. alcohol
c. amphetamines
d. barbiturates
b

The immunization history of a 23 month old is as follows:
Immunizations:
9 months: DTaP, HIB, Hep B, IPV, PCV 7
13 months: DTaP, HIB, Hep B, IPV, PCV 7, MMR, Var
The most appropriate recommendation is to now administer
a. Hep B, IPV, PCV 7
b. DTaP, HIB, Hep B
c. DTaP, HIB, Hep B, IPV, PCV 13
d. DTaP, HIB, Hep B, IPV, PCV 7
c

A 2 yo presents with a history of limping, fever, and refusal to walk or bear weight on extremity. INITIAL management includes
a. referral to hospital emergency department
b. thorough physical examination assessing for asymmetry and inflammation
c. referral to pediatric orthopedic surgeon
d. laboratory evaluation including CBC with differential and ESR
b

A 13-year-old adolescent complains of having difficulty walking and the arms feeling weak. The adolescent had a cold 2 weeks ago. On examination, the PNP is not able to elicit deep tendon reflexes. The PNP knows that the most appropriate course of action is to:
A) Refer the adolescent to an emergency room
B) Refer the adolescent to an orthopedic surgeon
C) Schedule an EEG
D) Schedule a follow-up visit in 1 week
a

The PNP is managing the case of a 2.5-year-old child with otitis media with effusion after an episode of acute otitis media. The effusion has now been present for 3 months. The most appropriate step is to:
A) Begin steroid medication
B) Refer the child for a hearing evaluation
C) Refer the child to an ENT for placement of tympanostomy tubes
D) Initiate antihistamine and decongestant therapy
b

A 4-year-old child sustained an insect bite near the left eye. The area is now red, warm to the touch, tender, and not well demarcated. The PNP:
A) Treats with antibiotics and an antihistamine
B) Orders sinus x-ray films
C) Consults with the physician
D) Orders a CBC and treats with antihistamines
c

An infant brought to the clinic is diagnosed with failure to thrive (FTT), persistent growth deficiency of the head, and delayed development. Some unusual facial abnormalities are noted. The mother has a history of consuming large amounts of alcohol. The PNP suspects fetal alcohol syndrome (FAS) and refers the infant:
A) For consultation with an endocrinologist
B) For consultation with a neurologist
C) For a genetic workup
D) To a First Steps or Birth to Three Program
d

A 5-year-old child is brought to the clinic for a “bad cold.” On physical examination a grade II/VI sharp, systolic murmur that had not previously been documented is noted. The radial and femoral pulses are 2+ and equal. The heart rhythm and tempo do not change with position, and the remainder of the examination is normal. An appropriate plan of care should include:
A) Obtaining pulse oximetry, basic laboratory studies, an ECG, and a chest x-ray examination
B) Reexamining the child in 1 month
C) Observing and monitoring the innocent (Stills) murmur at subsequent visits
D) Consulting with the supervising physician, and then referring to a cardiologist
d

A 16-year-old adolescent is brought to the emergency room with bradycardia, miosis, depressed respiration, and stupor; the adolescent does not respond to verbal stimulation. On examination, possible needle tracks are noted between the toes. The PNP suspects a heroin overdose and after consulting with a physician begins treatment with:
A) Clonidine
B) Dopamine
C) Methadone
D) Naloxone
d

A parent brings a 1-week-old neonate into the clinic. The mother is concerned that the infant’s testicles do not seem “even.” The PNP notes that the cremasteric reflex is absent and the right testicle cannot be palpated in the scrotum. The PNP should:
A) Not allow the neonate to be discharged from the hospital
B) Order a sonogram
C) Consult with the physician
D) Refer the neonate to the urology clinic
c

A 10-year-old child has been referred for evaluation of scoliosis, which was identified in a school screening program. On physical examination the PNP finds a curvature of the spine when it is anteriorly flexed from the trunk. There is slight shoulder asymmetry, with prominence of the scapula. The curvature is at 10 degrees. The PNP should:
A) Refer the child to an orthopedist for further evaluation
B) Inform the parents that the child is going through a growth spurt and schedule an appointment for further evaluation in 1 year
C) Consult with the clinic physician
D) Order x-ray films of the spine
c

A father brings his 9-month-old because the infant has been tugging on the right ear. While examining the ear, the PNP notes that the infant has cranial and facial asymmetry. There is no evidence of otitis media. The PNP should then:
A) Refer the child to a neurologist for consultation
B) Perform an extensive developmental assessment
C) Continue to observe the infant, and recheck cranial shape at the 1-year well child visit
D) Consult with a physician and obtain a cranial radiologic film, CT scan of the skull, or both
d

A 9-year-old child is brought to the clinic for the evaluation and treatment of a rash. While examining the child, the PNP detects a speech dysfluency. The most appropriate plan of treatment should include:
A) Treating the rash and scheduling a well child visit to address the speech problem
B) Treating the rash and making no referral because it is too late for speech therapy to be beneficial
C) Using a clinical screening tool to determine whether a language disorder exists
D) Treating the rash and making a referral for speech, language, and hearing evaluation with a speech pathologist
d

A 12-month-old child is brought to the clinic for a repeat urinalysis and culture. Results of the urine culture obtained at the 12-month well-child visit were abnormal. There is no previous history of UTI and no fever of unknown origin. The PNP finds that the child is not circumcised and the foreskin does not retract with moderate traction. The parents state that they have not routinely retracted and cleaned under the foreskin. The most appropriate treatment would be to:
A) Refer to a urologist for evaluation as soon as possible
B) Obtain another clean catch urine specimen, and repeat urine culture
C) Forcibly retract the foreskin, obtain a catheterized urine specimen, clean and lubricate the glans penis, and then protract the foreskin
D) Because the child has no previous history of UTI, instruct the parents on foreskin care and perform repeat urinalysis in 6 months
a

A 1-year-old child is brought to the clinic with a temperature of 102ºF and left flank pain. The urine dipstick test indicates nitrates and leukocyte esterase. The presenting signs and symptoms suggest left pyelonephritis. Based on the data, the most appropriate action for the PNP is to:
A) Send urine for culture and sensitivity testing
B) Provide symptomatic treatment for 24 hours, and repeat the urine dipstick test
C) Refer the child to a urologist for diagnosis and treatment
D) Consult with a physician
d

An 18-month-old child is brought to the clinic with a rash and temperature of 103ºF for the past 5 days. The child has bilateral conjunctivitis; dry, cracked lips; and strawberry-like tongue. Cervical lymph adenopathy and a heart murmur are noted on physical examination. The PNP should:
A) Order a CBC
B) Order an ECG
C) Refer the child to the infectious disease clinic
D) Consult with the physician
d

An 18-month-old child is being evaluated in the clinic for falling with increasing frequency and loss of speaking ability. The PNP considers:
A) Urine amino acid screening
B) Referral to a physical therapist
C) Referral to a pediatric neurologist and MRI of the brain
D) Chromosomal studies for Tay-Sachs disease
c

The PNP is in the nursery to examine a neonate. The results of the neonatal screen are noted in the chart. The T4 level is low, and the TSH level is elevated. The PNP should:
A) Consult with the physician, and repeat the tests
B) Discharge the neonate, and schedule a follow-up visit in the office in 1 week
C) Examine the neonate for signs of hypothyroidism
D) Begin the neonate on thyroxine based on the test results
a

A newly delivered mother wants to breastfeed but admits to using marijuana occasionally. She reports last using marijuana over 6 weeks ago. Her drug toxicology screen is positive for marijuana. What would the PNP advise regarding breastfeeding?
A) Discourage the mother from using marijuana and recommend not breastfeeding until she stops using marijuana and her toxicology screen is negative
B) Recommend breastfeeding
C) Tell the mother she is not a good candidate for breastfeeding because her toxicology screen indicates recent use of marijuana
D) Discourage the mother from using marijuana, but if she does, instruct her to pump and discard her breast milk for at least 24 hours before resuming breastfeeding
d

The PNP is counseling the parents of a child with recurrent episodes of acute otitis media. The PNP informs the parents of the following environmental risk factors for acute otitis media:
A) Not wearing a hat when it is cold outside and going to sleep with a bottle
B) Swimming in pools without ear plugs and attending group day care
C) Passive exposure to smoke and breastfeeding longer than 6 months
D) Attending group day care and passive exposure to smoke
d

The father of a 10-month-old is concerned because over the lack of eruption of primary teeth in the infant. The PNP suggests:
A) This is within the range of normal dental development
B) Consulting the pediatrician at the 15-month-old well-child visit
C) The infant’s teeth will erupt on a similar schedule as the parents
D) An immediate referral to the dentist
a

A mother brings a 4-year-old child to the clinic for a well-child visit. She is concerned that the child has autism. Atypical behavior traits that appear in autistic children include:
A) An intense relationship with the mother during the first year of life
B) Obsession with repetitive hand cleansing
C) Onset of behaviors noted after the third year of life
D) Impairment in reciprocal social interaction
d

The PNP sees a 14-year-old adolescent in the clinic. The adolescent is sexually active and does not want to become pregnant. There is a history of high blood pressure and cardiac problems in the family. The patient states that she does use condoms to prevent STDs. The patient asks the PNP what the best form of birth control to use to prevent pregnancy?
A) Diaphragm
B) Depo-Provera
C) Ovral
D) Intrauterine device (IUD)
b

The PNP is discussing laser therapy for a capillary vascular malformation with the parents of a child and advises the parents:
A) “Multiple treatments over many months may be required.”
B) “Transient edema can be expected to last several months.”
C) “Gray-black discoloration will fade in 3 to 4 days.”
D) “The birthmark will gradually fade over 1 to 2 years.”
a

A 16-year-old adolescent comes to the clinic because of a latex allergy. The adolescent is concerned about sexually transmitted diseases (STDs) and using condoms. He states that he and a girlfriend have been “experimenting.” The PNP suggests that:
A) The girlfriend should use a cervical cap
B) The girlfriend should use a diaphragm
C) The adolescent boy should use a double condom (i.e., a non-latex condom covered by a latex condom)
D) The girlfriend should take oral contraceptives
c

A 2-month-old breastfed infant weighing 11 lb 8 oz (birth weight 8 lb) is seen in the clinic. The mother appears exhausted and reports that she is nursing on demand, every hour for 5 to 10 minutes. The PNP suggests that the mother:
A) Supplement the infant with formula after breastfeeding
B) Nurse longer than 10 minutes at each feeding
C) Give 1 tablespoon of rice cereal once a day to the infant
D) Continue nursing on demand
b

A parent is concerned her young child may have food allergies. “They run in my husband’s family.” The parent would like to know the most common food allergies. The PNP responds:
A) Dairy products, nuts, eggs
B) Soy milk, bananas, nuts
C) Fish, wheat cereal, eggs
D) Beef, oat cereal, wheat cereal
a

While obtaining a family history, a mother tells the PNP that her maternal grandfather, her mother, herself, her son, and her daughter have von Willebrand disease. The PNP describes this inheritance pattern as:
A) Mosaic
B) Autosomal recessive
C) Autosomal dominant
D) X-linked
c

A grandfather brings his 8-month-old grandson to the clinic and states that the infant screams and cries whenever anyone besides the grandfather comes near. The PNP diagnoses stranger anxiety. The grandfather wants to know if he should be concerned. The PNP responds:
A) Force the infant to be held by others
B) Reassure the parents this is normal and will pass
C) Ignore the screaming
D) This is not normal, and a referral to a child psychologist may be helpful
b

A 17-year-old adolescent who is at the clinic for a “football physical” relates to the PNP that he and a girlfriend have had sex two times this past month. The adolescent is concerned about preventing pregnancy and protection from STDs. This was his first sexual experience, and he has questions about condoms but does not want his parents to know that he is sexually active. The PNP offers the following information:
A) “Use natural condoms because they are easier to apply.”
B) “Use a petroleum-based lubricant when applying a condom.”
C) “Use a latex condom with spermicide and lubricant.”
D) “Condoms have a long half-life, so once purchased, they last for over 2 years.”
c

A 12-month-old child is brought to the clinic. On physical examination the PNP notes that the child’s right eye drifts to the nasal side of the eye. The PNP explains to the mother that a complication of this condition may be:
A) Strabismus
B) Astigmatism
C) Amblyopia
D) Myopia
c

Which of the following explanations can help parents of preschool-aged children distinguish night terrors from nightmares?
A) Nightmares are scary dreams followed by complete awakening
B) The child does remember night terrors and talks about them in the morning
C) Night terrors usually occur during the second half of the night
D) The child does not remember nightmares in the morning
a

The mother of a 2-year-old child is concerned about the child’s fear of the dark. The PNP responds that:
A) Parents should not allow the child to withdraw from a fearful situation
B) Fears that interfere with social development resolve as the child develops
C) Young children with fears tend to have phobias as adults
D) Fears of this type are a normal developmental process
d

A parent asks at what age separation anxiety is most often expected. You explain to the parent that separation anxiety usually occurs at:
A) 12 to 18 months of age with peaking at 24 to 26 months
B) 9 to 15 months of age with peaking at 20 to 22 months
C) 3 to 9 months of age with peaking at 18 to 20 months
D) 1 to 6 months of age with peaking at 9 to 12 months
c

What suggestions would the PNP offer a father calling because his 3-month-old infant is passing hard stools?
A) “Add honey to a 4-oz bottle of water.”
B) “Time will take care of the problem.”
C) “Offer the infant a little extra water with added pasteurized apple or prune juice.”
D) “Hold the infant upright so that gravity can help the stool pass.”
c

An 18-month-old child is brought to the office for irritability, nightmares, and generally being tired for the past few nights. The child has had an upper respiratory tract infection for which the grandmother has administered an over-the-counter medication for rhinorrhea. The child is afebrile and alert, with obvious clear rhinorrhea. The PNP:
A) Orders sinus radiologic studies
B) Prescribes amoxicillin
C) Suggests that the mother administer acetaminophen
D) Discusses side effects of decongestants and antihistamines
d

The PNP preceptor is discussing with the PNP student the use of an aminoglycoside in the treatment of an infant in a special care nursery. The use of an aminoglycoside may result in the following complication:
A) Permanent liver damage
B) Ototoxicity and nephrotoxicity
C) Bone marrow depression (if not properly monitored)
D) Encephalopathy
b

The mother of a 3.5-year-old child is counseled about the need to maintain a therapeutic level of an antibiotic prescribed to treat otitis media. The PNP informs the mother that the term therapeutic level refers to the:
A) Concentration of a drug that is required to elicit the desired clinical response without causing toxic effects
B) Lowest concentration of the drug reached between doses
C) Amount of drug taken into the body, which is equal to the amount excreted
D) Time required for 50% of a dose to be excreted
a

A mother calls the office because her 2.5-year-old has a fever of 101°F (38.2°C). The child is playing and taking plenty of fluids. The PNP advises the mother:
A) To dress the child warmly to prevent chilling
B) An appointment is needed to determine the cause of the fever
C) A fever can actually be a good thing as long as the child is comfortable
D) To treat the fever with acetaminophen or ibuprofen if it is greater than 100.1°F
c

An infant with congenital hypothyroidism is to start treatment with Levothyroxine. When describing the administration of the medication to the parents, it is important to reinforce:
A) Evening dosing is preferable to morning dosing
B) Cow’s milk formula interferes with absorption
C) Dosage must be monitored and increased significantly as the child grows
D) A crushed tablet should be administered to the infant each morning
c

An 8-year-old child is evaluated for “growing pains.” The pains are intermittent and last up to 2 hours. The child awakens at night, and the pain is not specific to joints. The physical examination is normal. The PNP decides to order:
A) A monospot test
B) A test to determine ESR
C) A throat culture
D) An x-ray film of the left hand
b

A 5-year-old child has a 3-day history of sore throat, fever (101ºF to 103ºF), headache, and one episode of vomiting. The child’s symptoms started with a runny nose and slight cough yesterday. The physical examination is normal except for +2 to +3 tonsils, palatal petechiae, erythema, and a clear runny nose. The PNP’s best course of action is to:
A) Perform a throat culture before prescribing an antibiotic
B) Discuss symptom control for seasonal allergies
C) Prescribe antibiotics for strep throat because the child has classic symptoms of the infection
D) Reassure the parent that only symptomatic treatment is necessary and that the virus needs to run its course
a

A 2-year-old child is brought to the clinic with periumbilical pain and vomiting that began this morning. The examination is unremarkable with the exception of tenderness noted on abdominal palpation. The differential diagnosis includes appendicitis. What should the PNP do next?
A) Order an abdominal x-ray examination, and instruct the parents not to feed the child
B) Order ultrasonography, and instruct the parents to feed the child only clear liquids
C) Advise “watchful waiting,” and instruct the parents regarding the signs and symptoms that indicate a more serious situation
D) Order a CBC and differential count, and send the child home with instructions to return if there is a change in symptoms
c

A 15-year-old adolescent is brought to the clinic complaining of chronic nasal congestion and fatigue. The father gave the adolescent an antihistamine that did not help with the congestion. Upon examination a small perforation is noted in the nasal septum. The PNP suspects:
A) Foreign body perforation
B) Allergic rhinitis
C) Chronic use of cocaine
D) Nasal septal defect
c

A 2-month-old infant is brought to the clinic because of continued vomiting. Because the PNP suspects GER, time has been arranged for the visit to include a feeding. During the feeding the PNP observes the infant for:
A) Calm, attentive feeding position
B) Head moving back and forth and searching
C) Disinterest in the bottle
D) Back arching and crying
d

The PNP examines a 12-year-old child with a dry, unproductive cough; a low-grade fever; and copious amounts of sputum. On physical examination, coarse breath sounds and moist rales are heard in the upper part of the lungs. The PNP makes the diagnosis of bronchitis and treats with:
A) A decongestant and ibuprofen
B) Amoxicillin and a cough suppressant
C) An albuterol inhaler and acetaminophen
D) An antihistamine; a cough suppressant, such as dextromethorphan hydrobromide; and use of a vaporizer
b

A foster mother phones the office because her 10-year-old child has knocked out a permanent tooth. The PNP instructs the foster mother to:
A) Rinse the tooth in warm water, and call the dentist
B) Place the tooth back in the socket, and call the dentist immediately
C) Take the child and the tooth to the dentist tomorrow
D) Take the child to the hospital emergency room immediately
b

Which of the following is a true statement regarding nocturnal enuresis?
A) Psychologic stressors are a common cause of nocturnal enuresis
B) There is a true organic cause in most cases of nocturnal enuresis
C) Physiologic causes of enuresis include having a small bladder capacity
D) Urinalysis is unnecessary in most cases of enuresis
c

The most appropriate laboratory tests to obtain for evaluation of a neonate with jaundice presenting in the first 36 hours of life include:
A) G6PD assay and an Rh type test
B) Direct Coombs’ test and G6PD assay
C) Total and direct serum bilirubin levels and a reticulocyte count
D) Maternal and infant blood group and Rh type tests and a direct Coombs’ test
d

A 17-year-old adolescent comes to the clinic because of weight loss, irritability, insatiable hunger, and heat intolerance. The PNP screens for hyperthyroidism. In the patient with hyperthyroidism:
A) The TSH level is normal, and the T4 level is elevated.
B) The TSH and the T4 levels are elevated.
C) The TSH level is normal, and the T4 level is below normal.
D) The TSH level is below normal, and the T4 level is elevated.
d

The PNP suspects that a sexually active adolescent has pityriasis rosea. Based on the differential diagnosis, the PNP screens for:
A) Chlamydia
B) Secondary syphilis
C) Erysipelas
D) Molluscum contagiosum
b

A 10-year-old child is brought to the clinic for what appear to be nongenital warts on the hand. The PNP treats the warts with:
A) Referral to a dermatologist for laser treatment of the lesions
B) Cryosurgery and referral to a dermatologist
C) Topical salicylic acid applied twice a day for several weeks
D) Podophyllum resin
c

The most important aspect of bruising in relation to possible maltreatment or nonaccidental trauma is:
A) Dating of the bruise
B) The history of the cause of the bruise
C) The location of the bruise
D) The size of the bruise
b

The PNP examines a 17-year-old adolescent with fatigue, joint stiffness in the morning, anorexia, weight loss, and a rash across the nose. The rash is bleeding into the cheeks. The PNP suspects systemic lupus erythematosus and orders:
A) An ANA
B) Electrophoresis
C) An ESR
D) A CBC and differential count
a

The PNP sees a 7-year-old child for “an ear recheck.” Clinical findings include tympanic membranes that are opaque bilaterally, with visible landmarks and light reflexes. A fluid line is visible on examination. The diagnosis of otitis media with effusion is made. Of the following treatment plans, which is the most appropriate?
A) Amoxicillin 60 mg/kg three times a day for 10 days (erythromycin if the child is allergic to penicillin)
B) Prednisone 10 mg/kg/day for 7 days
C) ENT referral
D) Observation and recheck in 4 to 6 weeks
d

RSV affects infants and young children and presents with:
A) Sore throat, rash, and cough
B) Apnea spells, poor feeding, and wheezing
C) Sore throat, wheezing, and poor feeding
D) Rash, lymphadenopathy, and cough
b

A 17-year-old adolescent comes to the school-based clinic because of swelling, pain, and tenderness involving the Achilles tendon. The adolescent has been running long distances several times a week. The PNP diagnoses tendonitis and recommends rest, elevation, and:
A) Heat applied to the area of the Achilles tendon
B) Ice applied to the area of the Achilles tendon
C) Steroids
D) Ibuprofen
d

A 7-year-old is brought to the clinic with “a cold that just won’t go away” and green, thick rhinorrhea of 10 days’ duration. The PNP diagnoses sinusitis and treats with:
A) Amoxicillin and a decongestant
B) Amoxicillin and an antihistamine
C) Saline nasal drops and a decongestant
D) A topical nasal steroid and cefixime (Suprax)
a

The PNP is evaluating a 10-year-old child with a 12-day history of rash and intermittent fever. The child is alert and febrile (100.8°F) and has an annular, indurated lesion measuring approximately 16 to 20 cm on the posterior lateral thigh. There also are two small papules with 3 to 4 cm of surrounding erythema on the posterior thorax. The child reports feeling tired and “achy,” primarily in the legs, but has been pursuing normal activities. The mother reports no known injuries or insect bites. An appropriate approach to treatment should include:
A) Admission to the hospital for parenteral ceftriaxone
B) Administration of amoxicillin, doxycycline, or erythromycin at the appropriate dose for 14 to 21 days; administration of antipyretics or analgesics; and observation
C) Dermatology referral for skin biopsy of the lesion
D) Application of warm compresses, administration of antipyretics, and close observation
b

A 5-year-old child is brought to the clinic because of a “runny nose.” The rhinorrhea is clear, and on physical examination the PNP notes that the nasal mucosa is pale, boggy, and edematous. The PNP prescribes:
A) Amoxicillin
B) Loratadine (Claritin)
C) Erythromycin
D) Fexofenadine (Allegra)
b

At an office visit, the mother of a 15-year-old basketball player reports that the adolescent arrived home from a postgame party and was euphoric, hostile, and hyperactive. The PNP suspects use of:
A) Glue
B) Marijuana
C) Alcohol
D) Amphetamines
d

The PNP is examining a full-term neonate with jaundice at age 15 hours. The bilirubin level is 13 mg/L. The mother tested positive for group B streptococcus, but antibiotics were not administered during labor because there was not enough time. There were no other complications during birth. The infant is breastfeeding well. Currently the only other significant finding on the infant’s examination is a large cephalohematoma. The least likely cause of jaundice is:
A) Sepsis
B) Rh or ABO incompatibility
C) Physiologic jaundice
D) Hemolysis
c

A 5-year-old child is brought to the clinic for drop-foot gait. The child lifts the leg to avoid tripping over the drooping distal foot and toes. The PNP suspects that the child has:
A) Duchenne muscular dystrophy
B) Chronic sensorimotor neuropathy
C) Myasthenia gravis
D) Becker muscular dystrophy
b

The PNP is examining a 16-year-old adolescent with painful vesicular skin lesions. The most likely diagnosis is:
A) Molluscum contagiosum
B) Herpes infection
C) Coxsackievirus infection
D) Impetigo
b

During a routine sports physical examination the PNP asks a 13-year-old adolescent whose parents smoke about tobacco use. Which of the following is the best rationale for asking if the adolescent smokes?
A) Smoking will inhibit athletic performance
B) Children whose parents smoke are less likely to begin to use tobacco
C) The average age at which tobacco use begins is 14 years
D) The incidence of nicotine dependence is higher in boys
c

A 14-year-old adolescent is brought to the clinic for primary dysmenorrhea. The PNP prescribes:
A) Ibuprofen
B) Acetaminophen and codeine
C) Codeine
D) Acetaminophen
a

When considering child abuse or maltreatment, which of the following statements is correct?
A) The most common reporters of abuse are primary care providers
B) The most common perpetrator of physical abuse is the father
C) Abusive deaths are most often caused by biological family members
D) Disabled children are safe from abuse and neglect
c

An 8-year-old child is in the clinic because of an inability to lift the right arm. After playing on the rock wall in the playground, the child complained to the mother of arm pain and inability to lift the arm. On examination the PNP notes decreased voluntary control of the right arm, specifically with an inability to lift the arm. There is no obvious deformity of either arm, and the child has full range of motion in the neck and other limbs. The only abnormal finding is slight tenderness in the right shoulder. The clinical presentation is consistent with the diagnosis of:
A) Pinched cervical nerve and pseudoparalysis
B) Clavicular fracture and pseudoparalysis
C) Fracture of the right humerus
D) Muscle strain and concurrent weakness
b

Which of the following statements is true regarding childhood obesity?
A) The most common problem associated with childhood obesity is diabetes
B) If one parent is obese and the other is of normal size, 9% of their children will become obese
C) Endocrine causes of obesity are common
D) The older the obese child is, the more likely it is that he or she will become an obese adult
d

A 9-year-old child is brought to the clinic for joint pain, a rash on the trunk, and a “racing heart.” The child looks ill, and the skin is hot to the touch. The child had a sore throat about 3 weeks before this illness but recovered without medical treatment. The PNP orders a:
A) Immunofluorescent stain to detect ANA antibodies.
B) Antistreptolysin O (ASO) titer
C) Monospot test
D) CBC
b

When approaching adolescents about sensitive issues, such as drug use and sex, an effective interviewing approach is to:
A) Introduce the topics early in the history
B) Begin by asking about their friends’ activities
C) Let the adolescent know that the parents are concerned
D) Avoid promising confidentiality
b

A 17-year-old adolescent attended a party and returned home. The grandmother calls the office and is concerned that the adolescent’s eyes “look funny.” They are puffy, the pupils are slow to react or nonreactive, and tearing is present. With these presenting signs and symptoms the PNP suspects use of:
A) Cocaine
B) Phencyclidine hydrochloride (PCP)
C) Alcohol
D) Marijuana
d

A 3-year-old child is brought to the clinic with the complaint of vomiting; low-grade fever; and watery, frequent stools for the past 24 hours. The PNP recommends:
A) Increased liquids and a modified diet of bananas, rice, applesauce, and toast
B) A diet of bananas, rice, applesauce, toast, and milk
C) Oral antidiarrheal agents
D) Clear fluids and rest for 24 hours
a

A 3-year-old child is brought to the clinic with painful sores in the mouth. On inspection the PNP notes two to five pin-sized vesicles covered with a yellow-gray membrane on the child’s lips and in the mouth. The PNP diagnoses:
A) Epstein-Barr virus
B) GABHS infection
C) Aphthous stomatitis
D) Herpangina
c

An 18-month-old child is examined because of a cough and runny nose lasting 7 days. The nasal discharge is yellow-green. The mother has been using over-the-counter cold medications every 6 hours. There is no fever, and the child’s appetite, activity, and elimination are as usual. The mother had the same symptoms and was put on an antibiotic for a sinus infection. She is concerned that the child has the same infection. The most appropriate step for the PNP to take is to:
A) Prescribe an appropriate antibiotic to treat the sinus infection
B) Order a Water’s view sinus x-ray film to rule out acute sinusitis
C) Reassure the mother that the child probably has a viral infection, and provide instructions regarding supportive care
D) Inform the mother that no treatment is necessary, but suggest that she bring the child in if symptoms do not improve within 4 weeks
c

A 3-year-old child is brought to the office for the first time. The child is adopted, and little is known about the prenatal and birth history. A diagnosis of fetal alcohol effects (FAE) is suspected. The physical findings that lead the PNP to suspect this diagnosis are:
A) Carious teeth, hyperactivity, toe walking, and macrocephaly
B) Growth retardation, thin or wide lips, flat midface, and finger anomalies
C) Abnormal hair pattern, cherubic lips, temper tantrums, and protuberant belly
D) Balance problems, irritability, multiple scars on the arms and legs, and microcephaly
b

Appropriate interventions for a 3-month-old infant diagnosed with sickle cell anemia include the following:
A) Initiate penicillin prophylaxis, and begin folic acid supplementation
B) Teach spleen palpation to parents, and administer Pneumovax vaccine
C) Administer Pneumovax vaccine, and initiate penicillin prophylaxis
D) Begin folic acid supplementation, and administer Pneumovax vaccine
a

A 10-year-old child is brought to the office because of persistent fever (102ºF) lasting for 5 days. The child complains of a headache and sore throat and was seen twice by different providers in the past 5 days. An earlier rapid antigen-detection test for GABHS was negative, and a viral infection was diagnosed. Physical examination now reveals slightly enlarged anterior cervical lymph nodes and a red throat. A faint, pink, slightly raised rash is present on the abdomen and in the groin area; the remainder of the physical is unremarkable. The most appropriate action is to:
A) Treat with penicillin for probable GABHS without performing a throat culture
B) Obtain a throat culture
C) Order a monospot test
D) Reassure the parents and the child that the symptoms are caused by a viral infection and that symptomatic care is all that is needed
b

The PNP examines a 4-month-old infant with hypotonia and motor delay since birth. The most significant finding indicating lower motor neuron involvement in this infant would be:
A) Lack of deep tendon reflexes
B) Hyperactive deep tendon reflexes
C) Persistence of a Moro reflex
D) Persistence of the tonic neck reflex
a

The PNP is evaluating a 2-year-old child with a 48-hour history of a dry, coarse cough and fever. Upon examination the child appears ill but nontoxic. The child has a harsh, forceful cough with intermittent stridorous breaths after a coughing spasm but no stridor at rest. The parents have been giving the child an expectorant and cough suppressant that have provided little relief. The PNP makes the diagnosis of croup without respiratory distress. Treatment for this child should include:
A) Nebulized albuterol and expectorants
B) Narcotic cough suppressant as needed, fever management, mist therapy, and rest
C) Antibiotic therapy combined with steroids and an expectorant
D) Continued use of an expectorant if helpful, oral steroids, mist therapy, and rest
d

Appropriate management of a full-term, 3-day-old, breastfed infant with a bilirubin level of 16 mg/L would consist of:
A) Observation, close follow-up, parental instructions to breastfeed every 2 hours, and an office visit to determine serum bilirubin level in 10 days
B) Exposure of the infant to indirect sunlight, supplementation with formula after breastfeeding if the mother’s milk has not yet come in, and close clinical follow-up
C) Phototherapy and follow-up in the office 10 days later to determine serum bilirubin level
D) Exchange transfusion and then hospital discharge with close clinical follow-up
b

A 4-year-old child is brought to the clinic with ulcers on the tongue and oral mucosa. There are vesicles on the palms, soles, and interdigital areas. The PNP diagnoses:
A) Infectious mononucleosis
B) Hand-foot-and-mouth disease
C) Rubeola
D) Herpangina
b

The parents of a 6-month-old infant report that the infant is “different” from their previous child. Which of the following behaviors would alert the PNP to a diagnosis of autism? The infant:
A) Is extremely passive, with little interaction with others or the environment
B) Is cuddly only when held by the mother
C) Initiates eye contact only with siblings
D) Seems to have normal language skills
a

The PNP is evaluating an 8-year-old child with an eye injury. About 1 hour ago the child was hit in the eye with a foreign body. When the eye is stained with fluorescein, a vertical linear abrasion is visible. The pupils are equal in size and reactive, and a positive red reflex is noted bilaterally. There are no abnormal extraocular movements. No foreign body is visible. An appropriate plan of care for this patient would include the following:
A) Instill antibiotic drops, and apply patches over both eyes
B) Close both eyes, and apply eye patches
C) Apply antibiotic ointment, but do not apply an eye patch
D) Instill ophthalmic steroid drops three times a day, but do not apply an eye patch
c

A dark-skinned 14-year-old adolescent comes to the school-based clinic with symptoms of a nonpruritic rash, with loss of pigment. The PNP diagnoses tinea versicolor and treats with:
A) Steroid cream and erythromycin
B) Selenium sulfide shampoo and systemic ketoconazole
C) Systemic ketoconazole and steroid cream
D) Silver sulfadiazine cream and an antihistamine
b

A 10-year-old child is examined in the school-based clinic for unilateral chest pain that radiates to the back and abdomen. On physical examination, tenderness over the costochondral junction is noted. The PNP diagnoses that the chest pain is caused by:
A) Costochondritis
B) Gastric reflux
C) Skeletal deformity of rickets
D) A fractured rib
a

The PNP is examining a 2-year-old child with a history of vomiting, diarrhea, and fever for the past 24 hours. The physical examination reveals a tender abdomen. The PNP should:
A) Reassure the parents that the child has gastroenteritis
B) Obtain a more definitive history and physical examination
C) Send a stool specimen for culture of ova and parasites
D) Treat with clear fluids, and call the parents in 24 hours
b

A 9-year-old child has circumscribed, coalescent, generalized, erythematous raised lesions of various sizes. The PNP recognizes that the most common cause of hives is:
A) Ingestion of foods and food additives
B) Cold exposure
C) Sun exposure
D) Insect bites
a

A 14-year-old adolescent has a second-degree burn along the hairline from using a curling iron. The lesions are erythematous and blistering. The adolescent complains of pain. The PNP advises:
A) Icing the involved area to decrease pain and beginning a prophylactic antibiotic
B) Applying silver sulfadiazine cream and icing the involved area to decrease pain
C) Taking ibuprofen for pain and applying silver sulfadiazine cream
D) Taking prophylactic antibiotics and ibuprofen for pain
c

A 10-month-old infant is brought to the clinic because of an acute illness. On examination the PNP notes a large, bulging umbilical mass. The mass is easily reducible although prominent when the child cries. The PNP:
A) Discusses with the parents the need to apply a bellyband to “hold in” the hernia
B) Refers the infant to a surgery clinic for evaluation of the hernia and treats the acute illness
C) Explains to the parents that surgical repair should be performed when the child is between age 2 and 3 years
D) Treats the acute illness and comments on the hernia and measures the abdominal opening
d

A 17-year-old basketball player has burning and itching between the toes on both feet. The PNP diagnosis tinea pedis and treats with:
A) Griseofulvin given orally (and suggests wearing sandals in the shower)
B) Permethrin cream (and suggests wearing sandals in the shower)
C) Clotrimazole cream (and suggests wearing cotton socks)
D) Steroid cream applied to the lesions (and suggests wearing cotton socks)
c

The PNP sees a preschool-aged child with symptoms of periumbilical pain, nausea, abdominal distension, and diarrhea. A microscopic examination of the stool reveals that Giardia organisms are the cause of the diarrhea. The PNP treats with:
A) Erythromycin/sulfisoxazole acetyl (Pediazole)
B) Metronidazole
C) Fluconazole (Diflucan)
D) Sulfamethoxazole
b

A 14-year-old adolescent has ataxia, muscle weakness, nystagmus, disorientation, and dizziness. The mother is concerned that the adolescent has been inhaling correction fluid (e.g., Wite-Out). The most likely consequence of long-term inhalant use is:
A) Cardiac dysrhythmia
B) Cerebral hemorrhage
C) Encephalopathy
D) Pneumothorax
c

A teacher brings a 14-year-old adolescent to the school-based clinic. The adolescent has dilated pupils, diaphoresis, and severe headache; cannot concentrate; and is displaying paniclike behavior. The adolescent denies drug or alcohol use. The PNP orders a:
A) Urine screen for drugs
B) Test to determine blood glucose level
C) Test to determine blood alcohol level
D) Urine screen for drugs and radioimmunoassay for lysergic acid diethylamide (LSD)
d

The PNP is evaluating a 15-year-old adolescent with acne. Upon examination the adolescent has papules and a few pustules on the face, chest, and back. An appropriate plan of care for this patient would include:
A) A combination topical antibiotic and exfoliating agent
B) Oral tetracycline and anticipatory guidance for skin care
C) Oral isotretinoin and pregnancy prevention counseling
D) A topical exfoliating agent
a

The PNP examines a fair-skinned 17-year-old adolescent. The adolescent had a maculopapular, red, scaling rash on the trunk a few weeks ago and now has five scaling lesions that follow a line of cleavage confined to the trunk. The PNP diagnoses:
A) Eczema
B) Tinea versicolor
C) Pityriasis rosea
D) Contact dermatitis
c

A 16-year-old adolescent comes to the school-based clinic with chest pain. The most common cause of cardiac pain in children and adolescents is:
A) Tachycardia
B) Mitral valve prolapse
C) Myocarditis
D) Preventricular contractions
b

A single, 15-year-old mother, who attends high school, brings a 3-month-old infant to the clinic. The infant seems to be healthy and thriving, but the mother is vague when reporting the infant’s feeding and sleeping habits. To get more accurate information about the specifics of the infant’s routine and care, the PNP should:
A) Ask about the day care arrangements
B) Make a referral to a social service agency
C) Ask the mother if she has a learning disability
D) Make a referral to a home nursing agency
a

A 12-year-old child reveals to the PNP during a visit for acne that she may be pregnant. In this community the pregnancy rate among females aged 15 to 19 years is 3 in 100. What is the most important area to explore?
A) The source of prenatal care
B) The expected date of confinement
C) The possibility of sexual abuse
D) Plans for telling her family
c

The PNP is evaluating a 9-year-old child with a 2-week history of heel pain. The onset of pain coincides with the start of football practice. The child does not limp but experiences heel pain with weight bearing and plantar flexion. Laboratory findings rule out an infectious or inflammatory disorder. This presentation is consistent with a diagnosis of:
A) Osteochondritis
B) Osgood-Schlatter disease
C) Legg-Calvé-Perthes disease
D) Freiberg’s disease
a

A 2-year-old child is examined as part of a well-child visit. The mother’s main concern is bowing of the child’s legs, which is evident on examination. The PNP:
A) Assures the mother that this is normal development
B) Refers the child to an orthopedic clinic for evaluation
C) Obtains a detailed family history for genu varum
D) Suggests that the child may need bracing
b

In counseling a parent regarding possible lead poisoning in the child, the PNP states that the most common source of lead is:
A) Soil
B) Batteries
C) Lead-based paint
D) Drinking water
c

The PNP is examining a 2-month-old infant. At delivery, the infant’s cord was wrapped around the neck three times. The parents are concerned that the infant is “floppy.” On physical examination, the PNP notes that the deep tendon reflexes are increased, there is poor tone, and scissoring of the legs is marked. The PNP:
A) Orders chromosomal studies
B) Discusses the findings with the staff physician
C) Orders a computed tomography (CT) scan
D) Orders routine vaccinations and schedules a return visit in 2 months
b

The PNP is performing well-child examinations on 2-year-old twins. One twin is verbal and interactive, initiating play with the PNP. The other twin is nonverbal, acts fearful, and refuses to make eye contact with the PNP. The mother reports that the second twin rarely speaks intelligible words. Growth parameters are normal for both children, and the physical examinations are normal. Intervention for this family should include:
A) Encouraging the mother to initiate and play verbal games with the twins that require verbal responses from both children
B) Scheduling a professional speech, language, and hearing evaluation for the second twin
C) Performing audiometry examinations to rule out hearing deficits in the second twin
D) Continued observation, rechecking in 6 months to evaluate language development in both twins, and reassuring the mother that development is different in each child
b

A mother brings her 2-year-old child to the clinic for a well-child visit. The mother’s chief complaint is the child’s rebellious behavior. The mother says that she has tried time-out, yelling, and spanking but nothing has worked. The child has never slept through the night and still gets a bottle in the middle of the night. In the examination room the PNP observes as the child repeatedly gets into the mother’s purse when she is not paying attention, despite the mother telling the child “no.” Based on the history and observation, the PNP suggests that:
A) The child shows signs of attention deficit-hyperactivity disorder and referral to a specialist may be necessary
B) The child is normal; this rebelliousness is only a phase and will improve with time
C) The child exhibits behavior atypical of 2-year-old children and immediate referral to a child psychologist is necessary
D) The child needs to have limits and boundaries set consistently and the mother could benefit from parenting classes
d

A neonate in the neonatal nursery has a spongy area over the parietal region of the skull. The PNP diagnoses craniotabes and:
A) Assesses for hydrocephalus
B) Assures the mother this is a normal variant
C) Obtains a skull x-ray film
D) Refers the neonate to a neurologist for evaluation
b

A 12-year-old child is seen in the office for a yearly physical. The boy states that he has recently noticed an enlargement of his testes and scrotum. When counseling the child about what to expect next in pubertal development, the PNP states that:
A) The penis will grow in length
B) Changes in voice will occur
C) The penis will grow in width
D) Facial hair will appear
a

A number of the PNP’s adolescent patients are using natural herbal remedies. What general advice should the PNP give these patients?
A) “Herbs are drugs even though they are marketed as being grown naturally.”
B) “Interactions between different types of herbal remedies are rare.”
C) “Herbs are safe because they are natural.”
D) “Let’s look for evidence-based resources to check out the herbal remedies that you are interested in trying.”
d

The mother of a 4-year-old who was adopted at birth asks the PNP when is the most appropriate time to tell the child that he is adopted. The PNP responds:
A) Do not offer the information until the child is developmentally ready at about age 10 years
B) When the mother feels the child is able to understand the concept of adoption
C) Wait until the child is in third grade and can share with other children his adoption story
D) Most children by the age of 3 years are old enough to understand the story of adoption
d

The PNP is examining a 6-month-old exclusively breastfed infant. The infant is growing appropriately. The mother would like to delay introducing solid foods until the infant is at least 12 months of age. The PNP tells the mother the nutrient most likely lacking if solid foods were to be delayed is:
A) Vitamin B6
B) Phosphorus
C) Iron
D) Calcium
c

In providing anticipatory guidance to the parents of a 5-year-old child with cerebral palsy, the PNP teaches the parents to be aware of abnormal eye movements, drooling, and shaking movements that cannot be stopped. These are all signs of:
A) Syncope
B) Seizures
C) Febrile seizures
D) Infantile spasms
b

A 16-year-old adolescent comes to the clinic for a yearly examination. During the discussion of breast self-examination (BSE), the PNP should inform the adolescent that:
A) Breast masses that are painless do not require further evaluation
B) Asymmetric breast development is common in puberty
C) The best time to perform a BSE is during menses
D) Palpation of the nipple area is not necessary
b

The grandmother of a 2-year-old child requests information regarding toilet training readiness. The PNP tells the grandmother that the most important factor in determining readiness to toilet train is that the child:
A) Is strong willed and shows an interest in the potty
B) Is able to communicate needs and follow directions
C) Is not bothered by wet or soiled pants
D) Wakes up from naps with soaked diapers
b

A 3-year-old child has recently started having breath-holding spells. The parents state that an older sibling “did the same thing” and ask for help. The PNP advises the mother that after a breath-holding event she should:
A) Do everything possible to keep the child from feeling any extreme emotions
B) Ignore the child’s behavior similar to ignoring temper tantrums
C) Throw water on the child’s face to stimulate breathing
D) Stand and watch but do nothing while the breath-holding occurs
b

The PNP is completing a preadmission college physical for a student entering college in the fall. The PNP discusses with the student the need for a:
A) Hepatitis A vaccine
B) Meningococcal vaccine
C) Lyme disease vaccine
D) DT vaccine
b

A new mother requests that her infant be placed on low-iron formula. She has heard that regular formula causes constipation. The PNP discusses with the mother:
A) Changing to low-iron formula and using iron drops as a supplement
B) Changing to a soy-based formula, which is low in iron and more easily digested
C) The need for the formula to provide adequate iron for normal growth and development
D) The importance of not listening to what people who are not professionals say about how to feed and care for the infant
c

A 5-year-old who occasionally reverses letters without any other suggestion of learning problems:
A) Should be referred to a neurologist immediately
B) May need special education classes for dyslexia
C) Will probably grow out of this but should be monitored for other signs of difficulty
D) Is likely to have school problems with reading and writing
c

A 14-year-old adolescent comes to the school-based clinic with PMS. The PNP advises the adolescent to:
A) Limit intake of chocolate, fats, and sodium and increase exercise
B) Limit intake of sugar and fat and decrease exercise
C) Limit intake of fats and sodium and decrease exercise
D) Limit intake of sugars, fats, and sodium and participate in aerobic exercise
d

A 5-year-old child is at the clinic for a routine well-child visit before beginning kindergarten. The mother reports the child’s list of chores as follows: clean own room, make own bed, wash and dry own clothing, take out the trash, and help wash the dishes. If the chores are not completed, the child is grounded from other activities, such as playing with friends or watching television. The mother states that the child frequently must be grounded. How should the PNP respond?
A) Suggest use of natural consequences (e.g., having no clean clothes to wear if the laundry does not get done) as an additional way to discipline
B) Tell the mother she is expecting too much from the child and should reduce the number of chores
C) Perform a complete physical assessment of the child, as well as a parenting assessment
D) Tell the mother to make a chart so the child can remember to do the chores and record their completion and to continue grounding the child from activities when necessary
c

The mother of a 4-year-old child tells the PNP that her son “plays with himself in front of the TV.” The PNP advises the mother to:
A) Explain to the child that this is normal behavior, but if it continues, he will have to see a mental health professional
B) Send the child to his room with no comment; the PNP should explain to the mother that this is normal behavior
C) Explain to the child that masturbation is normal but should be done in private
D) Ignore the behavior and make no comment to the child
c

The PNP is providing anticipatory guidance to the foster mother of a 10-month-old infant weighing 20 lb, 1 oz. The most appropriate advice the PNP can give the foster mother concerning the use of car seats is:
A) “A child weighing more than 20 pounds may face forward in the car seat.”
B) “Keep using a rear-facing car seat until age 1 year.”
C) “It is safe to keep your child in a car seat in the front seat of the car as long as there is not a passenger-side airbag.”
D) “A booster seat should be used when your child has outgrown a convertible car seat but is still too small to fit properly in a vehicle safety belt.”
b

In counseling an adolescent vegetarian, the PNP discusses the need to ensure adequate intake of:
A) Niacin, thiamine, iron, and calcium
B) Vitamin C, calcium, iron, and zinc
C) Calcium, vitamin D, thiamine, and iron
D) Calcium, zinc, iron, vitamin B12, and vitamin D
d

The mother of a 5-month-old breastfed infant requests information about weaning. The PNP advises the mother:
A) Immediately substitute all breastfeedings with formula
B) Wean the infant slowly from the breast to the bottle
C) Substitute all breastfeedings with formula, and pump breasts frequently to avoid engorgement
D) Continue breastfeeding for at least 1 more month
b

A 12-month-old infant is at the clinic for a scheduled well-child visit. The mother expresses concern that the infant awakens in the middle of the night screaming. The mother refuses to let the infant cry because the crying wakes up siblings, so she rocks the infant back to sleep. In addition to telling the parents that the infant’s nighttime awakening has become a habit, the PNP should offer what other advice?
A) The infant should be allowed to stay up late so he or she will be tired and sleep longer at night
B) Rocking is reinforcing the habit. The parents may comfort the infant but should not pick up, rock, or make eye contact with the infant when the infant awakens at night
C) The parents should offer a security blanket or toy at bedtime and use a night light
D) The parents should arrange for the siblings to spend a few nights with relatives or friends and then should allow the infant to cry long enough to fall back asleep
b

A 4-year-old child is examined in the clinic. The parents report that the child has limited language skills and does not speak in three-word sentences. The PNP is interested in further assessing the child and chooses the following:
A) Clinical Adaptive Test/Clinical Linguistics Auditory Milestone Scale (CAT/CLAMS)
B) Vineland Social Maturity
C) Denver Articulating Screening Examination (DASE)
D) Denver II
c

A 12-month-old is seen in the clinic for a well-child check-up. The mother expresses concern about the infant’s sleep pattern. What is the average daily sleep time in infants at age 12 months?
A) 6 to 12 hours
B) 12 to 18 hours
C) 10 to 15 hours
D) 3 to 7 hours
c

What advice would the PNP give a parent about fruit juice during the 4-month well child visit?
A) “Offer any juice except orange juice now.”
B) “Begin to offer orange juice now.”
C) “Do not offer the infant fruit juice now.”
D) “Begin to offer juice to the infant now.”
c

The mother of a 4-month-old infant is planning to return to work. The mother is investigating licensed day care centers and asks about the appropriate ratio of infants to adults. The PNP responds:
A) Three infants to one adult
B) Eight infants to one adult
C) Five infants to one adult
D) Four infants to one adult
a

A mother is asking the PNP about puberty in her daughter. The PNP advises the mother that:
A) Papanicolaou testing should begin at age 16 years
B) Pubic hair development precedes breast budding
C) Menses occur about 2 years after thelarche
D) The adolescent growth spurt begins after pubic hair development
c

A 5-year-old child with a gross motor deficit is brought to the office for a preschool physical examination. The child’s language skills are normal. The parents ask about potential learning difficulties. The PNP stresses the importance of being vigilant and involved with school to optimize the child’s success. The PNP also informs the parents that:
A) The child’s language skills are a strength because language performance is the skill most highly correlated with school success
B) It is impossible to predict learning issues at this age
C) The child is at minimal risk for learning problems because the fine motor development is normal
D) The child is at high risk for learning problems secondary to the gross motor condition
a

A mother asks about the term Moro reflex. The PNP explains that the reflex is present in a neonate and disappears or phases out at about age:
A) 6 to 8 months
B) 3 to 5 months
C) 7 to 9 months
D) 10 to 12 months
b

The father of a 2-year-old child calls the office, requesting help in managing bedtime resistance. The PNP suggests that children should be expected to:
A) Set their own bedtime based on their internal time clock
B) Put themselves to sleep somewhere other than their own bed and then be carried to bed once asleep
C) Make ongoing requests, requiring the parent to come to the room and meet those demands
D) Protest going to bed but parents should be consistent in expecting the child to stay in bed
d

The father of a 2-year-old child calls the office, requesting help in managing bedtime resistance. The PNP suggests that children should be expected to:
A) Set their own bedtime based on their internal time clock
B) Put themselves to sleep somewhere other than their own bed and then be carried to bed once asleep
C) Make ongoing requests, requiring the parent to come to the room and meet those demands
D) Protest going to bed but parents should be consistent in expecting the child to stay in bed
d

The mother of a 2-year-old child is concerned about the child’s thumb sucking during the day when not playing and at night while sleeping. The PNP suggests that:
A) The parents should ignore the behavior because it is not harmful at this age
B) The mother should obtain an orthodontic device for the child’s mouth to discourage the sucking behavior
C) This is not a problem unless the thumb sucking continues past age 3 years
D) The parents should send the child to time-out when caught thumb sucking during the day
a

A 12-year-old girl has questions about the potential for her to get much taller. She aspires to be a professional basketball player. On examination she is in Tanner stage IV. Menarche was 1.5 years ago. In considering answers to her questions, it is important to realize which of the following?
A) Girls reach peak height velocity immediately before menarche
B) She will not reach her peak height velocity until she is in Tanner stage V
C) Menarche occurs an average of 2 years after thelarche
D) Boys reach peak height velocity at the average age of 15 years
a

At the 6-month well-child visit the PNP is counseling a mother about what foods to add to the infant’s diet. The mother asks if she can offer cow’s milk in a cup. The PNP informs the mother that cow’s milk:
A) Is safe and nutritious
B) Has too much fat for the infant
C) Should not be introduced to an infant
D) Is more expensive than formula
c

A 7-month-old infant is brought to the clinic for a well-child visit. The infant has attained the expected developmental milestones for age. The physical examination is normal, and the infant appears to hear well. The mother is concerned that the infant no longer gibbers and imitates sounds as in past months. The PNP responds:
A) “We need to administer the Denver II before providing a referral to an audiologist.”
B) “The child’s hearing needs to be checked, and I’ll give you a referral to an audiologist.”
C) “As an infant develops, milestones may come and go as others are emerging or developing. We will watch the infant closely.”
D) “A change in milestones is of concern; I will refer the infant to a developmental specialist and a neurologist who can rule out cerebral palsy.”
c

A 3-year-old child is brought to the clinic for a well-child visit. The mother reports that the child has recently begun waking up in the middle of the night screaming and appearing frightened and requests information on how to deal with nightmares. The PNP responds:
A) Anxious parents tend to have fearful, anxious children
B) The mother should ignore the behavior and not go to the child’s room
C) The mother should not allow the child to relive the nightmare
D) The mother should listen to the child’s nightmare and reassure the child
d

A 13-month-old child is brought to the clinic for a well-child visit. The child is in the 15th percentile for weight and the 50th percentile for height. The head circumference is in the 50th percentile. The child drinks about 23 oz of skim milk per day and drinks about 20 oz of juice per day. How would the PNP guide this parent?
A) “The child should not be allowed to drink all that skim milk and juice.”
B) “The height is at the 50th percentile, but the weight is at the 15th percentile; give the child about 12 oz of whole milk per day and eliminate the juice.”
C) “The growth is good; continue doing what you are doing”
D) “Let’s talk about food groups and the child’s favorite foods other than milk and juice.”
d

A 13-year-old is in the clinic for her yearly checkup and asks the PNP when she will start her menstrual periods. Her breast and pubic hair development are Tanner stage II and her height is at the 25th percentile. The best response to her is:
A) “You need to be seen by an endocrinologist immediately.”
B) “Looks like you are going to be a late bloomer.”
C) “You will probably begin in the next 6 months.”
D) “Tell me more about your concerns.”
d

A 3-year-old child is brought to the clinic for a well-child visit. The mother requests help with toilet training. The child refuses to use the toilet to have a bowel movement and squats, wearing a diaper, in a secluded corner to pass a stool. The parents offer encouragement through rewards and the “big kid” approach to using the toilet, but nothing seems to help. The PNP should respond that:
A) Refusal to use the toilet after age 2 1/2 years is the result of caregivers (parents, day care workers, etc.) putting too much emphasis on toilet training
B) All responsibilities, including clean up (with appropriate assistance), should be turned over to the child after wetting or soiling pants
C) Autonomy struggles are common at this age. Most children eventually train themselves
D) The child should be forced to sit on the toilet every 2 hours for 5 to 10 minutes during the day
c

A father asks when his premature infant will “catch up” to a normal rate of growth and development. The PNP tells the father “catch up” growth in a premature infant is:
A) Maximal between 36 to 44 weeks after conception
B) Predictable by using growth charts for premature infants
C) Achieved by the age of 5 years
D) Optimal at 6 months from actual delivery
b

The PNP is returning a phone call to a parent who has questions regarding the use of over-the-counter medications for a 2-year-old child. The mother reports that the child has been running a temperature of 101.2°F with no other symptoms of illness. The mother has been treating the fever by alternating doses of acetaminophen and ibuprofen every 3 to 4 hours. The child is playful and has a normal appetite. Appropriate counseling for this mother should include:
A) Continue giving the medicines every 3 to 4 hours at the appropriate dosage for the child’s weight until the fever subsides
B) Increase the interval of the medications to every 6 hours
C) This amount and frequency of antipyretics may be inappropriate and harmful in a child whose only symptom is a low-grade fever
D) Stop the medications, because a temperature of 101.2°F in a child with no other symptoms does not necessitate the use of antipyretics.
c

When asked at a prenatal visit by the mother, “How should I feed my infant—breast or bottle?” How should the PNP reply?
A) “Breastfeeding is the best way to feed the infant.”
B) “A combination of breast and bottle is best.”
C) “Formula is an easy way to offer adequate nutrition.”
D) “Feed the infant whichever way you want.”
a

A 2-week-old neonate is brought to the clinic for a well-child visit. The infant weighed 3.18 kg at birth; today the weight is 2.95 kg. The mother is breastfeeding about every 2 hours; the infant is latching on well and nursing 15 minutes per side. How many calories a day does this infant need for growth?
A) 300 Kcal
B) 345 Kcal
C) 375 Kcal
D) 500 Kcal
b

The most sensitive physical finding for the diagnosis of testicular torsion is:
A) Abnormal results of urine culture
B) Pain lasting for hours or days
C) Absence of the cremasteric reflex
D) The alleviation of pain when the testis is elevated
c

When speaking to a fifth grade class about injury prevention, based on the national top five causes for fatal injuries in this age group, which of the following topics would the PNP emphasize?
A) Bike safety (wearing a helmet) and drowning prevention
B) Drinking alcohol and driving, and safe sex
C) Sniffing glue and smoking prevention
D) Drowning prevention and substance abuse
a

A child who is able to run, kick a ball, wash and dry the hands, tower two cubes, and use pronouns indiscriminately is probably aged:
A) 24 months
B) 18 months
C) 40 months
D) 36 months
a

A 16-year-old female adolescent with Down’s syndrome, who is healthy and has had no major medical problems, comes to the clinic for a well-child visit. The adolescent has been “mainstreamed” into a regular classroom and is doing well. The physical examination reveals Tanner stage III breast development and pubic hair. Today the PNP should:
A) Order cervical spine x-ray films, a vision screen, and echocardiography
B) Perform hearing and vision screens, and refer the adolescent to a long-term planning counselor
C) Order thyroid and hearing screens, and provide sexuality education
D) Provide nutritional counseling, order cervical spine x-ray films, and refer the adolescent to a family support group
c

Children born with cleft lip/palate are prone to:
A) GER
B) Pyloric stenosis
C) Milk bottle caries
D) Nasal allergies
c

One of a toddler’s greatest fears is:
A) Not being accepted by peers
B) Not getting enough food
C) Separation from parents or caregivers
D) Having limits set by caregivers
c

The mother of an 18-year-old with epilepsy brings her to the clinic for family planning. The PNP discusses the options and recommends:
A) An intrauterine device (IUD)
B) Levonorgestrel (Norplant)
C) Oral contraceptive pills (OCPs)
D) Medroxyprogesterone (Depo-Provera)
d

A 2-year-old child is brought to the clinic for the first time. The child has microcephaly, age-appropriate weight and height, and developmental functioning below expectations in all categories on the Denver II. The PNP should:
A) Refer the child to an early intervention program
B) Refer the child to a speech pathologist
C) Order chromosomal studies and a CT scan
D) Repeat Denver II screening in 1 month
a

Which of the following is the major cause of death among adolescents and young adults aged 15 to 24 years?
A) Homicide
B) Motor vehicle-related accidents
C) Acquired immunodeficiency syndrome (AIDS)
D) Suicide
b

The PNP examines a 12-month-old child for routine care. Of the following fine motor milestones, which would be most characteristic of a 12-month-old child?
A) Uses a neat pincer grasp
B) Transfers objects between hands
C) Scribbles spontaneously
D) Builds a tower of 4 cubes
a

The grandmother of an 8-month-old infant brings the infant to the clinic for a well-child visit. The infant missed both the 4- and 6-month examinations and is behind on immunizations. The grandmother states that the parents were substance abusers during the prenatal period and are still using drugs and alcohol. At this visit, appropriate care and guidance should include:
A) Growth assessment, Denver II screening, and immunizations
B) Growth and developmental assessment, referral for genetics evaluation, and scheduling an appointment in 1 month for immunizations
C) Complete physical examination, immunizations, and referral to a genetics center for professional developmental evaluation
D) Growth and developmental assessment, immunizations, social services referral, and scheduling an appointment in 1 month to update immunizations
d

The PNP is seeing a 4-year-old Romanian child for an initial checkup. A local couple, whose other two children also come to the clinic for care, recently adopted the child. The child had been living in a state-operated orphanage, and no medical records are available. Today’s visit should include the following:
A) Physical examination and referral for counseling with a counselor specializing in adoptive families
B) Complete physical examination and an attempt to obtain previous medical records
C) A discussion of problems and determination of the child’s medical needs
D) Complete physical examination, a CBC, urinalysis, immunizations (DTaP, IPV, MMR, varicella, and Hep B), HIV, and a PPD test
d

A 2-year-old child screened for lead poisoning has a lead level of 14 mcg/dL. The intervention provided by the PNP should include:
A) A long bone x-ray film for lead
B) A low-calcium diet
C) Referral to the health department for environmental investigation
D) A high-calcium diet
d

A 5-year-old child is brought to the clinic for a prekindergarten physical. The father has refused the MMR vaccine, stating that the child is allergic to eggs. Upon further questioning, it is determined that the child experienced stomach cramps and diarrhea after eating eggs. The child has had 4 DTaP, 3 IVP, 3 Hep b, 3 Hib, a PPD, and varicella. What immunizations would be appropriate to administer to this child today?
A) DTaP, IPV, MMR, and varicella, as well as a PPD test (if warranted)
B) DTaP and IPV, as well as a PPD test
C) DTaP, IPV, and varicella, as well as a PPD test
D) DTaP, IPV, MMR, varicella (if warranted), and pneumococcal vaccine (Pneumovax)
a

A child diagnosed with HIV can best be monitored by assessing:
A) CD4 antigen counts and viral burden
B) Anorexia and fatigue
C) ELISA results
D) Weight loss and fatigue
a

A 14-year-old adolescent has been diagnosed with early hypothyroidism based on increased TSH and decreased T4 levels. The PNP consults with the physician and in discussing the management decides to:
A) Increase the dietary intake of iodized salt
B) Refer to an endocrinologist
C) Treat with levothyroxine
D) Recheck levels in 2 months
c

A 7-year-old child is brought to the clinic for a routine examination. The child likes to skateboard and does it almost every day. The PNP reviews specific injury prevention related to skateboarding. The most common injuries that occur during skateboarding are:
A) Injuries to the trunk
B) Head and neck injuries
C) Injuries or fractures of the lower extremities
D) Injuries or fractures of the upper extremities
d

The PNP is performing a neonatal examination in the nursery. A pustular type of eruption is noted on the neonate’s face and trunk; some of the lesions are transient. The PNP diagnoses:
A) Streptococcal infection
B) Staphylococcal infection
C) Erythema toxicum
D) Herpes infection
c

The PNP is examining a 14-year-old adolescent with a strained ankle received during football practice. Which of the following is true regarding sports-related injuries?
A) Wearing appropriate protective equipment reduces the injury risk
B) Rapid growth phases in which muscle growth is greater than bone growth cause increased clumsiness and increase the injury potential
C) Children younger than 14 years have twice the injury risk of those aged 14 years and older
D) Overuse injuries cause less permanent damage than accidental injuries
a

The first step in the process of developing an IEP for a child with disabilities is to:
A) Perform a comprehensive assessment of the child by a multidisciplinary team
B) Perform a battery of psychological tests that are valid and reliable
C) Perform a complete history and physical examination to identify organic causes
D) Provide government funds to the child and family to supplement income
a

While examining a 3-year-old child for a well-child visit, labial adhesions are noted. The PNP discusses with the mother:
A) Surgical treatment of the adhesions
B) Proper hygiene
C) Concern of sexual abuse
D) Causes of UTI
b

The PNP notes anal fissures on the physical examination of a 2-year-old child. There is no history of constipation. The PNP is concerned about:
A) Ulcerative colitis
B) Crohn’s disease
C) A foreign body
D) Sexual abuse
d

The PNP is speaking with a sixth grade class about injury prevention. Which of the following topics would the PNP emphasize?
A) Bike safety and wearing a helmet, drowning prevention
B) Drowning prevention, substance abuse
C) Drinking alcohol and driving, safe sex
D) Sniffing glue, smoking prevention
a

A 6-month-old infant is brought to the clinic for a well-child visit. On physical examination, increased tone and lower leg scissoring are noted. The PNP recognizes this as a probable sign of:
A) Down’s syndrome
B) Cerebral palsy
C) Fragile X syndrome
D) Muscular dystrophy
b

The PNP is performing a preschool physical examination on a 4-year-old child who is new to the practice. A bifid uvula is noted, and on further examination of the back of the hard palate, the PNP finds an abnormal contour. The PNP decides to:
A) Check the child’s gag reflex and determine whether the child has tonsils
B) Determine whether the child’s thyroglossal cranial nerve is intact
C) Test the child’s speech and hearing
D) Test the child’s hearing and ability to lateralize sound
c

A child who has a 250-word vocabulary, uses three-word sentences, walks upstairs alternating steps, rides a tricycle, and uses a spoon well is approximately aged:
A) 3 years
B) 6 years
C) 5 years
D) 4 years
a

The first sign of puberty in girls is usually:
A) Acne
B) Pubic hair growth
C) Thelarche
D) Menarche
c

Which of the following best describes the Denver II?
A) It screens fine and gross motor, expressive and receptive language, and cognitive academic skills by direct observation of children aged 2 to 6 years
B) It assesses receptive and expressive language based on observation and parental interview of infants and children aged 1 to 36 months
C) It assesses receptive and expressive language by interview, observation, and test administration for infants and children from birth to age 36 months
D) It is a global screening tool, including personal/social, fine motor/adaptive, and gross motor skills of infants and children from birth to age 6 years
d

At an 18-month-old child’s well-child visit the mother states that the child is a picky eater and has 4 oz of milk by bottle at nap time and 8 oz of milk by bottle at bedtime. The rest of the child’s fluids are taken by cup. Of the following, what is the most pertinent information for the PNP to obtain?
A) The total intake of milk for 24 hours
B) The family’s financial status
C) A list of foods the child refuses
D) A 24-hour recall diet
d

The mother of a preschool-aged child is discussing intellectual giftedness and asks the PNP what is the best predictor and most frequently noted indicator of an intellectually gifted child. The PNP responds:
A) Ability to entertain self
B) Early language development
C) Extreme curiosity
D) Participation in an early enrichment program
b

The PNP is examining a 12-year-old female child with enlarged breasts and areolas and curly, dark, coarse pubic hair. The PNP uses Tanner staging and classifies this child as:
A) Tanner stage V
B) Tanner stage II
C) Tanner stage IV
D) Tanner stage III
d

The PNP should consider referring for a more comprehensive developmental evaluation a child who:
A) Has explosive temper tantrums at age 19 months
B) Does not speak by age 15 months
C) Has lost the ability to perform previously attained developmental milestones
D) Has consistently tracked in the 5th percentile for growth since birth
c

A 2-week-old infant is brought to the clinic for a well-child visit. During the interview, which of the following questions would be most appropriate to ask the parent?
A) Do you have any questions about your car seat and how to use it?
B) Is your hot water heater set at 120°F or lower?
C) Have you installed safety locks or window guards on all windows?
D) Do you use sunscreen to prevent sunburn?
a

A 4-year-old child is brought to the office for a preschool physical examination. The PNP asks the child to sit on the floor in a cross-legged position and then get up without using the hands. The child is unable to do so and rolls onto all four extremities before standing. The PNP is concerned that the child may have a:
A) Developmental problem
B) Cognitive deficit
C) Neurologic deficit
D) Neuromuscular disorder
d

Which of the following parental responses would discourage sibling rivalry?
A) Serving as role models by solving conflicts in a nonconfrontational (nonaggressive) manner
B) Comparing qualities of the siblings
C) Labeling children in negative and positive roles
D) Favoring one child over the other
a

Unintentional injuries are the major cause of death in preschool-aged children. Of the following, which causes the most deaths in preschool-aged children?
A) Motor vehicle injury
B) Burns
C) Drowning
D) Poisoning
a

The normal motor development of a 2-month-old infant includes:
A) Grasping a rattle and lifting the head and shoulder off the bed when in the prone position
B) Briefly fixating on a face or object and grasping an object when placed in the palm of the hand
C) Reaching for objects, transferring objects from one hand to the other, and lack of head lag when pulled to a sitting position
D) Bearing weight on the forearms when in the prone position, rolling from the prone to the supine position, and bearing weight when standing
a

A PNP is following a child placed in foster care. The most common health problem of children in foster care is:
A) Parasitic infection
B) Neurological disorders
C) Respiratory disease
D) Growth failure
c

An infant’s neurologic status is being evaluated. The PNP preceptor asks when the parachute developmental reflex occurs. The PNP student responds:
A) Age 8 to 10 months
B) Age 2 to 4 months
C) Age 5 to 7 months
D) Age 4 to 6 months
a

The PNP is examining a sexually active 17-year-old boy who admits to having unprotected sex over the last several weeks. Appropriate care for this patient includes:
A) Evaluation of immunization status, hepatitis B screening panel, HIV screen, and penile smear for gonorrhea
B) Complete blood count and penile smear for gonorrhea
C) Physical examination, STD history and screening tests, immunization update, anticipatory guidance regarding responsible and safe sexual behavior, and recheck in 6 months
D) No tests because the patient is asymptomatic, anticipatory guidance, and recheck in 6 months
c

The PNP is evaluating a postterm (42 weeks’ gestation) neonate in the nursery. On the physical examination the PNP would expect to find:
A) An elevated weight/length ratio
B) Smooth soles of the feet
C) Excessive vernix caseosa
D) Cracked, peeling skin
d

At a well-child visit the parents report that the child, who is now approximately age 2.5 years, frequently rocks back and forth and has done so since about age 6 months. The physical examination is normal, as is the developmental history. The most likely cause of the rocking behavior is:
A) Emotional deprivation
B) Cerebral dysfunction
C) Autism
D) Normal variation
d

In the event of a malpractice lawsuit against the PNP, several elements must be proved. Included in these elements are the following: the PNP owed the plaintiff a duty or responsibility, the PNP’s conduct or care fell below a recognized standard of care, and:
A) The PNP operated outside a recognized standard of care
B) The PNP was not properly credentialed
C) The PNP did not consult with a supervising physician
D) The plaintiff was injured by the PNP’s conduct
d

National certification of PNPs has been offered since the 1970s. The following credentials may identify nurses who have national specialty certification:
A) Master of science in nursing (MSN)
B) Certified (C)
C) Advance practice nurse (APN)
D) Nurse practitioner (NP)
b

The PNP has recently read a report, published by the Committee on the Quality of Health Care in America, about medical errors in office practice. Which of the following suggestions should the PNP make to the managed care administration to decrease medical errors in the clinical area?
A) Require that health care providers tell the patient one time the medications being prescribed
B) Encourage health care providers to dictate all chart entries
C) Stress the need for health care providers to write legibly
D) Advocate keeping electronic medical records
c

Public Laws 94-142 and 101-476 mandate that a child with disabilities is entitled to:
A) An individualized education program (IEP)
B) Genetic evaluation and counseling
C) Benefits provided by the Supplemental Security Income Program for the Aged, Blind, and Disabled
D) Evaluation with the Wechsler Intelligence Scale for Children III (WISC III)
a

In the health care system the major perceived barrier to use of a PNP’s services in a primary health care setting is:
A) Lack of quality of service provided
B) Organized nursing
C) Organized medicine
D) Lack of consumer confidence
c

Title V of the Social Security Act of 1935 was amended in 1981 and 1986 to expand coverage for children with special health care needs under:
A) The Supplemental Security Income (SSI) Program for the Aged, Blind, and Disabled
B) Title XIX of the Social Security Act
C) The Maternal and Child Health Services Block
D) The U.S. Public Health Service
c

The first nurse practitioner program was designed to:
A) Manage severely ill children at Colorado General Hospital
B) Provide limited access to health care for low-income children in Colorado
C) Provide increased access to health care for children in Colorado
D) Provide emergency care to children in Colorado
c

The most common form of reimbursement for services provided by PNPs is:
A) Fee charged at 85% of the physician’s payment
B) Same fee as physician, “equal pay for equal service”
C) Fee for service
D) Fee charged at 90% of the cost under the physician’s billing
c

The PNP is employed by a physician and performs services under direct supervision of the physician; these services provided by the PNP are billed under the physician’s name and are called:
A) Incidence to service
B) Salary
C) Direct billing for service
D) Capitation reimbursement
a

  1. A 10mo old child has been diagnosed with gastroenteritis. He attends a child care facility. What is the most likely cause of his illness?
    a. Clostridium difficile
    b. Rotavirus
    c. Salmonella
    d. Cryptosporidium
    b
  2. In a healthy, 8mo old with diarrhea but not dehydration, what would be the most appropriate device to give parents?
    a. Encourage ½ strength formula for 12hrs
    b. Give oral hydration solution (ORT) for 12hrs
    c. Give only fluids until stool turns to normal
    d. Give bananas and cereal as tolerated
    d
  3. When evaluating a child with abdominal pain, what symptom would lead to a likely organic etiology?
    a. Night waking
    b. Pallor
    c. Suprapubic pain
    d. Sweating
    a
  4. Vomiting in infancy has a long list of differential diagnoses. Which accompanying symptom would most likely point to pyloric stenosis?
    a. Diarrhea
    b. Appropriate growth
    c. Acts hungry after vomiting
    d. Sausage shaped mass in abdomen
    c
  5. Which of the following is the appropriate regimen for pinworm medication?
    a. Daily x 7days, repeat as needed
    b. TID for 10 days, repeat as needed
    c. BID for 3 days, repeat in 2 weeks
    d. 1 dose/1 time, repeat in 2 weeks
    d
  6. Mrs. Doyle is upset. 2mo old John’s frequent vomiting has her convinced that “something is seriously wrong”. Which of the following is most suggestive of GER?
    a. He’s gained 5oz this month
    b. He has a slight wheeze today
    c. He eats hungrily after vomiting
    d. He drinks 7-8oz every 3-4hrs
    d
  7. You see Jack, a 20mo old toddler with normal growth and development in your office for diarrhea. His mother tells you that he is passing up to three loose stools a day and that he drinks 20oz of apple juice/day. What is the most likely diagnosis?
    a. Crohns disease
    b. Giardia lamblia
    c. Celiac disease
    d. Nonspecific “toddler” diarrhea
    d
  8. Baby Sally was in your office last week for her 6mo check up. Her weight was 7kg. Today she presents with diarrhea and vomiting for 4 days. Today her weight was 6.5kg. What is her percentage of dehydration?
    a. 5%
    b. 7%
    c. 10%
    d. <1%
    b
  9. What clinical signs would you expect to see in Sally on your examination (with 7% dehydration)?
    a. Normal capillary refill
    b. Normal fontanel
    c. Cool mottled skin
    d. Dry mucous membranes
    d
  10. Sally is vomiting and diarrhea has stopped. If she needs oral replacement therapy (ORT) today, what would be the appropriate amount to recommend?
    a. 325-350 cc over 4hrs
    b. 600-700cc over 4hrs
    c. 600-700cc over 12hrs
    d. 325-350cc over 8hrs
    b
  11. Pinworms can cause which of the following?
    a. Constipation
    b. Anal itching
    c. Abdominal pain
    d. Diarrhea
    b
  12. In evaluating Billy, a child with bloody diarrhea, which of the following would not be appropriate first action?
    a. Check growth chart
    b. Stool culture
    c. Upper GI
    d. Hemoccult test stools
    c
  13. Billy’s family eats at fast food restaurants 4-5x/week. If you suspect the diarrhea is infectious in nature, what is likely causative organism?
    a. Adenovirus
    b. E. coli
    c. Giardia lamblia
    d. S. aureus
    b
  14. Which of the following conditions would be most likely to occur in a 4yo boy?
    a. Pyloric stenosis
    b. Recurrent abdominal pain
    c. Intussuseption
    d. Giardia infection
    d

Which of the following findings would be expected to occur in a baby with intussusception?
a. Inconsolable screaming
b. Olive shaped mass
c. Left to right peristaltic waves
d. Weight loss
a

  1. Which of the following may occur with suspected appendicitis?
    a. Pain not relieved with ambulation
    b. Young children appear very ill in the early phase
    c. Fever is 102-103
    d. Leukopenia with left shift
    a
  2. In the U.S. parasitic gastroenteritis is most commonly caused by which organism?
    a. Enterobius vermicularis
    b. Entamoeba histolytica
    c. Cryptospordidium parvum
    d. Giardia lamblia
    d
  3. Which of the following serological findings indicates a carrier state for HBV?
    a. HBs Ag negative for 6mo
    b. IgM anti HBc negative and HBsAg positive
    c. Anti-HBc positive
    d. Anti-HBs positive
    b
  4. Children in child-care facilities are at greater risk of being exposed to which of the following infections?
    a. HAV
    b. HBV
    c. HCV
    d. HDV
    a
  5. Infant immunizations for hepatitis B often raises many parental questions about the disease. Which of the following is not true about Hep B virus?
    a. It can survive for more than 1 week on fomites
    b. It is most common form of hepatitis in the world
    c. Contaminated water & shellfish are the major source
    d. Perinatally infected infants are likely to become carriers
    c
  6. Two day old baby Jamie is in the hospital nursery and still has not passed meconium. This is a red flag for what condition?
    a. Intussuseption
    b. Hemolytic uremic syndrome
    c. Pyloric stenosis
    d. Hirschsprungs Disease
    d
  7. Consistent with Hirschsprungs Disease, Jamie’s findings on rectal exam would be which of the following?
    a. Tight anal canal with no stool in vault
    b. Impacted stool with fissure
    c. Large dilated rectum
    d. Soft stool, normal tone
    a
  8. What treatment would be appropriate for Jamie’s Hirschsprungs Disease?
    a. Emulsified mineral oil, ½ tablespoons per day
    b. Referral to gastroenterologist/surgeon
    c. Malt soup extract, 2 tsp for 3 days
    d. Rectal dilation with thermometer
    b
  9. When evaluating a child with suspected IBD, which of the following diagnostic tests would not be helpful?
    a. Amylase & lipase
    b. ESR
    c. Serum total protein & albumin
    d. CBC with diff
    a
  10. Your patient has inflammatory bowel disease. Which finding is most consistent with ulcerative colitis?
    a. Occult blood
    b. Perirectal abcess
    c. Aphthous ulcers
    d. Left sided abdominal pain
    d
  11. Antimicrobials will improve the condition of a 4yo child with diarrhea caused by which of the following organisms?
    a. Salmonella
    b. Rotavirus
    c. Shigella
    d. E. coli (0157:H7)
    c
  12. Katie has functional abdominal pain. When counseling her family on management of painful episodes you would recommend which of the following?
    a. Take ibuprofen 200mg for pain
    b. Stay home from school during episode
    c. Decrease milk products
    d. Go to school during episode
    d
  13. Which of the following would not be consistent with a diagnosis of functional constipation in an infant?
    a. Vomiting
    b. Anal fissure
    c. Straining
    d. Starting solids
    a
  14. A child has developed her 2nd perirectal abscess in 6mo. She should be evaluated for which condition?
    a. Giardia lamblia
    b. Crohn’s disease
    c. Ulcerative colitis
    d. Enterobiasis
    b
  15. Which of the following symptoms are most common in the early phase of appendicitis in children?
    a. Abdominal pain after eating
    b. Fever & diarrhea
    c. Severe localized RLQ pain with pallor & sweating
    d. Anorexia, vague, diffuse pain
    d
  16. Steatorrhea is not consistent with which of the following?
    a. C. diff
    b. Giardia lamblia
    c. Celiac disease
    d. Cystic fibrosis
    a
  17. Jamil has had diarrhea for 3 days. His mother calls concerned. Which of the following would not be helpful advice?
    a. Monitor stool for blood or mucus
    b. Encourage solid food
    c. Avoid milk products
    d. Monitor for urination for at least 6hrs
    c
  18. Of the following advice, which would be most helpful for the parents of a baby with gastroesophageal reflux?
    a. Most babies continue to vomit until they are walking, at around 1 yr of age
    b. Laying prone after eating will decrease the amount of vomiting
    c. Increase the interval between feedings to a minimum of 4hrs
    d. Medications are generally necessary to prevent further problems
    b
  19. Which of the following foods would be appropriate for a child with celiac disease?
    a. Oatmeal for breakfast
    b. Boiled rice with butter
    c. Commercially baked bread
    d. Cream of wheat
    b
  20. A parent requests that her 6mo old child receive immunoglobulin as protection against hepatitis A prior to international travel. Which of the following does this parent need to know?
    a. After IG administration, a 3mo interval is needed prior to the next measles vaccination
    b. There is no impact on future immunizations
    c. No immunizations can be given for 1year
    d. Since children do not have symptoms with hepatitis A, IG is not necessary
    a
  21. Septicemia in the newborn period is likely caused by which organism?
    a. Listeria monocytogenes
    b. Haemophilus influenza
    c. Neisseria meningitides
    d. Streptococcus pneuomniae
    a
  22. Signs & symptoms of bacterial sepsis in children beyond the neonatal period include?
    a. Cough, fever, abdominal pain
    b. Vesicular rash, pruritus, fever
    c. Irritability, fever, lethargy
    d. Abdominal pain, diarrhea, vomiting
    c
  23. Which of the following vaccines provides protection against a common type of sepsis/meningtitis?
    a. Smallpox vaccination
    b. Hep B vaccine
    c. Haemophilus influenzae vaccine
    d. Inactivated polio vaccine
    c
  24. Although relatively rare in the U.S., diphtheria can occur among under-immunized children. Which of the following clusters of signs, symptoms, and physicals findings would suggest diphtheria in a child presenting with upper respiratory complaints?
    a. Low grade fever, sore throat, nasal discharge, grayish-white pseudo-membrane in his/her throat
    b. Abrupt onset of high fever, severe sore throat, nasal discharge, and grayish-white pseudo-membrane in his/her throat
    c. Low grade fever, abrupt onset of severe sore throat with difficulty swallowing and drooling
    d. Abrupt onset of high fever, severe sore throat, with difficulty swallowing & drooling
    a
  25. Infants younger than 6mo of age with pertussis frequently require hospitalization to manage:
    a. Fever, cough, dehydration
    b. Coughing paroxysms, apnea, cyanosis, feeding difficulties
    c. Coughing paroxysms, dehydration, renal failure
    d. Seizure, fever, pneumonia
    b
  26. One of the most appropriate agents used to treat influenza A is:
    a. Acyclovir
    b. Osteltamivirc
    c. Erythromycin
    d. Tetracycline
    b
  27. Which of the following symptoms are characteristic of rubella?
    a. Vesicular, crusted lesions & high fever
    b. Postauricular lymphadenopathy & low grade fever
    c. Intense pruritus, usually in finger webs, buttocks, thighs, & ankles
    d. Rough textured maculopapular rash that blanches with pressure
    b
  28. Although uncommon, a potential sequela of rubella may include:
    a. Pneumonia & chronic otitis media
    b. Arthritis, thrombocytopenia & encephalitis
    c. Oophoritis & infertility
    d. Arthritis, carditis, and neurological involvement
    b
  29. Rubeola is:
    a. Preventable by active immunization
    b. Caused by human herpesvirus 6
    c. Treated with intravenous acyclovir
    d. Not associated with severe complications (e.g., encephalitits, pneumonia
    a
  30. You are examining a child who has fever, coryza, cough, conjunctivitis, malaise, & anorexia. During the oral examination, you observe red eruptions with white centers on the buccal mucosa. What are these eruptions called?
    a. Pastia’s spots
    b. Rubelola spots
    c. Koplik’s spots
    d. Strawberry spots
    c
  31. Which of the following best describes the treatment for roseola?
    a. Acetaminophen or ibuprofen for fever, parental reassurance
    b. Warm compresses for salivary gland swelling
    c. Oral acyclovir, 20mg/kg/dose, four times/day
    d. Bed rest, saline gargles for sore throat
    a
  32. Fifth disease is usually:
    a. Seen in age 5-14yo children
    b. Transmitted via the deer tick
    c. Treated with oral erythromycin
    d. Characterized by prolonged coughing episodes
    a
  33. Which of the following statements is not true regarding the transmission of chickenpox?
    a. Susceptible individuals can contract chickenpox from patients with varicella zoster (shingles)
    b. Children with chickenpox are infectious only during the period of time when skin lesions are present
    c. Children with chickenpox are no longer infectious once crusting of skin lesions has occurred
    d. Varicella-zoster immune globulin (VZIG) should be administered to susceptible immunocompromised individuals who are exposed to a patient with varicella zoster infection
    b
  34. A child with chickenpox and temperature of 102F should receive which medication for fever?
    a. Aspirin
    b. Amoxicillin
    c. Acetaminophen
    d. Acyclovir
    c
  35. The most appropriate agent for use in treating varicella zoster infection in an immunocompromised host is:
    a. Ganciclovir
    b. Acyclovir
    c. Cetriaxone
    d. Chloramphenicol
    b
  36. Varicella zoster infection is most commonly associated with which of the following skin lesions?
    a. Vesicle
    b. Comedone
    c. Nodule
    d. Macule
    a
  37. Which of the following is not a complication of mumps?
    a. Meningitis
    b. Pneumonia
    c. Oophoritis
    d. Pancreatitis
    b
  38. What recommendation would you make to a parent whose son has been diagnosed with mumps and wants to know when he can return to child-care?
    a. He can return once he becomes afebrile and can tolerate eating
    b. He can return 9 days after onset of symptoms
    c. He can return when he is well enough participate in activities
    d. He can return after a minimum of 5 days of antibiotic therapy
    b
  39. Which of the following are symptoms of cat scratch disease?
    a. Joint pain, conjunctivitis, mild neck stiffness
    b. Irritability, fever, hypotension
    c. Fever, malaise, lymphadenopathy
    d. Sever cough, vomiting, anorexia
    c
  40. Which of the following describes a characteristic rash associated with which disease? Initially erythematous and macular, becoming maculopapular and petechial. The rash first appears on the wrists & ankles, spreading proximally to the trunk. The palms and soles are often involved.
    a. Lyme disease
    b. Roseola
    c. Rubeola (measles)
    d. Rocky mountain spotted fever
    d
  41. A 10yo child manifests symptoms of fever, sore throat, and swollen lymph nodes. Spleen tip is palpable. Throat culture and monospot test results are negative. The next logical diagnostic test would involve:
    a. Repeat throat culture
    b. Chest radiograph
    c. Bone marrow examination
    d. Epstein-barr virus titer
    d
  42. Which of the following factors is not associated with increased risk for infantile botulism?
    a. Rural environments
    b. Use of honey
    c. Use of corn syrup
    d. Farm families
    c
  43. Which of the following interventions would not be appropriate for a 6mo old infant with a suspected diagnosis of infantile botulism?
    a. Stool & blood cultures
    b. Immediate administration of equine antitoxin
    c. Stool softeners
    d. Supportive cares
    b
  44. Which of the following are associated with paralytic poliomyelitis?
    a. Lacy, erythematous, pruritic rash
    b. Respiratory compromise, speech disturbances, urinary incontinence
    c. Abdominal swelling, lympthadenopathy, and jaundice
    d. Nonspecific abdominal pain, nausea, and vomiting
    b
  45. Muscle spasms associated with tetanus are aggravated by which of the following?
    a. Fever
    b. Tetanus immunoglobulin
    c. External stimuli
    d. NSAID
    c
  46. Classic symptoms associated with malaria include:
    a. Low grade fever, upper respiratory congestion, cough
    b. Annular rash, conjunctivitis, headache, arthralgia
    c. High fever, chills, rigors, sweats, headache
    d. High fever, jaundice, lethargy, vomiting
    c
  47. Lyme disease is most closely associated with which of the following skin lesions?
    a. Erythema migrans
    b. Nodule
    c. Scale
    d. Pustule
    a
  48. Many infectious diseases present with rashes along with general complaints of fever, malaise, and headaches. Which of the following clusters of symptoms would make you consider Lyme disease as a likely diagnosis?
    a. Fever, malaise, headace, arthalgia, and well circumscribed, erythematous, annular rash with central clearing
    b. Fever, malaise, headache, transient bone pain, and generalized erythematous, maculopapular rash that began on the face and spread to trunk and extremities
    c. Fever, malaise, anorexia, and confluent, erythematous, brownish maculopapular rash
    d. Fever, malaise, anorexia, and erythematous rash beginning on wrists and ankles then spreading to the trunk
    a
  49. Which of the following would be included in patient education regarding lyme disease?
    a. Educate caretakers regarding complications, including hypertension and renal calculi due to immobility
    b. Avoid use of aspirin-containing products for fever control due to association with increased risk for Reye’s syndrome
    c. Protective clothing and tick repellent should be worn in heavily wooded areas
    d. Educate caretakers regarding natural history of the illness, and specific follow up needed after infection with Rickettsia rickettsii
    c
  50. Which of the following disorders is usually associated with adduction of the forefoot?
    a. Internal femoral torsion
    b. Talipes equinovarus congenital
    c. Genu valgum
    d. Internal tibial torsion
    b
  51. The most common rheumatoid disease of childhood is:
    a. Systemic lupus erythemtosus
    b. Kawasaki disease
    c. JRA
    d. Legg-Calve-Perthes disease
    c
  52. Radiographic findings of disease progression and sphericity of femoral head is helpful in the diagnosis and followup of:
    a. Transient synovitis of the hip
    b. Osgood-schlatter disease
    c. Legg-Calve-perthes disease
    d. Slipped Capital Femoral epiphysis
    c
  53. A 4yo boy is brought in by his mother, concerned about the sudden onset of painful limp in his right leg 2 days ago. Today he has a low grade fever. Which of the following diagnosis is most likely?
    a. Osgood-schlatter
    b. JRA
    c. Osteomyelitis
    d. Transient synovitis of the hip
    d
  54. Which of the following would be the most appropriate initial management of a newborn diagnosed with developmental dysplasia of the hip?
    a. Observe & reexamine at 2 week well child visit
    b. Triple diapering in nursery
    c. Pavlik harness
    d. Surgical reduction
    c
  55. A physical finding not usually associated with talipes equinovarus congenita is:
    a. Contracture of the illiotibial bands
    b. Deep crease on medial border of foot
    c. Atrophy of calf muscles
    d. Small foot with limited dorsiflexion
    a
  56. A characteristic feature of polyarticular JIA disease is:
    a. The involvement of 5 + inflamed joints
    b. Confinement to lower extremity joints, knees, and ankles
    c. Asymmetric involement
    d. High, daily intermittent spiking fevers
    a
  57. ANA seropositivity for antibodies is:
    a. A valuable diagnostic marker for JIA
    b. Is not + in any other childhood diseases
    c. More commonly found in older boys or in systemic disease
    d. Present in over 75% of cases
    a
  58. Dislocation in the hip of a child 6mo or older may typically present with:
    a. Asymmetry of skin folds
    b. Atrophied hip muscles
    c. Positive Galeazzi sign
    d. Negative trendelenburg sign
    c
  59. For a newborn, the correct management of hip dislocation should include:
    a. Use of flexion-abduction device such as Pavlik harness to stabilize hip
    b. Follow and observe closely for 3-4 weeks then refer to ortho
    c. Surgical reduction
    d. Traction for 6 weeks
    a
  60. Duchenne muscular dystrophy is characterized by which of the following s/s?
    a. A birth, affected infants are notably hypotonic, “floppy” babies
    b. Earliest symptom is often refusal to bear weight
    c. Abnormalities of gait and posture become evident during preschool years
    d. Unable to keep up with peers when running by school age
    c
  61. Most children with Duchenne muscular dystrophy become wheel-chair dependent by what age?
    a. 7-9 yo
    b. 10-12 yo
    c. 14-16yo
    d. highly variable depending on response to treatment
    b
  62. School aged children and young adolescents involved in athletic activities may not be at increased risk for?
    a. Osgood-schlatter disease
    b. Chondromalacia
    c. Spondylolysis
    d. Slipped capital femoral epiphyis
    d
  63. Management of scoliosis depends on the severity of curve as well as the age of the child. Which of the following would require surgical intervention?
    a. Curves of 15 degrees in a child who is still growing
    b. Thoracic and/or lumbar curve greater than 25 degrees, even if growth is complete
    c. Thoracic curve greater than 30 degrees or lumbar curve greater than 40 degrees that has not progressed while in brace
    d. Thoracic curve greater than 50 degrees or lumbar curve greater than 40 degrees
    d
  64. In performing a diagnostic workup and management plan for a child with osteomyelitis, which of the following is not accurate or recommended?
    a. Elevated ESR confirms diagnosis
    b. Aspiration is usually indicated
    c. Antibiotic treatment for 4-6 weeks is recommended
    d. Surgery is recommended if abscess is present
    a
  65. A 6yo child presents with a limp and knee pain. The PNP finds limited passive internal rotation and abduction of the hip joint on physical exam. The most likely diagnosis is:
    a. Slipped capital femoral epiphysis
    b. Osgood-schlatter disease
    c. Transient synovitis of the hip
    d. Legg-calve-perthes disease
    d
  66. Which of the following statements is true about acute osteomyelitis?
    a. Occurs more frequently in females than males
    b. Peak ages are infancy (<1yo) & preadolescence (9-11yo)
    c. Most common sites are radius & ulna
    d. Self limiting disorder
    b
  67. Which of the following statements is not true of slipped capital femoral epiphysis?
    a. Thought to be precipitated by hormone changes during puberty
    b. Unilateral involvement is more common than bilateral
    c. More common among males and African Americans
    d. Thought to be caused by repetitive stresses in young athletes prior to growth spurt
    d
  68. Genu varum is considered an abnormal condition when:
    a. Extreme knock-knees continues after 7 yrs of age
    b. Extreme bowing continues after 2yrs of age
    c. Parents are concerned about their child’s appearance
    d. Evident before 2yo
    b
  69. Tibial torsion is commonly associated with:
    a. Pain
    b. Restricted ROM
    c. Internal rotation of lower extremities
    d. Occurrence in adolescents 13-16 of age
    c
  70. Which of the following diagnoses is associated with contracture of one of the sternocleidomastoid muscles?
    a. Lordosis
    b. Torticollis
    c. Scoliosis
    d. Kyphosis
    b
  71. Sports injuries are commonly associated with:
    a. Improper training
    b. Higher frequency in females
    c. Scoliosis
    d. Low socioeconomic status
    a
  72. Initial treatment of a sprain includes which of the following?
    a. Rest, ice, compression, elevation, & NSAIDs
    b. Heat, ROM exercise, compression, elevation, & NSAIDs
    c. Rest, heat, compression, elevation, & NSAIDs
    d. Rest, ice, ibuprofen, compression, & NSAIDs
    a
  73. The most definitive features for a diagnosis of “growing pains” includes:
    a. Exclusion of other causes of lower extremity pain
    b. Pain, swelling, erythema
    c. Loss of ambulation
    d. Decreased ROM
    a
  74. Systemic-onset JIA is most commonly associated with:
    a. High, daily intermittent spiking fevers & rash
    b. Single joint involvement
    c. Positive RF factor
    d. Painless joint involvement
    a
  75. S/S associated with Duchenne muscular dystrophy are:
    a. History of delayed developmental milestones
    b. Visual motor disturbance, calf hypertrophy
    c. Delayed motor development, positive ortolani maneuver
    d. History of “clumsiness” visual-motor disturbance
    a
  76. Complications of SLE commonly include with of the following?
    a. Pericarditis, arthritis, nephritis
    b. Encephalitis, nephritis, pericarditis
    c. Nephritis, arthritis, rheumatic fever
    d. Nepthritis, hemolytic anemia, contact dermatitis
    a
  77. Which of the following children need an orthopedic referral?
    a. A 6yo with mild bowing of the lower legs
    b. A 6mo old with internal tibial torsion
    c. A 3 week old with equinovarus of feet
    d. A newborn with positive pavlik sign
    c
  78. Antonio is a newborn and the PNP notes on physical assessment that both his feet turn in. When attempting range of motion, she finds that both feet move relatively freely in all directions. Antonio has:
    a. Clubfoot
    b. Syndactyly
    c. Matarsus adductus
    d. Fracture of his feet
    c
  79. Which of the following is an appropriate goal for a child being treated for osteomyelitis?
    a. Prohibiting activities
    b. Complete course of antibiotic therapy
    c. Encouraging a low fat diet
    d. Restricting visitors
    b
  80. In a newborn, a diagnosis of hip dislocation is suspected when”
    a. +galeazzi, Barlow, and Ortolani
    b. wide hip abduction that is symmetric
    c. flaccidity of the left leg following extension of both legs with return to flexion
    d. tonic neck reflex in which the left leg is flexed
    a
  81. Which of the following statements is true regarding slipped capital femoral epiphysis?
    a. More common in females
    b. Generally occurs following severe sudden trauma
    c. Incidence more common in athletes
    d. The goal of treatment is to stabilize or improve the position of the femoral head
    d
  82. In Legg-Calve-Perthes disease, which of the following signs & symptoms are seen?
    a. Insidious onset of limp with knee & groin pain
    b. Sudden onset of limp & pain in lateral hip
    c. Fever & insidious onset of limp
    d. Afebrile and sudden onset of limp
    a
  83. Which of the following is true for idiopathic scoliosis, which occurs primarily in adolescents?
    a. Mild curves occur equally between the sexes
    b. Generally there is no family history
    c. Back pain is usually associated with curves of 35 degrees or greater
    d. Bracing is indicated for thoracic curves of 10-25 degrees
    a
  84. A typical febrile seizure is most likely to be the problem of which of the following children?
    a. A 1yo with otitis media & a fever of 104F
    b. A 3mo old with unequal pupils & bulging fontanels
    c. An 11yo with fever of 101F who is on valproic acid for seizure disorder
    d. A 5yo with bacterial meningitis
    a
  85. Which of the following responses during a tonic-clonic seizure is most important to teach family members of a child who has these seizures?
    a. Restrain the child
    b. Insert an airway into the mouth to prevent tongue biting
    c. Note the time, duration, & activity of the seizure
    d. Protect child from injury
    d
  86. Which of the following is an appropriate strategy to instruct the parents of an 18mo old child who has just been diagnosed with a febrile seizure and who has a fever of 104F?
    a. Give asprin every 4hrs for temp over 101F
    b. Bathe with tapid water
    c. Dress warmly to avoid chills
    d. First aid management during a seizure
    d
  87. What is the primary public health measure to decrease the incidence of bacterial meningitis in infants & children ?
    a. Keeping infants & young children away from crowded places
    b. Set up screening measures in schools
    c. Routinely giving immunizations such as Hib & PCV7
    d. Provide written information about meningitis to high school students
    c
  88. The type of seizure that first presents during infancy in which the infant goes into a “jack knife” posture is:
    a. Febrile seizures
    b. Infantile spasm
    c. Absence seizure
    d. Atonic “drop” attacks
    b
  89. Which of the following would not be associated with an uncomplicated concussion?
    a. Focal motor signs
    b. Brief loss of consciousness
    c. Headache
    d. Confusion or amnesia for the event
    a
  90. Which of the following factors is not usually present in a 4yo with migraine headaches?
    a. Complaints of pain in one side of the head only
    b. Positive family hx in immediate relatives
    c. Pain relieved by brief sleep
    d. Recurrent abdominal pain
    a
  91. The parents of a Sarah, a 13mo old, are very concerned after she experienced a 2nd simple febrile seizure. At today’s visit, they ask you about the use of medications to prevent any further seizures. Which of the following statements is true regarding febrile seizure prophylaxis?
    a. Short-term anticonvulstant prophylaxis with valprior acid is indicated after 3 febrile seizures
    b. Prolonged anticonvulsant prophylaxis with phenobabital for preventing recurrent febrile convulsions is now recommended
    c. Diazepam may be indicated for children <12mo who have recurrent, complex febrile seizures
    d. Fever management with Tylenol to keep the fever below 102F will prevent another seizure.
    c
  92. Problems associated with NF1 (neurofibromatosis) most likely require medical treatment to include the following?
    a. Learning disabilities
    b. Transient tics
    c. Obsessive-compulsive behavior
    d. Progressive mental retardation
    a
  93. An 11yo girl is brought in by her mother complaining of severe headaches associated with nausea & vomiting. Which of the following s/s would lead you to consider a brain tumor as part of your differential diagnosis?
    a. Throbbing pain accompanied by severe light sensitivity
    b. Bilateral throbbing pain
    c. Preceded by a visual aura
    d. More severe in the morning followed by vomiting
    d
  94. The National Institutes of Health Consensus conference on NF (neurofibromatosis) describes guidelines for diagnosis of NF. Which of the following is not among the diagnostic criteria?
    a. Family hx of first degree relative with NF1
    b. 6 or more “café au lait” spots >15 mm
    c. freckling in armpits or groin
    d. optic glioma
    b
  95. Although meningitis related mortality decreased dramatically after the neonatal period, as many as 50% of survivors experience some sequelae. Which of the following is the most frequent post-meningitis sequela in older infants and children?
    a. Motor deficits
    b. Seizures
    c. Language delays/disorders
    d. Visual impairments
    c
  96. A neonate is being worked up for meningitis after experiencing a seizure preceded by fever, irritability, and poor feeding for one day. On physical examination, which of the following findings would be most consistent with a diagnosis for meningitis?
    a. Bulging fontanel
    b. Positive Brudzinski’s sign
    c. Nuchal rigidity
    d. Positive Kernig’s sign
    a
  97. Diagnostic criteria for Tourette syndrome include which of the following?
    a. Must have both motor & vocal tics
    b. Steadily increasing pattern of motor tics
    c. Presence of any tic for 6 months
    d. Multiple tics with significant voluntary control
    a
  98. Which of the following statements about Tourette syndrome is not true?
    a. It is the most severe type of tic disorder
    b. It is more common among males than females
    c. Mean age of onset is between 6 & 7 yrs
    d. Symptoms decrease after puberty
    d
  99. A prominent feature in simple partial seizure is:
    a. Very short postictal drowsiness
    b. Microcephaly
    c. Bilateral tremors
    d. No loss of consciousness
    d
  100. Absence seizures:
    a. Often begin between 1 & 2 yrs of age
    b. Appear as altered awareness & blank stare for brief period
    c. More commonly occur in first-born children
    d. Usually progress to a more severe seizure disorder beyond childhood
    b
  101. The differential diagnosis for seizures includes which of the following?
    a. Poliomyelitis without paralysis
    b. Schizophrenia
    c. Complicated migraine headaches
    d. Multiple sclerosis
    c
  102. The most useful diagnostic tool for diagnosing epilepsy is:
    a. MRI
    b. EMG
    c. EEG
    d. CT scan
    c
  103. Which of the following is a primary drug used for treatment of generalized tonic-clonic seizures?
    a. Ethosuximide
    b. Valproic acid
    c. Clonazapam
    d. Corticosteroids
    b
  104. Which of the following is included in the diagnostic criteria for migraine headache in postpubertal children?
    a. Recurrent abdominal pain, throbbing head pain not relieved by sleep
    b. Nausea & vomiting, positive family history, aura
    c. Positive family history, vomiting without nausea, pain not relieved by sleep
    d. Aura, positive family history, pain in both hemispheres of the brain relieved by sleep
    b
  105. The type of head injury that is easily treatable but potentially the most lethal is:
    a. Concussion
    b. Epidural hematoma
    c. Acute subdural hematoma
    d. Linear skull fracture
    b
  106. Tuberous sclerosis is a progressive, neurocutaneous syndrome in which there is a combination of:
    a. Skin abnormalities, blindness, & cognitive deficits
    b. Skin abnormalities, seizures, cognitive deficits
    c. Cerebral palsy, cognitive deficits, & skin problems
    d. Skin abnormalities, blindness, & cognitive deficit
    b
  107. Inheritance of tuberous sclerosis is by:
    a. Autosomal dominant mutant gene
    b. Fragile x chromosome
    c. Autosomal recessive mutant gene
    d. Trisomy 23
    a
  108. The most important diagnostic measure for identifying tuberous sclerosis is:
    a. EEG
    b. Through neurological physical exam
    c. MRI
    d. Skin biopsy
    c
  109. For children 1-4 yrs of age, the most common cause of head injury is:
    a. Motor vehicle accidents
    b. Falls
    c. Child abuse
    d. Tricycle accidents
    b
  110. Dwayne, a senior in high school, suffers a concussion playing football. Immediately following the event, a typical clinical picture would include:
    a. Brief loss of consciousness and/or amnesia
    b. Confusion and lethargy lasting about 2hrs
    c. Slowed & sometime unequal pupils response
    d. Associated Battle sign
    a
  111. A head injury in which bruising or tearing of the cerebral structures occurs in a:
    a. Contusion
    b. Concussion
    c. Hematoma
    d. Diffuse axonal injury
    a
  112. The most common type of skull fracture in children with excellent prognosis is:
    a. Depressed
    b. Basilar
    c. Linear
    d. Simple
    c
  113. Which of the following is most likely to be the etiologic cause of meningitis in the newborn?
    a. Neisseria meningitis
    b. Escherichia coli
    c. Streptococcus pneumoniae
    d. Haemophilus influenzae B
    b
  114. Drug therapy commonly used to treat absence seizures, in addition to valproic acid, includes:
    a. Phenytoin
    b. Phenobarbital
    c. Carbamazpine
    d. benzodiazepines
    c
  115. Urinary tract infections (UTI) are the most common pediatric urinary tract problems seen in primary care. Which of the following statements is not true regarding UTIs?
    a. Symptoms are often nonspecific, especially in infancy
    b. Urine culture is required for definitive diagnosis
    c. Trimethoprim-sulfamethoxazole is a drug of choice for most children
    d. Radiologic studies are rarely indicated with first infection
    d
  116. A 2yr girl presents with symptoms of painful urination, frequency, & occasional incontinence over the past week. When seen in your office, she has a temperature of 101.6F. Which of the following would be your approach in establishing a definitive diagnosis?
    a. Clean-catch midstream collection of specimen for urine analysis
    b. Clean-catch midstream collection of specimen for urine culture
    c. Straight catheterization collection of specimen for urine culture
    d. Voiding cystourethrogram (VCUG)
    c
  117. The most likely organism to cause UTI in the pediatric population is:
    a. Staphylococcus saprophyticus
    b. Klebsiella
    c. Chlamydia
    d. E. coli
    d
  118. One of the most commonly suggested reasons for primary enuresis is:
    a. Certain medications, such as theophilline
    b. Genitourinary abnormalities
    c. Family disruptions and stress
    d. Delayed maturation of voiding inhibitory reflex
    d
  119. Suzanne, a 7yo comes to you for a physical exam prior to participation in soccer. Suzanne’s mother is concerned that the child “still has accidents at night.” You determine that Suzanne has primarily nocturnal enuresis and your first recommendation to her is to:
    a. Avoid use of criticism or punishment
    b. Use a sticker/star chart
    c. Treat with medication
    d. Purchase an enuresis alarm
    a
  120. The incidence of cryptorchidism at one year of age is about 1%. The best explanation for this is:
    a. Examination of the scrotum begins at this age
    b. A child can usually stand, making palpation of the testes easier
    c. Spontaneous resolution often occurs in first year
    d. Surgical repair can now be done in neonatal period
    c
  121. Communicating hydrocele is best differentiated from the noncommunicating type by the fact that:
    a. There is no association with hernia
    b. It usually resolves on its own
    c. The fluid is static in the scrotum
    d. Frequently develops into hernia
    d
  122. In counseling parents when their child is diagnosed with mild hypospadias, suggest that the following may be part of the management:
    a. Circumcision
    b. Radiography
    c. Consult with pediatric urology
    d. Surgical correction at 2yrs of age
    c
  123. On physical exam of a 2yo uncircumcised male, you note that the foreskin is retracted and discolored. There is swelling of the glans. The most likely diagnosis is
    a. Phimosis
    b. Balanitis
    c. UTI
    d. Paraphimosis
    d
  124. Meatal stenosis, narrowing of the distal urethra, is seen in the following:
    a. Orchiopexy
    b. Circumcision
    c. Epididymitis
    d. Hypospadias repair
    b
  125. During a track meet, a 14yo male pole vaulter falls to the ground screaming in pain. He complains of intense, searing pain in his right scrotum. He vomits twice while waiting for the ambulance. He most likely has:
    a. Orchitis
    b. Hydrocele
    c. Acute epididymitis
    d. Testicular torsion
    d
  126. Treatment for testicular torsion is primarily:
    a. Scrotal evaluation
    b. Ice
    c. Immediate surgical referral
    d. Bedrest
    c
  127. Labial adhesions are relatively common finding among infants & young girls. Which of the following statements about this condition is true?
    a. Adhesions are usually present at birth but may be missed on exam
    b. Highest incidence is from birth to 3yrs
    c. Simple lysis of adhesions is often recommended
    d. Most cases resolve without intervention
    d
  128. Which of the following statements is true regarding the use of topical application of conjugated estrogen cream with labial adhesions?
    a. It is highly successful in resolving most adhesions within 2 months
    b. It is no longer recommended because it may stimulate precocious puberty
    c. Topical applications of bland creams or petroleum jelly are equally effective
    d. Mechanical lysis is preferred treatment today
    a
  129. Sheryl, a 12yo complains of vaginal discharge for the past 8-9mo. She tells you her underpants are frequently wet. When she wipes after urinating, there is “white stuff” on the tissue. Sheryl denies urinary problems, genital itching, or odor. She also denies sexual activity. Her menses have not yet started, but she reports she “started to develop” in her breasts about the age of 10. Her vaginal discharge is most likely the result of:
    a. A fungal infection
    b. Poor hygiene
    c. Retained foreign body
    d. Physiologic leukorrhea
    d
  130. Your recommendations to Sheryl (with physiologic leucorrhea) regarding management of the discharge includes which of these?
    a. Vinegar & water douche
    b. Placing sanitary “mini pad” in her underpants
    c. A 10 day course of penicillin or erythromycin
    d. Use of monilial cream for 1 week
    b
  131. Which of the following is not true of dysmenorrhea?
    a. Onset is usually within the first 2-3 months following menarche
    b. A leading cause of school absenteesism in adolescent females
    c. Systemic symptoms include vomiting & dizziness
    d. Pain is from start of menses to about 24-48hrs later
    a
  132. Amy, a 16yo has symptoms of premenstrual syndrome. She refuses to take NSAIDS, preferring instead more “natural” treatments. Her options include:
    a. Eating foods rich in sodium & fat
    b. Including more foods ore supplements with vitamin c
    c. Limited fluid intake to avoid “bloating”
    d. Adequate rest, a healthy diet, and exercise
    d
  133. In cases of accidental genitourinary trauma, which of the following is not commonly seen?
    a. Extensive tears of the vaginal wall
    b. Hematuria
    c. Hematoma of the urethra, scrotum, lower abdomen
    d. Periurtehal lacerations
    a
  134. The most common form of glomerulonephritis in children is?
    a. Mesangial proliferative
    b. Poststreptococcal
    c. Memranoproliferative
    d. Mesangiostreptococcal
    b
  135. Which of the following s/s is not associated with acute forms of glomerulonephritis?
    a. Edema
    b. Hematuria
    c. Increased urine output
    d. Dark urine
    c
  136. One of your male patients presents with weight loss, abdominal pain, and decreased urine output. On exam, you palpate a right sided mass, noting tenderness in the abdomen & flank. Urinalysis reveals significant leukocytosis. An intravenous pyelogram is ordered which shows marked delay of emptying from renal pelvis. The most likely diagnosis is?
    a. Glomerulonephritis
    b. Pyelonephritis
    c. Hydronephrosis
    d. UTI
    c
  137. Which of the following is true of renal tubular acidosis type 1?
    a. Genetically transmitted as autosomal recessive disorder
    b. Distal tube defect affecting secretion of hydrogen ions
    c. Distal tube defect affecting bicarb reabsoption
    d. Most children remain short in stature in spite of early treatment
    b
  138. Vulvovaginitis may be caused by all of the following except:
    a. Poor hygiene
    b. Herpes simplex virus
    c. Pinworms
    d. Condylomata acuminata
    d
  139. Meatal stenosis can be identified by all of these with the exception of?
    a. Crying with urination
    b. Inflammation of glans penis
    c. Slit-like meatus
    d. Wide urinary stream
    d
  140. Trina, age 14, comes in with complains of headache & nausea. She admits to having a boyfriend, whom she has sex with “once in a while”, and sometimes he uses a condom. She had a period last month, but it lasted 2 days instead of the usual seven. Which of the following is not usually perceived as risk factors for adolescent pregnancy?
    a. Early onset of sexual activity
    b. Familiarity with fertility knowledge
    c. Sporatic, if any, contraceptive use
    d. Low self-image
    b
  141. Trina’s urine pregnancy test is positive and confirmed by serology. Which of the following physical findings is not consistent with gestation of less than 12 weeks?
    a. Hegar sign
    b. Goodell sign
    c. Doppler auscultation of fetal heart sounds
    d. 4-6lb weight gain
    d
  142. Congenital defects are present in approximately 3-5% of infants born in this country. Which of the following factors is the lowest contributor to defects?
    a. Genetic
    b. Unknown
    c. Environmental
    d. Combination of environmental & genetic
    b
  143. Which of the following is the earliest screening test that you would use in managing Trina’s care during her first trimester of pregnancy?
    a. Amniocentesis
    b. Chorionic villus sampling
    c. Serum alpha-fetoprotein
    d. Fetal radiograph
    c
  144. Trina expresses to you that she may want to terminate the pregnancy (gestation <12 weeks). She has heard that abortion is a simple procedure, without complications. Your discussion with her is based on the fact that:
    a. Adolescents only choose to terminate pregnancy 10% of the time
    b. Menstrual extraction could be done at this point
    c. The risk is high for hemorrhage & fever
    d. Cervical injury overall from induced abortion is rare
    b
  145. You know that if Trina delivers her baby, close follow up of both is important. What is not a focus of care in the first weeks after delivery?
    a. Social support for the family
    b. Mother’s goals for education
    c. Infant’s continued weight gain
    d. Adaptation of mother to sleep changes
    b
  146. While child abuse occurs across age barriers, the adolescent parent may be at greater risk for abusing their child because:
    a. The pregnancy may be unplanned or unwanted
    b. Adolescents continue substance abuse after baby is born
    c. The infant mortality rate is low
    d. Education becomes more important that parenting
    a
  147. A thick, purulent vaginal discharge that is greenish-yellow is most likely an infection caused by:
    a. Chlamydia trachomatis
    b. Herpes simplex virus
    c. Neisseria gonorrhea
    d. Human papilomavirus
    c
  148. The primary treatment for neisseria gonorrhea would be?
    a. Amoxicillin
    b. Ceftriaxone
    c. Ofloxacin
    d. Penicillin
    b
  149. The primary treatment for neisseria gonorrhea would be?
    a. Amoxicillin
    b. Ceftriaxone
    c. Ofloxacin
    d. Penicillin
    c
  150. Which of the following tests provide the most definitive diagnosis for suspected syphilis?
    a. VDRL
    b. ART
    c. Dark-field microscopy
    d. FTA-Abs
    c
  151. Which of the following is not true regarding the use of acyclovir in the treatment of herpes simplex virus?
    a. Topical tx is not recommended
    b. Treatment is equally effective for active & recurrent lesions
    c. The focus of treatment is decrease in intensity symptom duration & viral shedding
    d. Therapy is best initiated within 6 days of onset of lesions
    b
  152. Your 15yo female patient presents with genital lesions. Your workup is based on the knowledge that:
    a. HPV is often associated with malignancy
    b. HPV manifests itself as molluscum contagiosum
    c. Most HPV infections cause genital itching & pain
    d. Syphilis serology should be included to distinguish HPV from condylomata lata
    d
  153. After your exam, you determine that your patient has condlyomata acuminata. Which of the following is not commonly used to treat these lesions?
    a. Topical podophyllum resin
    b. Laser treatment for unresponsive lesions
    c. Topical acyclovir
    d. Treatment is often not needed; many lesions regress spontaneously
    c
  154. During your eval of Lisa, a 16yo female pt, she relates recent, first time sexual intercourse with her boyfriend. Now she complains of “frothy” substance in her underwear that “smells weird”. She also has slight itching in her vaginal area. You suspect?
    a. Trichomoniasis
    b. Gonorrhea
    c. Chlamydia
    d. Herpes
    a
  155. What education needs to be included in prescribing metronidazole to treat trichomoniasis?
    a. It is safe for use during pregnancy
    b. Sexual contact may resume after 48hrs of treatment
    c. Sexual contacts do not require therapy
    d. Alcohol should not be used during treatment
    d
  156. A patient with a thin, white, malodorous vaginal discharge most likely has?
    a. Trichomoniasis
    b. Monilia
    c. Bacterial vaginosis
    d. Chlamydia
    c
  157. At which point of life is UTI more common in males?
    a. Toddler
    b. Adolescent
    c. School age
    d. Newborn/infant
    d
  158. Most patients with uncomplicated UTI can be treated on an outpatient basis. The first antibiotic you would consider using is?
    a. Amoxicillin
    b. Trimethoprim-sulfamethoxazole
    c. Cephalexin
    d. Amoxicillin/clavulanate
    b
  159. Cryptorchidism is more prevalent in:
    a. Term infants
    b. Premature infants
    c. Babies at 1 year of age
    d. Toddlers
    b
  160. A 3yo boy is in the process of toilet training. The parents come to see you because when the child urinates, all of the urine goes on the floor. You suspect hypospadias because:
    a. It is a rare disorder and would indicate a more serious problem
    b. Normally the urine stream is directed from the meatal opening at the tip of the penis in a straight path
    c. The child is circumcised
    d. Fusion of the urethral folds has occurred
    b
  161. Which of the following is not used in the management of the child with phimosis?
    a. Ice packs
    b. Gentle stretching when bathing
    c. Circumcision in cases of urinary obstruction
    d. Good hygiene
    a
  162. Jeanine, an 8yr girl presents with a 2 week history of brownish-red, very foul smelling, vaginal discharge. It is most likely caused by:
    a. Sexual abuse
    b. Accidental genital trauma
    c. Foreign body
    d. Poor hygiene
    c
  163. Which of the following is not a proposed etiologic factor in premenstrual syndrome?
    a. Imbalance of water & sodium
    b. Fluctuation in steroids
    c. Vitamin B12 deficiency
    d. Synthesis of prostaglandins
    c
  164. Treatment of a child with glomerulonephritis would include:
    a. Vasoconstrictors for hypotension
    b. Antibiotics for persistent infection
    c. Increased fluids to maintain hydration
    d. Avoidance of diuretics
    b
  165. Which of the following Is not a frequent sign of a gonorrhea infection:
    a. Diarrhea
    b. Urinary symptoms
    c. Menstrual problems
    d. Fever
    a
  166. The most significant finding in the case of Chlamydia infection is:
    a. The vaginal discharge is gray & frothy
    b. Most examinations are normal
    c. The external genitalia is markedly inflamed
    d. Infection by Chlamydia trachomatis alone is the rule
    b
  167. One difference between the lesions of primary syphilis (chancre) and HSV genital lesions that may help in differentiating the two is:
    a. Syphilis chancre is painless while HSV lesions are painful
    b. Syphilis lesions have a flat edge while HSV lesion edges are more raised
    c. Syphilis chancres are more likely to ulcerate than HSV lesions
    d. Erythema & edema is greater with syphilis chancre than HSV lesions
    a
  168. Education of sexually active adolescents concerning use of condoms to prevent STI’s would include:
    a. Only latex or natural skin condoms should be used
    b. Polyurethane condoms have inadequate STI protection
    c. Condom use is only method to prevent STIs & AIDS
    d. Only petroleum-based lubricants should be used
    c
  169. Which of the following contraceptive methods would be least suitable for adolescents?
    a. Condoms with spermicides
    b. Progestin-only mini pill
    c. Long-acting progestins
    d. Estrogen-progestin combination
    b
  170. Which of the following is not an absolute contraindication for oral contraceptive?
    a. Clotting disorder
    b. Impaired liver function
    c. Severe HTN
    d. Undiagnosed vaginal bleeding
    c
  171. Baseline management of all neonates with ABO incompatibility includes:
    a. Phototherapy
    b. Serial monitoring of bilirubin & hemoglobin levels
    c. Exchange transfusion
    d. Simple transfusion of packed RBCs
    b
  172. Which of the following is not associated with Rh incompatibility?
    a. Mother Rh negative, baby Rh positive
    b. Mother Rh positive, baby Rh negative
    c. More severe in subsequent sensitized pregnancies
    d. Hemolysis may occur up to 6 weeks or more
    b
  173. Clinical jaundice of the distal extremities would be noted at a bilirubin level of:
    a. <5mg/dl b. 5mg/dl c. 10mg/dl d. >15mg/dl
    d
  174. beta-chain synthesis is absent in:
    a. beta-thalassemia minor
    b. beta-thalassemia intermedia
    c. beta-thalassemia major
    d. alpha-thalassemia trait
    c
  175. which of the following are most often associated with hemoglobin C?
    a. growth retardation
    b. hepatosplenomegaly
    c. usually asymptomatic
    d. frontal bossing
    c
  176. Diagnostic findings consistent with beta-thalassemia are:
    a. Hemoglobin-normal
    b. Reticulocytes-normal
    c. Hgb A2 <3.6
    d. Hypochromia, microcytosis
    d
  177. Asplenic children are at increased risk for which of the following?
    a. Bacterial infections
    b. Fungal infections
    c. Viral infections
    d. Parasites
    a
  178. Which of the following is not considered preventative management for iron deficiency anemia?
    a. Iron fortified cereal from 6-12mo of age
    b. Iron fortified formula until 6mo of age
    c. No cow’s milk until 1yr of age
    d. If breastfeeding, supplemental iron drops or iron fortified cereal by 4-5mo of age
    b
  179. The expected clinical severity of hemoglobin sickle C disease (Hgb SC) is:
    a. Asymptomatic
    b. Marked to moderate
    c. Mild to moderate
    d. Severe
    c
  180. The expected hemoglobin range for sickle cell anemia is:
    a. 6.5-9.5g/dl
    b. 13.5-16.5g/dl
    c. 8.5-12.5g/dl
    d. 9.5-13.5g/dl
    a
  181. prophylactic penicillin should be initiated in children with sickle cell anemia by:
    a. 3yrs of age
    b. 12mo of age
    c. 2-3mo of age
    d. 9mo of age
    c
  182. Hemolysis does not contribute to which of the problems associated with sickle cell disease?
    a. Chronic anemia
    b. Splenic sequestration
    c. Aplastic crisis
    d. Delayed growth
    b
  183. the following blood level is not considered lead poisoning?
    a. <10ug/dl b. 10-14ug/dl c. >15ug/dl
    d. >25ug/dl
    a
  184. Which of the following is not a precipitating factor for hemolysis in G6PD deficiency?
    a. Drugs
    b. Exposure to extreme temps
    c. Ingestion of fava beans
    d. Infection
    b
  185. What percent of factor VII/IX is associated with severe hemophilia A & B?
    a. >1
    b. 1-5
    c. 5-25
    d. 30-50
    a
  186. What type of hemorrhage would be expected with severe factor VIII deficiency?
    a. Severe hemorrhage following moderate to severe trauma
    b. Gross bleeding following mild to moderate trauma
    c. Gynecologic hemorrhage
    d. Spontaneous hemarthosis
    d
  187. Which of the following is the most common type of congenital bleeding disorder?
    a. Hemophilia A
    b. Hemophilia b
    c. Von Willebrand Disease
    d. Idiopathic thrombocytopenia purpura
    c
  188. Which of the following medications should be avoided in a child with ITP?
    a. Decongestants
    b. Aspirin
    c. Acetaminophen
    d. Sulfa drugs
    b
  189. The following test is required to diagnose leukemia:
    a. Cbc with diff
    b. Bone marrow aspiration/biopsy
    c. Chest radiograph
    d. Biopsy of an enlarged lymph node
    b
  190. Which of the following is not included as part of the initial therapy for ALL?
    a. Chemotherapy
    b. Radiation therapy
    c. Bone marrow transplant
    d. Intrathecal chemotherapy
    c
  191. Which malignancy is associated with genitourinary anomalies?
    a. Acute lymphocytic leukemia
    b. Chronic myelogenous leukemia
    c. Osteosarcoma
    d. Wilm’s tumor
    d
  192. Which of the following statements are true about immunizations during treatment of childhood cancer?
    a. Children continue to receive immunizations as usual
    b. Immunizations are not given during active chemotherapy
    c. Only live vaccines are held during active chemotherapy
    d. No family member should be immunized while the child is receiving chemotherapy
    b
  193. The peak incidence of osteosarcoma is
    a. 4-7 years of age
    b. 8-11 yrs of age
    c. 12-14yrs of age
    d. 15-19yrs of age
    d
  194. the following type of infection is not associated with hypogammaglobulinemia:
    a. sinusitis
    b. pneumonia
    c. urinary tract infection
    d. cellulitis
    c
  195. the following diagnostic finding is consistent with X-linked agammaglobulinemia
    a. IgG-normal
    b. B-cells-decreased
    c. T-cells-decreased
    d. IgA-normal
    b
  196. The following is not a characteristic feature of DiGeorge syndrome:
    a. Hypertelorism
    b. Cleft palate
    c. Cardiac defect
    d. Frontal bossing
    d
  197. The following diagnostic finding is consistent with Wiskott-Aldrich syndrome:
    a. IgG-normal
    b. IgA-decreased
    c. IgM-increased
    d. B cells-decreased
    a
  198. Management of a patient with splenectomy does not include:
    a. Pneumococcal vaccines at least 2 weeks prior to surgery
    b. Prophylactic penicillin
    c. Blood culture & parenteral antibiotics for febrile illnesses
    d. Treating fever with antipyretics only & observing for resolution
    d
  199. Secondary hypothyroidism results from:
    a. Excess release of thyroid hormone beyond the newborn period
    b. Intrauterine exposure to thyrotoxic drugs
    c. Disease or disorder of the thyroid gland itself
    d. Disease or disorder of the hypothalamus or pituitary gland compromising thyroid function
    d
  200. Congenital hypothyroidism has a higher incidence in which of the following populations?
    a. African-americans
    b. Hispanic & native Americans
    c. Asian-americans
    d. Euro-americans
    b
  201. Which of the following is not a sign or symptom of congenital hypothyroidism?
    a. Hoarse cry
    b. Frequent stooling
    c. Coarse features
    d. Lethargy
    b
  202. The most common cause of hyperthyroidism in children & adolescents is:
    a. Graves (autoimmune disease)
    b. Thyroid cancer
    c. Thyroid nodules
    d. Pituitary tumor
    a
  203. Which of the following is not found in an adolescent with untreated Grave’s disease?
    a. Behavioral problems
    b. Sleep disturbances
    c. Tendency to gain weight easily
    d. Tachycardia
    c
  204. In which of the following children would you most suspect hyperthyroidism?
    a. A 16yo male who complains about restlessness
    b. A 14yo adolescent female who is heat intolerant & has amenorrhea
    c. A male preteen with behavior problems
    d. A 6yo female who complains of tiredness
    b
  205. The most common thyroiditis is:
    a. Subacute thyroiditis caused by a viral infection of the gland
    b. Acute suppurative thryoiditis caused by bacterial infection
    c. Caused by exposure to radiation or trauma
    d. Hashimoto’s or chronic autoimmune thyroiditis
    d
  206. Nephrogenic, or vasopressin-resistant, diabetes insipidus:
    a. Is caused by anatomic defects in the brain causing hypofunction of the pituitary or hypothalamus
    b. Results from damage to the hypothalamus or pituitary from surgical trauma or infection
    c. Caused by reduced renal responsiveness to antidiuretic hormone (ADH)
    d. Has oliguria as a primary presenting symptom
    c
  207. An infant with polydipsia, polyuria, irritability, and failure to thrive, should be evaluated for:
    a. Diabetes insipidus
    b. Homocystinuria
    c. Growth hormone deficiency
    d. Hyperglycemia
    a
  208. Which one of the following is not characteristic of constitutional growth delay?
    a. There is generally no hx of a similar growth pattern in other family members
    b. The child usually remains constitutionally small as an adult
    c. Final adult stature tends to be normal
    d. Weight & height at birth are generally in the lower percentiles
    c
  209. A newborn or infant with birth length <50% and microphallus should be suspected of having:
    a. Growth hormone deficiency
    b. Congenital hypothyroidism
    c. Primordial short stature
    d. Down syndrome
    a
  210. An adolescent male who fails to develop secondary sex characteristics at puberty and who has small, underdeveloped testes should be suspected of having:
    a. Adrenal hyperplasia
    b. Klinefelter’s syndrome
    c. Marfan syndrome
    d. Cerebral gigantism (sotos syndrome)
    b
  211. Individuals with chronic adrenal insufficiency often have:
    a. Frequent otitis media
    b. High energy levels
    c. Love for physical activity
    d. A craving for salt
    d
  212. In the newborn period, infants of diabetic mothers (IDMs) are particularly at risk for:
    a. Small size for gestation age
    b. Intrauterine growth retardation (IUGR)
    c. Disorders in bone development
    d. Hypoglycemia
    d
  213. Which statement is true about true (complete) precocity or incomplete (pseudoprecocity)
    a. True precocity occurs because of hormonal stimulation from the pituitary or hypothalamus causing gonadal maturation & fertility
    b. Pseudoprecocity does not involve development of any secondary sex characteristics
    c. Incomplete precocity is caused by adrenal or gonadal tumor or dysfunction & results in increased lineal growth but no development of secondary sex charateristics
    d. Incomplete or pseudoprecocity leads to testicular enlargement and ovulation
    a
  214. An adolescent who has tall stature, increased arm span, arachnodactyly, laxity of joints, pectus excavatum, and abnormal echocardiogram would be suspected of having:
    a. Turner’s
    b. Beckwith-Wiedemann syndrome
    c. Marfan
    d. Klinefelter’s syndrome
    c
  215. Which one of the following is not found in children with growth hormone excess?
    a. Tall stature
    b. Prominent mandible & supraorbital ridge
    c. High or normal plasma growth hormone
    d. Short stature
    d
  216. A pathognomonic skin finding in children with chronic adrenal insufficiency (Addison’s) is:
    a. Purple striae
    b. Increased pigmentation in the axilla, groin, areola, hand creases, & in surgical scars
    c. Dry, thickened skin
    d. Increased perspiration
    b
  217. Which of the following findings is not characteristic of children and infants with hyperadrenocortism?
    a. Advanced skeletal maturation
    b. “moon” facies
    c. delayed onset of secondary sex characteristics
    d. “buffalo type” adiposity of face, neck, & trunk
    a
  218. transient neonatal hypoglycemia is:
    a. most common in AGA infants
    b. low in premature SGA infants
    c. most common in LGA infants
    d. least common in LGA infants
    c
  219. regular insulin
    a. has a quicker onset of effect and longer duration than NPH
    b. has slower onset of effect & shorter duration than NPH
    c. has quicker onset of effect & shorter duration than NPH
    d. has the longest duration of the insulins available
    c
  220. the preferred name now for insulin dependent diabetes mellitus (IDDM) is:
    a. maturity-onset diabetes
    b. type 1 diabetes
    c. type 2 diabetes
    d. insulin resistance syndrome
    b
  221. blood glucose levels of children 5-11yo with diabetes should be maintained between:
    a. 70-180mg/dl
    b. 100-200mg/dl
    c. 60-80 mg/dl
    d. slightly over 200mg/dl
    a
  222. blood glucose levels of younger children with diabetes are maintained at slightly higher levels than blood glucose levels of older children because:
    a. children have a greater need for available glucose in the blood system
    b. younger children tend to be more active
    c. younger children become more irritable than do older children
    d. lowering the risk of hypoglycemia in younger children is particularly important in order to avoid the potential for hypoglycemia with consequent neurological system damage
    d
  223. glucagon should be used to treat:
    a. children with mild hypoglycemia
    b. children with moderate hypoglycemia
    c. children with severe hyperglycemia
    d. children with severe hypoglycemia
    d
  224. which finding is not a sign of symptom of diabetes onset in children?
    a. Alopecia
    b. Glycosuria
    c. Polydipsia
    d. Polyuria
    a
  225. Abdominal pain & vomiting are particularly critical to monitor in children with diabetes because these findings may represent the onset of:
    a. Ketoacidosis
    b. Gastrointestinal infection
    c. Hyperglycemia
    d. Autoimmune response to the pancreas
    a
  226. Which of the following statements is not true about type 1 diabetes?
    a. The “honeymoon” period postdiagnosis is of variable duration
    b. Diabetes is relatively common disease in childhood
    c. Children with type 1 diabetes can switch to oral insulin agents once they reach adulthood
    d. 3 factors influence a child’s potential to develop diabetes-genetic predisposition, autoimmune response, & exposure to viral or chemical agents
    c
  227. precocious puberty development is defined as the development of secondary sexual characteristics in boys before age_years and menses in girls before the age of _yrs.
    a. 10yrs; 10yrs
    b. 6yrs; 8yrs
    c. 9yrs; 9.5yrs
    d. 6yrs; 9yrs
    c
  228. in boys, lack of secondary sexual characteristics after 17yrs suggests:
    a. castration
    b. abnormal testicular fx
    c. true hermaphroditism
    d. pituitary adenoma
    b
  229. the peak incidence for adolescent gynecomastia occurs at age:
    a. 10yrs
    b. 13-14yrs
    c. 16yrs
    d. 18yrs
    b
  230. the most common cause of primary amenorrhea is:
    a. obstructions of menstrual flow
    b. primary ovarian insufficiency
    c. secondary ovarian insufficiency
    d. constitutional or familial
    d
  231. which of the following is a characteristic physical sign of fragile X syndrome in adolescent males?
    a. Small posteriorly rotated ears
    b. Macroorchidism
    c. Hypertonia
    d. Double hair whirl
    b
  232. Which of the following physical stigmata are common in newborns with Down syndrome?
    a. Microcephaly, large ears & mouth, flattened philtrum
    b. Hypotonia, large appearing tongue & small mouth, upward slant to eyes
    c. Fair mottled skin, large hands & feet, broad stocky neck
    d. Funnel or pigeon-breasted chest, Brushfield spots, extra digits
    b
  233. a 2mo infant, with hx of sacral myelomeningocele repair, has an increase in head circumference from the 75th to the 95th percentile. What is the most appropriate first action?
    a. Order a stat head CT scan
    b. Refer to neurosurgery for management
    c. Recheck it at the next well-child visit
    d. Recheck & replot the child’s head circumference
    d
  234. a 2 week old presents with mucopurulent eye discharge with injection and edema of the conjunctiva. The 15yr old mother had no prenatal care. What is the most likely organism?
    a. Staphylococcus aureus
    b. Gonococcus
    c. C. pneumoniae
    d. C. trachomatis
    d
  235. a newborn presents with a large VSD, rocker bottom feet, overlapping second & third fingers, & fourth & fifth fingers with hypotonia. What is the most likely diagnosis?
    a. Fragile X syndrome
    b. Down syndrome
    c. Edwards Syndrome
    d. Klinefelter’s syndrome
    c
  236. Which of the following problems is common in a child with Hurler’s syndrome?
    a. Developmental delay from birth
    b. Sleep obstructive apnea
    c. Ectopic lentis
    d. Congenital heart disease
    b
  237. Young infants with cerebral palsy often show:
    a. Voracious appetite & weight gain
    b. Increased muscle tone in the first weeks of life
    c. Hypotonia in the first weeks of life
    d. Unusually severe reactions to their first immunizations
    c
  238. A 5yo child is born at 28 weeks gestation has mild spastic diplegia. Which evaluation is important before the child is placed in regular kindergarten?
    a. Stanford-binet IQ test
    b. Carey scale of temperament
    c. CBC with diff
    d. Chromosome studies
    a
  239. A mother with Marfan syndrome comes with her 5yo for a checkup. The child has myopia & a + wrist & thumb sign. She reports that she was told by her last health care provider not to worry about this child. What is the next best step?
    a. Raise the issue of a genetic referral
    b. Reassure her
    c. Follow the child for further signs of Marfan syndrome
    d. Refer to ophthalmology
    a
  240. Which of the following is appropriate advice for the mother of a newborn?
    a. The child should sleep on her back
    b. The child should sleep on her back or side
    c. The child should be allowed to sleep in a carseat at night
    d. The child can sleep on her comforter
    a
  241. A 5yo female presents for a school physical with a complaint of hyperactivity, a mild developmental delay, aversion of gaze, hand mannerism, long thing face with slightly dysmorphic ear. What is the likely diagnosis?
    a. Fragile X
    b. Turner’s Syndrome
    c. Fetal alcohol syndrome
    d. Williams syndrome
    a
  242. Congenital HIV infection:
    a. Is diagnosed through finding maternal antibodies in infant serum
    b. Does not respond to antiretroviral therapy
    c. May be latent for years before clinical signs develop
    d. Does not include lymphadenopathy as a physical finding
    c
  243. an infant presents with cataracts, congetinal glaucoma, congenital heart disease, hepatosplenomegaly, thrombocytopenia, blueberry muffin rash, & growth retardation. What is the most likely diagnosis?
    a. Cytomegalovirus
    b. Taxoplasmosis
    c. Rubella syndrome
    d. Syphilis
    c
  244. A well-appearing, well dressed 11mo old has a long philtrum, midface hypoplasia, microcephaly, mild developmental delay, & myopia. The mother denies drinking alcohol during the pregnancy. What is the next best step?
    a. Refer the mother-child to child protective services
    b. Refer the child to early intervention
    c. Refer the child to cardiology
    d. Refer the mother to alcoholics anonymous
    b
  245. An 18mo old child exposed to HIV in utero had negative HIV DNA at 2 weeks and 8 weeks. What is indicated at the 18-month visit?
    a. No further testing is needed
    b. HIV DNA should be repeated
    c. Enzyme immunoassay for antibody to HIV-1
    d. CBC with diff & immunoglobulins
    c
  246. Which of the following infant diseases is not associated with exposure in the first trimester of pregnancy?
    a. Cytomegalic inclusion disease (CMV)
    b. Chlamydia conjunctivitis
    c. Taxoplasmosis
    d. Rubella
    b
  247. A baby is born to a mother with chronic hepatitis B. what is the best treatment approach?
    a. Administer interferon within 24hrs
    b. Administer hepatitis B vaccine within 24hrs
    c. Administer interferon & nucleotide in combination daily
    d. Administer HBIG & Hepatitis B vaccine within 12 hours
    d
  248. a 2yo female has lymphedema of the hands & foot, with low posterior hairline, cubitus valgus, & a hx of intrauterine growth retardation. Which of the following defects is the most common among the children with this defect?
    a. Aortic valve stenosis
    b. Mitral valve prolapse
    c. Dissecting aortic aneurysm
    d. Coarctation of the aorta
    d
  249. Joshua is the 9mo old infant son of parents of Lousinana-French descent. The child stopped rolling over. Mother reports he is increasingly irritable. Which of the following physical exam findings is most consistent with Tay-Sachs disease?
    a. Cardiomyopathy
    b. Retinal detachment
    c. “cherry red” spot on retina
    d. hepatomegaly
    c
  250. Prader-Willi syndrome is a congenital genetic disorder characterized by:
    a. Failure to thrive
    b. 100% detection rate with chromosome analysis for a 15q deletion
    c. emergence of spasticity during toddler years
    d. voracious appetite & development of obesity
    d
  251. A 15 day old infant with respiratory distress arrives in the ED. The exam reveals mild cyanosis, hepatosplenomegaly, & features consistent with Down syndrome. Which of the following is the most likely diagnosis?
    a. Complete AV canal (endocardial cushion defect)
    b. Patent ductus arteriosus
    c. Atrial septal defect
    d. Ventricular septal defect
    a
  252. A newborn presents with lymphadenopathy, a decrease in the ability to move the left leg, Coombs-negative hemolytic anemia, hepatomegaly, & snuffles. What is the most likely diagnosis?
    a. Congenital herpes infection
    b. Congenital cytomegalovirus infection
    c. Congenital syphilis
    d. Congenital gonococcal infection
    c
  253. Individuals who have spina bifida are at high risk for allergy to:
    a. Eggs
    b. Pollens
    c. Latex
    d. Dust mite feces
    c
  254. What is the preferred treatment of choice for syphilis?
    a. Erythromycin
    b. Penicillin
    c. Cefotaxime
    d. Zithromax
    b
  255. Which of the following is consistent with neonatal disseminated herpes disease?
    a. Hyperactive newborn with apparent spasticity
    b. Multiple papules scattered over the body
    c. Fever, grouped vesicles on the skin
    d. Purpuric rash in the acral distribution
    c

Leave a Comment

Scroll to Top